telling a coworker “that’s none of your business,” my team is mocking a coworker’s virginity, and more

It’s five answers to five questions. Here we go…

1. My coworker asks questions that are none of her business

I have a coworker who frequently asks me questions that aren’t a big deal but aren’t any of her business either. We’re on the same team, have equal responsibilities, and report to the same manager, but I get the sense from the questions she’s trying to position herself as higher up than me. Most recent example: I sent out an email informing the team of a change to my hours and she chatted me to ask, “Did [Boss] approve your schedule change?” I’ve started to reply “Why do you ask?” to these requests but almost always get a “just checking” or similar non-response.

It feels aggressive to answer such a simple question with, like, “Unless it affects you, I’d prefer to keep conversations about my schedule between me and Boss” when I could easily say “Yup, Boss approved” but it’s starting to really grate on me when she does this, especially since it happens multiple times a day. Is there a polite way to say, “Actually it’s not your responsibility to worry about that!” or should I just deal with my annoyance and answer her questions?

“Why do you ask?” is a good response — and when she responds with “just checking,” you can reply with, “I’ve got it covered.” Or skip the “why do you ask?” and go straight to “I’ve got it covered” or “Oh, I don’t need any help with that” or “I’m handling that with Boss” or anything else that declines to offer up info she’s not entitled to.

At some point it might also make sense to address the pattern by saying something like, “You’ve been asking me a lot of questions lately that I only expect from Boss — things like X and Y. Those aren’t things I’d normally report to a peer on, so I wondered why you’ve been asking.” And then if she says she’s “just checking,” you could say, “Yes, but why are you checking on those things when we are peers?”

2. My team is mocking a coworker’s virginity

A few months ago, one of my coworkers somehow found a video of another coworker (Bill) giving a religious testimonial for his church. In the video he speaks about why he is saving himself for marriage and why the church is against premarital sex. Bill never brings up his church or religion at work and I didn’t even know he was religious.

My coworkers have been relentlessly mocking Bill over the video. No one has done it directly in front of him that I know of but it’s prevalent enough that he is aware of the mocking and laughter going on. He hasn’t said anything but I can tell he is crushed. He doesn’t have a mean bone in his body, but ever since this started he is not his usual happy-go-lucky self. I tell people to stop if they do it in front of me, but then I get told it’s just some fun or to lighten up. Is there anything I can do to get people to stop? I feel awful about this.

Are your coworkers children? This is ridiculous — mean-spirited, immature, and generally horrible. Bill’s sexual choices are none of their business.

It also qualifies as religious harassment, which your employer has a legal obligation to put a stop to. So the most effective step would be to report to your company that your coworkers are creating a hostile workplace toward Bill because of his religious beliefs. You have the standing to do that yourself since you’re being exposed to it as well; the complaint doesn’t need to come from Bill. (Although even if you weren’t being exposed to it and just heard about it secondhand, you’d still have standing to report it, simply as someone who doesn’t want religious harassment occurring in your workplace and who assumes your company would want to be aware, since they’ll have legal liability if they don’t intervene.)

Your coworkers suck.

3. Can I apply for another job in the company a few weeks after being hired?

I started a new position last month at a large organization. I took the position as a foot in the door, as it’s extremely difficult to get an offer as an external candidate. I know this position isn’t long-term for me. This week, the organization posted *the perfect* position for me. If it weren’t for my integrity, the sick feeling in my stomach that I’m letting someone down, and my fear of establishing a reputation as a job-hopper, I’d apply in a heartbeat. There is a culture in this organization for gaining education and experience and moving into other positions, though. How inappropriate is it to inquire, apply, and/or accept a different position when you’ve only been with the organization for a few weeks?

This is the problem with taking a job just to get your foot in the door: you really need to stay in it for a while before you can try to move out of it. Not necessarily years, but in most cases an absolute minimum of six months (and in a lot organizations, closer to a year). Applying for a different job a few weeks after starting is highly likely to alarm your manager — they’ll have cut loose their other candidates and invested time and energy in training you, and you’ll come across as oblivious to that, as well as not particularly interested in staying. (To be clear, there’s always a risk someone could leave for a different job soon after starting — but by applying internally, you’re signaling that you think the company wouldn’t care and they generally will.)

Some organizations have formal policies about how long you need to be in a job before you can apply for a different position there, but either way it’s likely to go over really poorly with your manager.

There are some exceptions to this, like if your skills happen to perfectly meet a need they have and which they won’t be able to easily meet otherwise — in other words, if there would be a significant benefit to them, not just to you — but that’s the exception to the rule.

4. What’s with candidates not bringing anything to take notes on?

I’ve been interviewing new grad and intern candidates for my team for the past several years. During the height of the pandemic we transitioned to virtual interviews and it worked well. As we’ve returned to in-person interviews, I’ve seen more and more early career candidates than before the pandemic show up with nothing but the clothes on their back and a smile.

Nothing to write on or with, no copies of their resume. I am someone who hasn’t touched a piece of paper in years (I take all my notes electronically on a laptop or tablet) and so it’s not the pen/paper aspect that’s so weird to me, it’s the empty-handedness.

To be clear, it’s only happened a handful of times, it’s not a deal-breaker, and I don’t want to generalize about a generation that is a) younger and still learning professional norms and b) significantly impacted as a whole by the pandemic in professional growth opportunities and availability of mentorship.

I was reflecting on why it bothered me so much and I realized that so long as they bring something, I don’t really notice or care if they take their own notes. I always have a PDF of their resume and so I don’t even need them to bring that. I wonder if I’m overreacting or adhering to professional norms for tradition’s sake, which is not how I want to operate. Is there a good, logical reason for this expectation that I’m not thinking of? Should I care about this at all?

If so, is it appropriate to give that feedback to candidates that we end up passing on for other reasons? Obviously if we end up hiring them, we teach them to be ready to take notes in all meetings as it’s an expectation in our firm/industry.

It’s mostly inexperience — but I see some experienced candidates do this too, not just younger ones, so it’s not entirely inexperience. (For the older ones, I suspect it’s just that no one has ever told them to do it differently. Not everyone is exposed to the same job-search advice, and if you haven’t been on the interviewer side of the table and haven’t delved much into interviewing advice, you wouldn’t necessarily think of it. Yes, at some point in your career you’d think you’d pick up this kind of preparation for any business meeting, but clearly some people don’t.) I think you’re probably reading more into it than is really there.

I wouldn’t include it as feedback to candidates you reject; if you do, it’s going to make people think it was part of your reason for rejecting them, even though you’d intend it as a separate tip.

5. Using complete sentences on a resume

I recently completely rewrote my CV. I showed it to a few people and they are divided. I write mine in complete sentences, so the word “I” appears a lot (as the CV is indeed about me). For example: “I led a cross-functional project to identify, cluster, and describe visitors to Lucious Llamas Ltd. The resulting campaign delivered a 41% reduction in cost per llama groomed.”

My daughter (who works in HR in Australia — I live in the UK) HATES the use of the pronoun, and thinks it should read “led a cross-functional project…” but I can’t bear that — it feels like an incomplete sentence to me.

I realize that styles will differ between countries. I also realize that it is unlikely someone would read that CV and think “how dare she use ‘I’ throughout — I shall discard this resume at once.” But still, I’m curious to know what your feelings are.

I can’t speak to UK or Australian resume conventions at all — for all I know you print them on purple paper in 16 point font and everyone loves it — but in the U.S., your daughter’s advice is the correct way. And by “correct,” I just mean the standard convention.

{ 895 comments… read them below }

    1. MishenNikara*

      And honestly in 2023 it’s silly to be wasting paper on printing it out if you already applied electronically. In fact, I would see it as a red flag at this point if an employer I applied to didn’t already have a copy, or some other application, on file (assuming one was sent in the first place of course)

      1. Introvert girl*

        I would also consider it a mayor red flag when a potential employer didn’t bother to print out my resume. I’ve put it all the work in sending it, filling out multiple online questions and tests, learning about the company and preparing myself for the interview.

        1. ecnaseener*

          Even if they have it on a screen like LW? Or do you just mean if they don’t bother to have it in front of them in some way?

        2. MCMonkeyBean*

          I’ve had number of interviews where the interviewer just came from several other meetings in a row with no time to go back to their desk between any of them so there are plenty of normal reasons they might not have your resume on hand. It shouldn’t be a dealbreaker if you don’t have one, but it’s at your own risk not to carry extra copies just in case imo.

          1. Princess Trachea-Aurelia Belaroth*

            Interviewers also sometimes pull in other people who were not expecting to be interviewing candidates that day. I’m not coming down on the side of “you MUST have copies,” but I think the adverse reaction of thinking it’s a red flag if the employer doesn’t have enough copies is a bit extreme as well. Some people don’t need to take notes, some people have only ever interviewed with a fairly empty resume or over zoom before, and didn’t need additional copies. On the other hand, it’s not crazy that an interviewer might have lost track of a printed resume, or not anticipated how many copies they would need. I’m all for having high expectations of employers, but that’s more about expecting perfection from a person.

            I also usually bring my master resume (has all my experience back to high school, and is not tailored to the particular job) in addition to more copies of the resume I did submit, just in case something is brought up that I didn’t realize could be relevant from the job ad, and I need to show that experience. Doesn’t happen often, but once I was told that a job I left off was very helpful experience.

          2. ferrina*

            I’ve had this happen. Also seen a young interviewer not realize that they needed to have a copy of the resume on hand (she and I were conducting the interview together- I showed up with a copy of the candidate’s resume marked up with questions I wanted to ask, she showed up with….nothing. Not even paper to take notes with).

            Was it a red flag about the organization? Well, yes. But if you’re a new grad trying to break into the industry, it’s easier to move around once you’re inside than to try to get that first job. If bringing an extra resume lets you get a foot in the door, I’d definitely do it (plus it shows off my troubleshooting skills)

          3. BaskingInMyWindowlessOffice*

            This. We had a four-person interview team and we interviewed in pairs. There were times that I would have to unexpectedly fill in and would not have the resume on hand. I would never have discounted someone for not having a copy but it was always helpful when they did and I could quickly scan it.

          4. MigraineMonth*

            When I did technical interviews, we weren’t even given access to resumes so as not to introduce bias. Where the candidate went to college or worked previously was irrelevant to evaluating their ability to complete a particular exercise.

            1. MigraineMonth*

              We also had to confiscate any notes the candidate took during the course of completing the exercise, since we were trying to keep it confidential.

        3. StressedButOkay*

          The only time it’s been an issue was when it came to an interviewee not having their resume (we had two copies we’d printed – one for each of the reviewers) was when we asked them something about the resume…and they didn’t know. They couldn’t answer it without looking AT the resume, which they hadn’t brought. We quickly gave them one of our two copies and shared, of course.

          But it made us concerned about their fit for the job (on top of several other key issues), if they couldn’t remember something about one of their recent jobs without their resume. We’ve had others come in no resume in hand and haven’t had issues – I think it boils down to how put together the person is in general, regardless of materials.

          1. Lenora Rose*

            That sounds like a different kind of red flag. I might not remember exactly how I wrote something on my resume but I should know the details of jobs I worked recently enough that they show up on my resume, and of my own life…

        4. Observer*

          I would also consider it a mayor red flag when a potential employer didn’t bother to print out my resume.

          That’s every bit as extreme and non-reality based as the idea of dinging a candidate that didn’t bring a couple of copies.

          Others have covered why someone might not have a printed copy, or even any copy.

        5. Megan*

          I agree to an extent, but wouldn’t go so far as to call it a red flag.

          Many ATS systems convert your beautifully formatted, easy to read, one page resume into a multi-page, plaintext document that can make it difficult to follow or quickly find information.

          They sometimes can’t recognize columns and inappropriately collate information.

          They will also put everything with headers they don’t recognize into a misc header.

          This shouldn’t be held against the hiring manager as it’s typically a decision made well over their head.

        6. LawBee*

          This feels similar to red-flagging an applicant because they didn’t send a thank you note, in that they’re both something that feels really important to one side of the equation, and is a total non-issue to the other. Lots of reasons why I wouldn’t necessarily have the resume in front of me; it’s not ideal, but it happens.

      2. Wintermute*

        I always carry a copy because I have been surprised by recruiters adding in things I did not. A disturbing number of recruiters will try to sneak stuff targeted to the job in your “skills” or “software suites” or similar fields.

        Sure they have a copy… it may not be what I think it is.

        I had that happen in an interview once, they asked a question about some software suite, I said I don’t know, they said “but you say on your resume you have five years experience with foosoft?” I said I didn’t and they accused me of lying on my resume

        I pulled out the copy and said “this is what I mailed the recruiter!” and they saw a few things were added (and added poorly I might add, in terms of matching my existing content), and honestly not having foosoft was not a make or break for them, but dishonesty naturally would be. I got to a second interview when they were going to cut the interview short. Unsure if there were consequences for the recruiter/s, it’s sadly fairly common so I imagine not much was done.

        1. MCMonkeyBean*

          Another very good point on why you should try to have copies if you can! I know a lot of people don’t have a printer, but if you can borrow one from a friend (by which I mean print at their place not take their printer to yours) or find a local library I would really recommend it.

          1. Michelle Smith*

            The library is a cheap and convenient printing option, but you can also go to an office supply place like Staples, FedEx Kinkos, Office Depot, etc. to pay for printing. Just bring the resume on a flash drive.

            1. elle *sparkle emoji**

              I understand this is trying to be helpful, and thank you for that, but this advice is a little funny to read as a gen z person who hasn’t used a flash drive since elementary school. I’m not totally sure I even own one anymore. I understand having a physical copy of a resume is good to have but anything that requires a flash drive is not a solution, at least for me.

              1. skittles*

                So email it yourself and print it out at the library.

                Also flash drives are insanely cheap so it wouldn’t be an undue burdon to get one.

                1. SSSSSSSSSSS*

                  Many company IT policies now do not allow the use of flash drives not provided by the company due to the cybersecurity risk. I have a family member whose company was ransomwared, and they suspect it was by a new hire who mysteriously quit a few days after starting.

                2. LIBJess*

                  Email it to yourself/upload it to the cloud/etc. Some public libraries disable the USB drives to prevent potential security risks/viruses that sometimes come along with those flash drives.

              2. LawBee*

                You can order flash drives online for cheap, or get them at your local drugstore. They’re available literally everywhere; I have at least 10 and I never buy them. So a flash drive could absolutely be a solution. (I mean, I will give you one of mine, I am drowning in them.)

                1. Carol the happy elf*

                  I wondered at the adorable earrings a young coworker wore to a party- they were flash drives, blinged up with colored rhinestones. They also had a gold wire ring through the insertion with some tiny “Nerd Droppings” attached. She also had a necklace made of old transistors and blinged flash drives.
                  She mentioned that the pins had solder in them, so they couldn’t be used to steal information.

                  She had taken a jewelry class in college, and won a creativity prize.

            2. Anonymity*

              Last time I applied for a job I did not have a car, and there was no library or Kinkos on the bus route I took for everything else, so it would have added literal hours to my day for me to go print something out. At the time, I did have a printer at home, but now I don’t and so if I still didn’t drive, I definitely would not be bringing a copy of my résumé. I think if you can conveniently print something out, go for it, but also every time I have brought my résumé to a job interview they have told me they don’t need it and I ended up just having to throw it away and wasting paper.

        2. No lizards allowed*

          This happened to me as well. The recruiter didn’t add skills, but the took my resume apart and put it back together in their (terrible) format. The company was quite relieved when I pulled out a copy of my well-formatted resume. It was a technical writer job, so the ability to format documents was a significant part of the job.

        3. Captain dddd-cccc-ddWdd*

          Did the recruiter speak to you after the interview to see how it went? (well, this is what they do in the UK and I assume it is similar) – I would have given them a piece of my mind about that. Setting you up to be accused of dishonesty is pretty egregious imo (5 years of experience with software X when you don’t have any at all is very different from 5 years of experience with software X when you actually had 4 years…)

          It also makes me wonder how many falsehoods are on resumes/CVs because of agencies, and it doesn’t get identified at the time because they didn’t ask about software X or whatever and the candidate doesn’t see what was submitted… but then later the company introduces software X and someone says “Captain dddd has experience in that” and it comes out that I “lied” on my CV… Now that is a difficult position to be in.

          1. LadyVet*

            I just started volunteering for a veteran-focused career service to try to help their clients counter the dreadful resumes they create with their “proven” template. The one they created for me included a ton of stuff I hadn’t done, and was riddled with typos. I knew it was bad because I had civilian experience before enlisting, and have gotten jobs before, but I worry about the veterans who are trusting this service.

            I can’t tell the clients I’m paired with for 60-min. mentorship calls or LinkedIn feedback to change the format of their resumes, but I can tell them to pay close attention to the content and make sure it’s accurate, and advise them to tailor it to each job description.

        4. JustaTech*

          I haven’t had it happen with a recruiter messing with my resume, but with the application system at the university where I was already employed. I went into an interview with another group at the university and they asked about my experience with such-and such, they saw I had done X but not Y, which was weird because doing Y was at the top of my resume.
          Turned out that the application system had defaulted to my very first resume, from when I was brand new to the workforce! Thankfully I had printed copies of my current resume, but it was super embarrassing all around because it was suddenly clear that I was super over-qualified for the job.

      3. Samwise*

        Really? I have a copy of candidates’ resumes and cover letters of course, but I’m not looking at them during the interview. I have my questions jotted down already.

        How would you even know that I do/don’t have a copy of it on file? Like many employers, we have an online application system — if you uploaded it, why would you assume I might not have it? do you mean, it’s a red flag if I haven’t read it? or haven’t read it closely? I’m genuinely mystified here.

        1. Nephron*

          I think they mean red flag if you ask the candidate for a copy. Having a copy to give to the interviewer is one of the stated reasons for having copies of your resume.

        2. GingerP*

          I am super mystified too – Gen-Xer here (Australian if that matters), have worked professionally for 25 years or so, have not brought a copy of my resume in to an interview for years – very outdated advice for the Australian labour market. I always take a copy of a ‘cheat sheet’ I have done up with dot points next to all commonly asked questions, which I review on the way in. That goes in my bag for the interview though.

          I do a lot of interviewing of candidates and to me, it’s an interview about them and if they don’t know themselves and their work history off the top of their head then that’s a bit of an issue! In my field we do expect people to ‘study’ for their interview so if they haven’t practiced then they likely won’t get the job. I wouldn’t fault people for bringing a copy and glancing at it, but I would be put off if they had to rely on their resume for every answer. My interviews are generally 6 behavioural style questions with a panel of 3, followed by questions from the person I am interviewing – maybe 30-45 minutes.

          Very interesting to see so many different opinions! It really seems to vary so much between countries and fields.

      4. Inkognyto*

        I’ve shown up for interviewers where it was supposed to be 2 people. It was 4.

        Fine, I can handle 4, but the new people never saw the resume. The 5 extra copies I keep in my nice fake leather notebook holder, I hand over.

        Then I find out that the recruiter just FUBAR’d my resume when sending it over by messing with spacing, and it not very readable to the others. I also give them copies.

        First note I make is about how the recruiter needs to be warned to not touch my resume when submitting. If they want to edit it, then it’s no longer my resume.

        Where did I get copies (I also do not have a printer, they last a few months run out of ink and it’s cheaper to buy a new one so I never do)?

        There’s plenty of places that will print copies, cheaper than buying ink or a printer. Get a few copies for the interviews.

      5. nobadcats*

        HR recently switched to paperless. I had to spend a LOT of political capital to get them to print my W2 form and mail it to me. I don’t have a printer here at home, and there’s no nearby place I can print something out. I need that piece of paper for my records and my accountant.

        1. Michelle Smith*

          Your accountant does not accept emailed documents? I’m completely with you that your company should provide that option and definitely should not require you to invest $100+ in printer, ink, and paper for one document a year, but it’s so strange to me that your accountant can’t print it out themselves from an email.

          1. LikesToSwear*

            HR likely won’t email it due to the SSN on the document. The accountant may refuse for the same reason.

            1. Observer*

              HR is supplying electronic copies, but the accountant won’t take it emailed.

              For HR to refuse to email because of SSN would be beyond ridiculous and a sure sign that their system is a mess. There are a number of secure ways to provide the document securely, and if they don’t have a way to do that via email, they clearly have not though security through.

        2. Observer*

          Your HR is wrong – they are required to provide paper copies to whoever wants them.

          Also, does your accountant also refuse to file your taxes electronically? At this point, there is no good reason for an accountant to refuse to accept emailed documents.

          I get that this is not the point of your comment, but still…

    2. AnonyAnony*

      Yes to this. Hiring manager here. I really don’t see the need for candidates to bring copies of resume anymore. The way I see it, in this day and age, once a candidate submitted their resume to us, it becomes our responsibility to distribute it to the interviewers. I don’t see it as the candidate’s responsibility to provide it again. And if an interviewer doesn’t have it for whatever reason, it’s so easy for the candidate or another interviewer to forward an electronic copy. I think LW4 should let go of the expectation that they bring something.

      1. Molly*

        I’ve brought my resume to an interview so I could refer to it! My memory is not great, so during interviews, I need to jot down questions that occur to me or things I want to emphasize because I literally won’t remember it 5 minutes later.

          1. nobadcats*

            Same here, my brain is like a sieve these days. I have my resume ready on my tablet, but I also have a notebook and my favorite pen for notetaking.

        1. Elizabeth*

          Agreed, there’s phrasing in there that made perfect sense when I wrote it, but as time goes on, I’m not going to remember the exact wording. I’ve always thought it was my adhd sieve-head memory.

          I also bring a copy of the posting now, because I’ve been caught out a couple of times where I wasn’t sure if it was my memory or if there was a misalignment between the description and the current discussion. And I can jot my questions right on the posting ahead of time, too.

          1. General von Klinkerhoffen*

            You should also print (or at least print screen) the listing because in many cases it disappears from the website after your application or after the deadline. It can also be useful for those “wait, I applied for llama groomer level 2 but we’re mostly talking about alpacas” moments.

        2. Snow Globe*

          Yes, particularly if you tailor resumes to different companies, it’s helpful to have the version that was sent to *this* company.

      2. Captain dddd-cccc-ddWdd*

        I bring a copy if I’ve applied via an agency, just in case my CV has been “adjusted” from my original during the process. I’ve also been on the “interviewer” side enough to know that this happens a lot, including one I talked to recently that wasn’t even light editing but a complete rewrite, the agency hadn’t shared their final version with her so she was confused by some of the things I asked based on the agency version.

        1. The Rafters*

          Our system sometimes sends us old resumes, even though the applicant has updated and submitted the newer resume. It’s not something that anyone deliberately changes, it just happens.

        2. AngryOctopus*

          The site Indeed will actually reformat your resume before passing it on. I had a recruiter (a clearly young clueless one, based on our interaction as a whole) ask in 2019 “what I had been doing since 2012” because Indeed took my job title header and moved my two promotions (covering 2012-2019) somewhere else. Now he could have realized this by looking closely at my resume (the least of his issues, TBH), but your resume can get unintentionally jacked around by some websites.

          1. Once too often*

            Or intentionally, if you’re using Indeed! Their automated systems are screwy indeed.
            Combining separate but simultaneous things into one.
            Integrating separate sections (eg volunteer & paid) & then turning simultaneous things into single items; screwing up chronological order. Etc.
            They do have the option to upload your resume in your own format, thankfully, but their auto formatter has been the default when I’ve used it.

          2. Sunshine Gremlin*

            Indeed tried to convert my resume into their nonsense and it removed every single set of double-letters, turning my resume into nonsense. For example, a coffee company I worked at turned into “co ee” and “staff” turned into “sta.” No thank you, Indeed, I think I’ll keep my original resume that doesn’t seem like a spam bot vomited letters at you.

      3. Falling Diphthong*

        Yeah, bringing copies of your resume to distribute would imply that you’re prepared in case the company has lost your resume. Which definitely happens and people just surge on ahead, including the “someone adjusted my CV” version the caption has experienced.

        But I can see where the unspoken “I’m prepared” could risk landing as “because I figured you people weren’t.” Along with several copies of the resume, there’s usually going to be a whole separate set of phrases, tones, and actions around producing the resumes that help it land on the “I’m prepared, in a way you would like to see in a candidate” side of messaging.

      4. rayray*

        Yeah….I am surprised anyone thinks it’s the candidate’s responsibility to bring a printed copy of their resume. I could understand if maybe the interview came from networking in some way, and you were just meeting up to talk so you hadn’t formally applied for the position.

        If I have submitted my resume to the company already, there is no reason why they can’t review it before the interview and print their own copy or have it on their screen in front of them.

        1. Michelle Smith*

          It’s a holdover from an era of job searching where it was very, very much the standard expectation.

      5. WantonSeedStitch*

        Same. I don’t ever need a copy of a candidate’s resume from them. I have it open on a screen in front of me if I’m interviewing remotely, which is mostly the case these days, or print a copy and bring it with me if I’m interviewing in person. FWIW, we don’t work with a recruiter for the positions where I am a hiring manager, so there’s no danger of a candidate’s resume being altered by a recruiter. If I were a candidate in a recruiter situation, I would probably want to bring a copy of my resume to the interview just in case things might have been changed without my knowledge, and would let the interviewer know that this was what I provided to the recruiter.

    3. L'étrangère*

      I’d of course have an electronic copy handy, so that if I came across an interviewer so backwards as not to have my resume handy I could resend an email copy on the spot

      1. allathian*

        Yes, from my phone. I wouldn’t bring a personal laptop to an interview, because just setting it up and making my phone a wifi hotspot would cause unnecessary delays in an already stressful situation. I’m a pen and paper note taker, though.

    4. Tabihabibi*

      I’ve been told as much by employers and stopped printing resumes starting with my 2016 search or thereabouts. I must have printed 50 resumes before that point that never fully traded hands.

      And now in 2023, I, a mid-30s person working in offices for the last 15 year, have found myself forgetting about bringing something to take notes on a few times now. I blame it on being rusty, but imagine not even having much old muscle memory to rely on. I absolutely would not fault an empty handes interviewee.

      1. Allonge*

        We give interviewees paper and a pen to take notes with and HR prints their resumes for the interviewers. I interviewed at least 50 people in the last five years and I could not tell you if they brought anything of their own.

        For me – my memory usually works good enough that taking notes in the interview would be just a way to gain time, not really needed as such. Most likely I would take a pen just out of sheer habit, but plenty of people don’t need to jot things down.

        1. Allonge*

          This threaded wrong, but in any case I agree with you, Tabihabibi – no fault either way.

    5. spruce*

      On the note taking, I find it super awkward to take notes during a job interview. I don’t take notes as fast as most people speak, and I’d rather be 100% focused on what people are saying than at 90% so the last 10% can go into notes.
      If I can’t remember something well from the interview, I will email my contact back with additional questions afterwards.

      1. rebelwithmouseyhair*

        I must say if I interviewed you, I’d be thinking “hmm yes this is why you should have been taking notes”. And my next thought would be “what if Spruce did that after an important meeting with clients?” I wouldn’t want my clients to think my staff were scatterbrained, I’d expect my staff to make notes and be generally on the ball. Having to reach out to confirm something does not fit my expectation. (This doesn’t apply when the reaching out is clearly to confirm *in writing*.

        1. Allonge*

          For me there is a big enough difference between a job interview and a client meeting that this is too much of an expectation. Going into a client meeting we plan for who will be taking notes, or I can ask to record it so I don’t miss things, neither of which applies to job interviews. It would alos be ok to ask some questions afterwards in writing, or we would send minutes for approval even, which is an in-built place for clarifications.

          But if your field expects otherwise, then of course it makes sense to take this approach.

          1. Wintermute*

            It really depends on context here. In some fields you are expected to be able to focus and retain 100% however you do that is up to you, if you have trouble taking notes and paying full attention it probably isn’t a job for you.

            In most jobs though I think your method would be fine as long as the things you’re asking are detailed stuff or specific numbers, not something big and obvious.

      2. Victoria*

        I’ve been in the world of work for 25 years and have had a number of jobs of increasing seniority. I have literally never taken notes in an interview. Is this an American thing?

        1. Nebula*

          I (British) was wondering this as well. I have occasionally brought a notebook or similar to interviews, but even then I haven’t really found that I’ve used it. I think taking notes in an interview is more common than it used to be, but it’s definitely not expected. On the contrary, I feel like some interviewers would think you weren’t fully engaged if you were taking lots of notes. The expectation that if you aren’t taking notes in an interview, it suggests you wouldn’t take notes in a meeting is the strangest aspect of this for me – I can see the connection, but it’s not one that I would ever make or expect a potential employer to make. I suppose it shows that all of this is arbitrary, in the end.

          1. Nebula*

            Oh, also bringing a CV to an interview – not done at all here, and would come across as strange. It’s expected that the interviewers have copies of all your relevant documents, if you tried to hand them your CV in an interview, it would be embarrassing all round.

            1. amoeba*

              Yeah, for my first few interviews I had one in my bag because I guess I’d read somewhere that you should have a copy with you. Never needed to take it out, so stopped bringing it as well.

              But then we also have full CVs and not resumés in my field and the jobs I applied to were all for similar positions, so not a lot of tailoring possible.

            2. General von Klinkerhoffen*

              I’ve taken them to a UK interview (admittedly not recently) just in case the agency messed with them. I think I’ve only once or twice needed to bring one out to say “yes it might make more sense if you have the full version”.

              I agree it would be very odd to bring one out unprompted.

          2. Teapot Wrangler*

            I would always bring a notebook because then I could refer to my prepared questions. I would probably bring a spare CV too because agencies do sometimes change thing but I wouldn’t offer them round, just offer if it seemed necessary.

            I do think I would find it mildly weird should a candidate turn up with no notepad or anything but I’d find copious notes just as weird.

            1. Owlish_17*

              Right – my best candidate comes prepared – with paper and pen, copy of resume maybe list of references and importantly comes with prepared questions because they’ve thought about and/or done some research on this position/company in advance.

              1. WantonSeedStitch*

                Interesting. I don’t want a candidate’s list of references unless I’ve decided they’re my top candidate after interviews have been completed. Then HR gets the references and sends them to me to all. For us, checking references isn’t something we do unless we’re already pretty sure we want to make an offer to the person–it’s kind of a double check to make sure our impressions line up with what others say about the person.

          3. Empress Penguin*

            Also British, also confused!

            I’ve only had professional jobs in the public sector in the UK. I would probably have a copy of my CV/job application in my own files to read on the train on the way there, but I wouldn’t have multiple copies or give them out to interviewers.

            I’ve also never taken notes in an interview, or seen a candidate do it when I’ve been on a panel.

            1. Princess Trachea-Aurelia Belaroth*

              As an American, every interview I’ve ever had (apart from delivering sandwiches during summer 2020) included the both of us going through my resume during the interview, discussing each entry (or the bigger ones, at least). So I always bring a copy for ME, and it is an easy step further to have 2-3 extra copies in case they are needed.

              Most of my jobs have been in the public sector with strict interviewing processes, which has meant that often someone is pulled into my interview with no notice at all (because there have to be two people interviewing). Also, often the lead interviewer is not an HR person or hiring manager, and possibly not that good at preparing for an interview, so they might not have thought to get enough copies.

              Also, since a resume is not as comprehensive as a CV, I bring a more comprehensive “master” resume with me as well, in case some experience I didn’t include on the submitted resume comes up as relevant.

            2. kicking-k*

              I’m in the UK too and I do take notes, though often not very many (it’s usually the answers to “and do you have anything you’d like to ask us?”). I also have my STAR stories as bullet points on index cards (pretty well all my interviews are likely to be competency-based) and if I’m doing a presentation I’ll bring a printout of the slides in case my laptop won’t connect to their system or something (it has happened). I am very nervous in interviews and have suffered from my mind going blank a few times.

              I don’t typically bring my CV. I don’t think anyone’s ever gone through it.

          4. ABK*

            I just interviewed and took more notes than I was expecting. I had my questions written down, I asked about benefits and jotted down a few words about that, next steps/timeline, main type of work, performance review metrics, etc. They were specific things I wanted to make sure I had right to review later. But yeah, 90% of the conversation didn’t need notes.

          5. LexiBee*

            I agree, it seems strange to make assumptions based on behaviour in an interview room where the candidate is likely most focused on making a human connection and being remembered during a discussion. Regardless of whether industry norms traditionally dictate bringing a resume or require notebooks at client meetings – people have different learning styles and physical needs. You have no idea whether that person has arthritis and would normally be using a laptop in meeting situations rather than grasping a pen. They may be an auditory learner who is going to remember more from the conversation if they aren’t simultaneously trying to scribble stuff down. This smacks of school teachers who felt if you weren’t painstakingly transcribing their lectures word for word you weren’t learning/retaining. And nowadays students can snap a photo of a whiteboard! Norms change over time and we should think about why that might be.

        2. Irish Teacher*

          Yeah, I’m in Ireland and I was even wondering what I’d be taking notes on. Most interviews I’ve been at have been primarily the interviewer asking questions and me answering. Yes, there is the part at the end where I ask two or three questions, but noting down their answers to those would feel a bit strange, either like I am already assuming I’ve got the job (making notes, for example of what classes I would be teaching) or like I am keeping notes to complain afterwards or something.

          Though a lot of the interviews people talk about here are much longer than the ones I’ve experienced (generally 15-30 minutes) so perhaps there is more give and take as opposed to the interviewer asking you a fairly set list of questions (many of which are similar in most interviews) and then your asking a couple of questions (and honestly, often getting fairly set answers).

          1. Irish Teacher*

            I did always bring a folder with copies of qualifications, etc, in case interviewers wanted to see them and I always have a pen on me, but it would be very unusual I’d have anything to make notes on.

          2. ecnaseener*

            I don’t see how writing down their answers to your questions should come across like you’re assuming you already got the job! You asked, presumably, because you want to consider the information before accepting an offer if you get one – that’s understood. If asking isn’t presumptuous, writing down the answer isn’t either.

          3. Snow Globe*

            I wouldn’t assume they think they’ve got the job; I’d assume that the candidate is talking to a number of different companies, and is writing things down to keep straight who said what.

          4. Cruciatus*

            I just had an interview on Thursday. It was over Teams, but I took notes on things like what the day to day is like in the position, the qualities that are important in the position and things like that, and I wrote down the answers to things I asked. I do not in any way think I have the position, but I’d like to be able to recall them later, especially if I do get another interview. There’s a lot of info coming out of interviews that can go in one ear and out the other and I try to just highlight the important things and I try not to take too long. It’s usually just written words like “confidentiality, trust, work on behalf of other directors in program, no travel needed (despite the job posting saying this).”

            1. Cruciatus*

              And so if I do get another interview, since trust and confidentiality came up, for example, I can bring up experiences I’ve had that showcase these things. If I didn’t write them down I might sort of remember that came up but wouldn’t realize how many times it was said, etc. Now I really hope my interviewer didn’t think it was annoying I took notes….

        3. Just Another Techie*

          I’m American, have been working for twenty years, and have been on both sides of the interview table. I have never taken notes in an interview, and never cared if a candidate did or didn’t. The question and ensuing conversation are so bizarre to me.

          1. Falling Diphthong*

            I’m American and am having trouble picturing what you would write down. Like “At Amalgamated Teapots, I would focus on LIDS.”

            In meetings where I take notes for myself, those notes are relevant to tasks I need to do. In an interview, I haven’t agreed to take on these tasks.

            I imagine it varies by field, but I can see where being engaged in the conversation would be more important than note taking–in most business meetings, not only is one person usually tagged to take notes if a complete record is wanted, but that person is usually not conducting or the major speaker.

            1. Wintermute*

              this is where I’m at!

              I know what I’ve done and experienced so I’m not going to be taking notes on that. Unless it’s one of those interviews where you get introduced to a dozen people and want to keep them straight, I don’t know what I WOULD be writing down?

              Usually important details like salary and vacation days are talked about later and would be in an offer letter anyway (and subject to negotiation), I don’t know what I would need to remember, and I could comfortably remember what few things I may have to theoretically remember off the top of my head.

              1. Cmdrshprd*

                “I know what I’ve done and experienced so I’m not going to be taking notes on that.”

                Maybe I have been lucky and had good interviewers, but in my experience while the interviewer asks a lot of questions about myself, they also give me information about the company/role at the beginning, during the questions they ask, or info from the questions I ask.

                At ABC Teapots, we only manufacture teapots, clients come to us with the designs already in hand, do you have any experience dealing 3rd party design vendors?
                We have only been manufacturing teapots, but are looking to expand into the design of teapots, do you have any design experience?

                The x role will require a lot of contract management and organization tell me about your experience with contract management?

                Or asking about the make up of the team/department. Who the role will report to but who the roles works closely with.

                I don’t take full on notes just shorthand, “dealing w/design vendors” or “expanding into teapot design,” “contract management/organization”

                Team is 3 support, and 6 SME.

                While I take notes it is not the same level/amount as if it were for a meeting or something else the really needs detailed notes. I don’t think it has ever interrupted the flow of the conversation.

                Or I will also write down questions that popped up while the interviewer was saying something but I didn’t want to interrupt them, or the flow/questions moved in quickly, so writing down the questions for the end lets me remember to look at them.

                Occasionally at the end of the interview when they ask “do you have any/more questions?” I will say let me check my notes, and I will scan them for 3/5 seconds and follow up.

              2. Turquoisecow*

                And often the interviewer will give you a business card, or you’ve communicated with them beforehand (or you could get the info from your HR contact) so there’s no need to write down their name or anything.

            2. AngryOctopus*

              Yeah, I mostly take quick notes to remind myself that “company A does X process like company B, but has Y machine to make it easier”. But those are things that might inform my choice if I were to get an offer from both places. I take notes when actual job offers are verbally given over the phone (to make sure it matches written) and in case I need to compare to another offer.

            3. NeedRain47*

              I’ve made notes during the “Do you have any questions for us” part of the interview, but it’s just jotting down a few words about their answers if it’s something I want to remember. I usually bring a copy of my resume and the job description for my own purposes and make notes on the back or in the margins.

            4. MCMonkeyBean*

              If you’re interviewing with multiple companies and one of them would have you focusing on lids and the other would have you focusing on spouts, that seems like information worth noting down so you can remember later when you are weighing your options and deciding which job you are more interested in.

              If you can just remember then good on you, but I definitely need to note things like that down!

            5. WantonSeedStitch*

              As an example, last time I interviewed, I took notes on things like:

              * Top priorities for this position for the first year are X, Y, and Z
              * Team A’s biggest challenge working with this team is shared definitions of common terms – need to co-create a glossary?
              * They really want someone to break down the silo between this team and the Teapot team

              I wrote down stuff that I felt like I wanted to remember to focus on if I got the job. Of course, since the position I was interviewing for was an internal promotion, it also was stuff that was just good to know even if I DIDN’T get the job: I’d know what my new boss’s priorities would be, and would be able to support them in accomplishing those things.

              1. Cruciatus*

                I wrote elsewhere that I recently just took notes in an interview. I’m a little surprised at how strange people are finding this. My notes are similar to yours, I just highlight what I consider to be the important key words. I’m not writing out a novel. But when I look back over it I can see what ideas came up and, if I get another interview, I can focus on those ideas next time and just refresh myself on what the interviewer thought was important for the position vs. what the job posting said.

                1. Turquoisecow*

                  I wonder if some of this is just how people are. I’ve never been much of a note taker in general. Even in school, I mostly remembered things and found that if I did take notes I often didn’t look back at them, except for certain classes. I found it much more helpful to review for example the textbook or other reading material and I never for in the habit.

                  In work meetings lately I have gotten into the habit of jotting down my specific tasks at the end and confirming with my boss or whoever that’s what I’m doing “ok, to confirm, I’m going to send the llama reports to Bob, and you need X, Y, and Z information on that, and you want the data going back 2 years only in New York.” As others have pointed out, I don’t have any tasks after an interview. But also even in meetings like the one I mentioned, someone will often send a follow up meeting: “TC to send llama reports to Bob with (parameters), Bob to review by next Thursday, meet to review the following Wednesday,” so if my part is less complex I just remember it and then am reminded by that email.

                  As I get older the specific details like that are harder to remember but something about what a potential new job’s focus is and how that would affect me is definitely something I’d remember.

            6. InterplanetJanet*

              I formerly worked in academia where your interview was a full day of rotating groups, so my notes were usually for my own benefit later in the same day, because it’s an absolute non-stop onslaught of questions and conversations and it’s easy to forget who said what!

              In academia (probably other fields, but this is the one I know) it’s really useful to ask the same question to different groups, so it’s nice to refer to four hours later to be able to say “the so-and-so panel offered their thoughts on XYZ, and I’m interested to hear your perspective as well.”

            7. Queen Ruby*

              Agree with this. I bring a couple copies of my resume with me, but only once did they come in handy. I was interviewing at a company I had interviewed with a year or two earlier, and they had given the interviewers an old version of my resume.
              I also always have a notepad with me in a monogrammed leather portfolio that I like using just because it looks fancy lol. But I almost never take notes in interviews. I try to make it like a conversation, which makes it easier for me to engage and remember details. I tend to ask the same questions at every interview, so it’s not like I need to have them written down as a reminder. The leather portfolio 99.9% of the time stays in my bag.

            8. a clockwork lemon*

              The only real notes I’ve taken as an interviewee are in the early stages of talking to HR–benefits, comp structure, etc. Otherwise I won’t necessarily remember the broad strokes if/when I get to an offer stage, and sometimes the deciding factor between two comparable offers is, like, who they use for health insurance.

          2. Nancy*

            Agree, I don’t recall ever taking notes for interviews in my 25+ career. I have been interviewing candidates for the past 2 weeks and none have taken notes. I didn’t think anything of it.

            It’s been a long time since I handed anyone a paper resume, or received one.

          3. Christina*

            Agreed. I’m frankly astounded to find out that I’m expected to carry a notepad around with me even if the likelihood I’ll need to take notes is slim to none. I have a good memory and most interviews don’t exactly impart earth-shattering wisdom that needs to be written down anyway. Seems like a lot of theater — bring a notepad to show you’re prepared, even if you don’t need it. Bizarre that people care about this.

        4. Allonge*

          Looking at this discussion, and based on other US specificities I know about, it could be somewhat more of an American thing (although not universal as many USians don’t do it) because of:
          – thank you notes – if I am expected to remember names and immediately use them, I absolutely need a notepad
          – possibly because information on benefits is shared in interviews but not in writing, and more of benefits are negotiable
          – possibly because the start date could follow much sooner after an interview than over here in Europe. My notice period is several months, nobody will expect me to remember the team setup on my first day based on the interview. With a notice of 2 weeks, it’s probably different.

        5. PoolLounger*

          I’m American and have never taken notes during an interview. I have no idea what I’d even take notes about.

        6. Bunny Lake Is Found*

          Yup, I have interviewed hundreds of people and never had one person take notes and I definitely have never taken notes when being interviewed in person. The only time I have ever taken notes has been on at the phone screener/offer stage where the gritty details (salary, time off, 401K vesting) was covered. I honestly have no idea what one would really even take notes regarding in a typical interview…maybe it’s an industry specific thing?

      3. Colette*

        Yeah, I’m the same – I can jot down notes in an important meeting, but for an interview, I’m unlikely to have action items I need to remember; it’s a conversation, and I don’t need notes for that. (I might write down notes after the interview, but I usually don’t even do that.)

      4. Empress Matilda*

        It’s a different kind of note taking, though – you don’t need to take notes as if you’re prepping for an exam or recording a meeting. I mean, you don’t really need to take notes at all, but if you do they would be very point form. The salary range, if it’s not listed in the ad. The fact that they’re hiring two people for this position, or that the manager is going on maternity leave in a few months. Or the position is about llama grooming but you and the manager have a shared interest in alpaca husbandry and you want to refer to that in your thank you note.

        It’s not about writing down every detail of the conversation – just super quick notes about anything that you might want to refer back to later. And for me, it’s less about taking notes at all, than it is about having something to do with my hands so I don’t fidget.

      5. cee*

        @ Spruce This! I am an auditory learner and needing to take notes only distracts from actually listening. I usually bring a notebook for the interviewers benefit but only write down garbage half sentences, dates and numbers since I know I’ll remember without writing down if I’m actually able to devote full attention to listening.

        I have never needed to email with questions because I remember what I need to know after listening closely. I wish there wasn’t this perceived link between notes and engagement because i would love to stop the note taking performance.

        1. kicking-k*

          I’m a very visual learner and typically take loads of notes in meetings as if I don’t, the info just doesn’t go in. I try to keep it to “need to know” as otherwise I’ll be manically scribbling the whole time (I did this in lectures at college) and it can be off-putting. Or people ask you to write the minutes (which I hate doing!)

          I take fewer in interviews, but I still take some. It’s just too risky to have nothing to write on.

      6. Michelle Smith*

        I don’t take notes as fast as people speak either. I write more in an informational interview than a job interview, but I haven’t been to an interview for a job I actually wanted where I wrote nothing down.

        I always bring my questions to the interview written out on paper (not printed from a computer). I write in a way that it’s not as easy for them to read what’s on my paper from across the table unless they are really trying. If I think of another question or something I want to follow up on later while someone is speaking, I’ll jot it down. Maybe I have something on my list of questions, but it gets covered spontaneously during the conversation. I might jot down two or three words to help me remember the answer and also mark the paper so that I know I don’t need to ask it. I may also want to jot down something I might need to remember for the next interview, like what the timeline is for hearing back (so I know when to follow up), a fact about the job to follow up on in a subsequent interview, or an assignment I need to do, like sending them my references by X date. So for example if someone says something that suggests the hiring manager is a micromanager, I’ll jot down something that will remind me to ask questions to probe deeper on that when I interview with the HM. Or if I am not able to bring out something I think is relevant in a way that makes sense or if we make a good personal connection over something, I might make a note to include that in my email follow up (I usually email thank you notes, although I get that’s controversial.) For example, I once had a panel interview where one of the interviewers off-handedly mentioned something about her plants in response to a question about my hobbies (which includes gardening). I made a note of who that was and then in my follow up email to her, I included a sentence or two about our shared interest and a recommendation for solving her problem. I didn’t get that job, but she did refer me to another position in the organization that I actually did get. People remember how you treat them and my memory just isn’t good enough to remember little details like that unless I jot them down.

        When I’m job searching it’s not usual that I interview for one position at a time. It always seems to come in waves rather than trickles. I might have multiple job interviews that week or even that day for different positions. I’d never be able to keep it straight without notes.

    6. OP4*

      I’m actually not so sure about that! Most, if not all, of the folks that I’m referring to are in undergrad or grad programs in-person at the time of the interview and would, ostensibly, have more access to printers the most people. And I’m a printerless millennial myself, too although I’m not sure if early 30s still counts as younger folks.

      I tend to agree that the résumé is not particularly always necessary and that I as an interviewer should have it on hand myself. But (I thought) career advice to bring your resume to an interview was ubiquitous enough that it would go hand-in-hand with the career advice to bring something to write with so I paired both phenomena in the letter, and caveated the resume part. Thinking about it now though, I only ever see both present (resume and supplies to jot something down if need be) or neither.

      I’m seeing that it’s not ubiquitous!

      1. Darren*

        Yeah I can’t see any reason I’d need to bring a resume (you’ve already got it, and I already know what it says so why would I need a printed copy) and I also see no reason for supplies to jot down notes.

        While I am using the interview to assess whether or not I’m interested in the role and the company it’s not so complicated a process on my end that I need to take notes. As an interviewer it is because I’m assessing multiple candidates and need to have detailed notes to contrast them, and an interviewee I’m much more looking for red flags and things that will make me pull my candidacy entirely.

        Assuming I haven’t found any of those the contrasting as a interviewee is very much the details of the written offers/contracts.

      2. Falling Diphthong*

        What would the notes they took be about?

        That is a sincere, non-sarcastic question.

        I want to point out yesterday’s question from the frustrated resume-giver, where the resume-asker was apparently typing out the answers verbatim and very slowly. To the aggravation of someone who had agreed to make time to have a conversation with them.

        1. ThatgirlK*

          Most interviewers that I have come across genuinely talk too fast for me to take notes. I have been in the professional world since 2008 and I have never once taken notes during an interview. I have had plenty of job offers. (I am in the US).

        2. Hlao-roo*

          From my experience as an interviewee:

          At a career fair, I had a few (two or three) short interviews with companies that day and I took a few notes so I could keep the major points straight. What their major products are, where they were located, what the job was focused on. For example:
          Product – widgets
          Porridgeville, IL
          manufacturing engineering

          During other (non-career fair) interviews, I found it helpful to write down my questions beforehand and jot quick answers to them as they came up during the interview or at the end when I asked my remaining questions. For example, if I wanted to ask what the typical hours of the position are, I would have “hours?” written down on my paper pre-interview, and during the interview I would jot down “9-5.”

          In contrast to yesterday’s reference checker, I try to spend as little time writing as possible during an interview. I write down a few words/phrases so I can refer back to the main things I care about when I am (potentially) weighing a job offer later.

          1. Turquoisecow*

            If I was at a fair talking to multiple people, I’d assume they’d have handouts or brochures about the company so I’d probably take those. Maybe make my own notes in the margin. But if it’s one interview I can keep the info straight until I get done and can think more detailed thoughts about the job and whether the salary is enough and etc.

        3. Emmy Noether*

          I think taking down verbatim or very detailed notes would indeed be distracting and slow the interview down too much. The point is to be *prepared* to jot down just 2-3 things over an hour interview, in bullet points, and only if necessary.

          For example, things that should have been in the job description but weren’t, or any kind of numbers (I can’t remember numbers in my head for the life of me), like salary and vacation days, that kind of thing (yes, those things will be in the offer letter, but if an interviewer tells you this info, you want to remember exactly what they said).

          1. Wintermute*

            that’s about all I could think of, if something major changes or the details. But to me salary and vacation days are the single most important thing about the job and I’m not going to forget those! if it’s just 2-3 things I think most people can comfortably remember those without a note.

            1. Emmy Noether*

              I’m kind of weirdly bad at remembering numbers specifically (as compensation, I have a better-than-average ability to remember poems – companies should just make their leave policies rhyme). I’d probably remember important stuff like salary, but having to concentrate to fix it in memory will distract me for the rest of the interview. And I definitely won’t remember three weeks and four other interviews later.

              As there is a very simple solution to this problem, invented a few thousand years ago, I just write stuff down!

              1. wendelenn*

                Don’t ever get sick, your 5 days will go quick.
                If you go on vacation to explore our great nation, we’ll allow you two weeks for the travel you seek.

              2. kicking-k*

                I’m the same! Numbers just don’t stick but I know song lyrics and poems without trying. Fortunately, salary scales are usually in the job posting in my field (at least in the UK) but if they tell me that the job consists of roughly 70% this and 30% that, I won’t remember if I didn’t jot it down.

        4. ecnaseener*

          I’m wondering if you just have a great memory, FD. The notes are about anything you want to make sure you remember about the job, for those of us who need them – so if I ask a question about the nature of the workload, or the hours, or the culture, etc etc etc, I’m taking notes on the answers. Because I’ll want that information days or weeks later and I won’t just remember it.

          1. Allonge*

            A good memory definitely helps with not having to take notes! But the thing is, the original question here is not ‘is taking notes in an interview professional’ but ‘is showing up without a notepad and pen unprepared and therefore a bad sign about the interviewee’.

            And if you need notes to remember anything of what was said, then indeed showing up without a way to take notes is unprepared, but plenty of people have a memory that works better because that’s what they got in the genetics lottery.

            I don’t need to take notes to remember the one or two things I want to follow up on, or the generic info that will be shared in the interview. Therefore I am not unprepared if I were to come without paper and pen. But apparently people would still judge me as unprepared.

            1. ecnaseener*

              I don’t disagree with any of that! In an ideal world, we would all be able to do what works for us without having to perform the right kind of attention. Just responding to FD’s specific question.

          2. Slow Gin Lizz*

            This is a good point. I have a very good memory (for some things) and have never taken notes during interviews because the kinds of things I would care about that I’d learn in interviews are the kinds of things I’d definitely remember (or can google; I can find your company’s website pretty easily so I don’t need to write down the URL, for instance). Plus I have gotten very good at casually asking about things I know have been discussed that I cannot remember (“Can you remind me again what the deal is with the llama portraits project?”). I probably wouldn’t email after the interview to ask stuff like that that I’ve forgotten, but I think it would be fine to ask again if a job offer were extended. After all, it’s never a bad idea to clarify things like PTO, sick leave, or hours upon receiving an offer even if you discussed them already.

            So I don’t take notes in interviews and I think unless it were obvious that the candidate forgot vital information that was given to them in the interview, such as whether travel were required for the position or what types of projects the job focused on, I don’t think it’s worth focusing on whether or not the candidate is taking notes. Everyone’s brains work differently, so if some people can remember things in their heads and other people need to write things down, you as an interviewer should take that into account. (I am thinking specifically here of my dad who has dyslexia and doesn’t write *anything* down, except phone numbers. He draws diagrams sometimes, sure, but he remembers everything else in his head.)

        5. I am Emily's failing memory*

          When I’ve interviewed in the past I’ve jotted down things like the department structure (how many people? how many peers? how many layers of hierarchy?), the expected working hours (flexible with core hours or firmly set?), the specific name of the software package they use (one I already know? one I’ve never used but know has an active user community online?), something about whether I’ve learned I’d be working with a bunch of other new hires or a lot of people who have been there forever or a mix (and what the would-be direct manager’s tenure is), whether the employer conducts annual performance reviews, whether the team is growing/shrinking/stable in size, what kind of training or professional development employees receive or are eligible for… basically just stuff to flesh out the picture of what my day to day would be like if I worked there, that I won’t trust my lousy memory to keep straight if I’m interviewing at more than one place.

          Occasionally I’ll also jot down stuff that’s not strictly related to the hiring process but I still want to remember, like the name of a book the interviewer spoke highly of, or the name of a professional conference I hadn’t heard of before but might want to look into, etc.

        6. Snow Globe*

          I jot down brief notes about topics of discussion or answers to my questions, because if I’m interviewing with multiple companies I want to keep answers straight. Was it Company A that said they were getting ready for a big product launch, or Company B? Which employer said that there has been some turnover in the department? I’m not writing a ton, just enough to jog my memory “product launch in June, may be travel”.

          1. obleighvious*

            Like others, I’ll take notes of answers so I can refer to them later (“vacation is earned and goes up XX days after Y years” so I can go home and compare it to my current position if I need to, and haven’t forgotten the details in the meantime).

            BUT, typically, during an interview, I’m jotting down notes while the interviewers are talking that I want to ask a follow-up to that WASN’T one of my standard prepared questions. Like, say they mention a product offhand that wasn’t in the list of required qualifications, and I just wanted to follow up and say “Oh, you mentioned Product A – how much of the job is using that, I’ve mostly used Product B?” or even “Oh, you mentioned Product A – I have extensive experience”? So the notes are not long-winded verbatim notes, just a “oh, I want to follow-up on that”. And since often the conversations CAN be wide-ranging, I don’t want to forget that I DID have a follow-up when it’s time.

        7. Panda (she/they)*

          If you’re familiar with Alison’s advice that interviews are a two-way street, I often take notes on my “side” of the interview as the candidate. So I might ask about the ratio of tasks listed in the job description and write down “more X than Y”, or note down answers to other questions: how frequently people expect me to be in the office vs remote, the amount of travel required, who I would interact with most frequently (I am typically applying to director-level roles), what types of projects I would be working on, any skills they note are particularly important for the role so I can make sure to highlight them…and any questions I think of during the interview that I want to remember to ask later.

          I am not constantly note taking throughout the interview, but most of my interviews have been an hour and it’s been helpful to jot down a few things.

        8. ThatGirl*

          When I take notes during interviews, it’s to do one of the following:
          a) help me remember what we talked about later, including any big red or green flags about the job or the company
          b) write down a few words in case I have a followup question
          c) refer to my own previously written notes about myself and get important questions answered

          For me, even jotting a few words down can help me remember things later, and when I’m in full-blown job search mode and possibly doing multiple interviews a week, it’s really helpful for me to refer back.

        9. Inkognyto*

          I write down something (anything) nearly every time I ask a question that was detailed.

          I asked a question I’m making it seem like I put something down.

          Job said pays attention to details. I usually have a lot of back and forth in my interviews, and I’d love my interviews to be 15 minutes. Most of min are 30-45 minutes of discussion. That’s a lot of information to ‘remember’

          There’s often not a 2nd interview unless it’s with someone else.

          Sometimes I note something odd they state during the interview like they are using some odd version of software. I’ll jot it down.

          I take those notes and put them into my computer about the employer. When you have 4-5 interviews in a week you do not want them or any information to blur.

          Again sometimes it’s just words. They aren’t going to ask what I’m noting on. It’s maybe a sentence or some phrases. I once wrote “nope, take them out of the running”

          if you need a moment to formulate some thoughts, taking a few seconds to write something down. It can be refreshing then boom back to it.

        10. Cynthia*

          I have always taken quick notes to remember details about the job I want to remember, next steps/timeline, names of other people in the department and their roles. Basically, I note down things I learn about the job from the interview that are not part of the description. But, I tend to treat interviews as 2-way and in my head, I am interviewing the job for fit as much as they are interviewing me.

        11. bighairnoheart*

          Interviews are stressful enough to me that I’m unlikely to remember all the info I got from the interviewer, so usually I jot down a few things I think sound important as we go. Like if there’s anyone else interviewing me that I didn’t know about going in–I’ll write down their name so I don’t forget, also if they mention something like salary or an important component of the job that wasn’t in the description. I doubt I’ve written down more than a few quick notes at any interview I’ve been in, but I usually find something worth writing down. I wouldn’t be surprised if others don’t see a benefit to doing this, but I feel like enough people do benefit from taking some notes that either kind of person isn’t surprising to me!

        12. flora_poste*

          I have a notepad and pen in interviews with me, and have used them for jotting down questions or clarifications to ask later (eg ‘You mentioned that an organisational priority/strategy for the coming year is x, how would this department be expected to contribute to its realisation?’ otherwise I would immediately forget. Often this is just a one-word scribbled on the paper, but it’s something to jog my memory when it comes to asking!

        13. grapefruit*

          For me, bringing a pen and paper to an interview isn’t as much about taking notes during the interview–though I might jot down something brief from time to time–as it is about having notes I made in preparation for the interview available.

          For example, I might jot down key points I want to make sure to mention at some point about why I think I’d be right for the role, relevant experience I want to highlight in some way, questions I want to ask, details about the organization from my pre-interview research, etc. I’m not writing these things down during the interview–I’ve written them down for myself ahead of time. That way, if I get stuck on a question or can’t find the words for a point I want to make, I have something to glance at as a reminder.

        14. Donner*

          I write down my interviewers’ names and what they do. I take a couple notes on the conversation so I can remember which conversation I had with which person. Sometimes they tell me something I want to look up later, so I make a note of that. If I ask a question, I write down the question and answer.

          I typically meet with 4-5 people for 0.5-1.0 hour each, and they don’t all give me business cards.

          If my note taking aggravates them with how much time it takes, well, I guess I’m actually ok with that. I had to take time out my schedule for this, too, you know.

      3. Lexie*

        I’m on the younger side of Gen X. Some of the advice I was given way back when was to not take anything into an interview. Not even your purse. I’ve also been told to take a copy of your resume and obviously anything you were specifically asked to bring. As for note taking that wasn’t something that was ever suggested, like others have said there wasn’t really anything to take notes on except maybe the salary and benefits and those are easy enough to keep in your head.

        1. Lilo*

          This is funny because I’m a middle millennial and I was trained to bring pen and paper to interviews. But it could be my field (law).

          1. Eldritch Office Worker*

            I’m also a middle millenial and was taught to bring something to write on/with and copies of your resume – with the rationale being you would look disorganized otherwise.

            Definitely not in law! I’ve done this for everything from retail interviews in my teens to pandemic video interviews where I had a copy of my resume pulled up and ready to send over if they needed it. It makes me itchy to think about going into any meeting without note supplies lol

          2. Bunny Lake Is Found*

            Elder millennial in law…no one ever told me to bring a pen and paper and I’ve never seen anyone else do it.

            1. A.K. Climpson*

              That’s so weird to me, as a younger millennial in law. I absolutely brought my resume and pen and paper.

              I was thinking law might be different than lots of the other fields discussed, because lots of firms send you around to meet with many people sequentially, and I always found notes helpful (and have seen candidates take them now I’m interviewing). Plus, I feel like pen and paper notes are just more common across the board for lawyers — I say, looking at three different legal pads on my desk.

        2. I am Emily's failing memory*

          … what are you supposed to do with your purse?? This seems kind of sexist, women need to carry things and pockets on women’s clothes often range from non-existent to “can hold no more than a tube of chapstick and a couple quarters.”

          I mean, obviously there are some purses that would read less professionally than others so I’m not saying any and every purse is equally acceptable, but the idea that you’re supposed to somehow get yourself across town to an interview without a place to hold your keys, wallet, phone, transit pass, medications, etc is absolutely bonkers to me.

        3. Rex Libris*

          Maybe it’s a generational thing. I’m also Gen X, fairly senior management at this point, and I’ve never taken notes in an interview, heard it suggested, or even considered it. I do usually bring an extra copy or two of my resume, but I’ve only needed them once, so they normally stay in a folder.

      4. metadata minion*

        I really hope I’d make a checklist and remember to bring something to take notes on, but one reason why it might not be automatic for me is that in normal work meetings, I can take notes on my computer. It’s generally not appropriate to do that in an interview — at least for the candidate — so I can absolutely see myself sitting down at the table and going “oh, crud; forgot a notebook!”.

      5. Cat Lady in the Mountains*

        I always bring a few copies of my resume, but I rarely bring something to take notes on. I’m a heavily auditory processor – if I try to write things down, I absorb so much less than if I just actively listen in the conversation. So note-taking is a distraction to me. And even when I do take notes, my note-taking style looks so messy and insane from the outside that I generally want to avoid it in an interview — I prefer to listen in the conversation and then jot down my takeaways after I leave the interview. If someone isn’t taking notes and is routinely losing the thread of your conversation or asking you to repeat yourself, that would be a red flag to me, but the act of taking notes isn’t in and of itself a virtue. And I’m guessing you don’t want candidates performatively note-taking if it doesn’t actually help them anyway – on the hiring manager side of the table, I’d rather do everything in my power to get candidates to be their real work-selves in the interview, since that gives me more useful information of what they’ll be like on the job. If the job requires memory for detail, and a candidate exhibits that skill at a high level, I couldn’t care less if they accomplish that skill by writing things down or by verbally processing. (A job where taking notes is a key part of the role would maybe be the exception, but I’d look for a more direct way to test for that skill.)

      6. Warrior Princess Xena*

        The career advice I was received was to bring your resume/CV to job fairs or recruiting events, but no one really mentioned to bring one’s resume to a job. I might bring a printed out job description if anything.

      7. ClaireW*

        I’ve been working for 10 years and have done a lot of job interviews (I work in tech and have had some short-term contracts or startups failing). I have never once, in all that time, taken or felt the need to take notes during an interview. Maybe I have a better memory than I think, I don’t know – I take notes when interviewing, but not when being interviewed (outside of like, tech challenges where I’m taking notes as they describes the problem but then I have a laptop anyway). Maybe it’s a UK thing But I’ve never taken notes and most people I interview don’t take notes when I interview them either.

    7. Jenga*

      Gen Xer here.

      I don’t see why it’s necessary for a candidate to bring a copy of their resume to the interview. Any time I’ve been the interviewer, I already have this and have looked it over. If I needed to reference it, I brought it in myself. Interviewers need to prep for the interview as well.

      As an interviewee, I generally don’t pull out my notepad until it’s time for me to ask questions, which I have written down beforehand…with the exception of anything that came up during the conversation.

      1. rayray*

        Yeah, same. If the resume has been submitted through the application process, then the candidate has already provided their resume to the employer. Why would they need to bring another copy? If an interviewer hadn’t looked at my resume and/or didn’t have it with them when conducting the interview because they expected me to bring them a copy, I’d assume they and their company were disorganized and unprepared. Interviews are two-way streets. The interviewer should be prepared as well. I’ll do my part and research the role and company, and they should do their part and read the information I have provided.

      2. I am Emily's failing memory*

        I used to bring a few spare copies of my resume to interviews – the pre-interview trip to Kinko’s before the first interview was something of a ritual. I stopped sometime around 2013 or 2014; this was around the time when applying through systems had become decidedly more common than instructions to email your resume to a specific email address, and also around the time that it had become more common for me to see people bringing their laptops to any sort of meeting (internal meeting, external conference, conducting a job interview). I hadn’t needed to give out any of my printed copies in years at that point, so I felt safe stopping the practice at that point. To date it hasn’t presented any issues.

        I always bring a notebook, because I know I’ll want to remember details that I won’t if I try to rely on meat-based memory instead of paper-based memory. It’s for me. I’ve interviewed a TON of candidates for different level roles over many years, and I don’t know if I could say with any confidence whether any specific person I’ve EVER interviewed had one – I’m certain that many did, and if I try to conjure up the image of different interviews in my mind, I picture the candidates all with notebooks… but are those accurate memories down to that level of detail, or just my mind filling my own tendencies into all the parts I don’t have a clear specific memory of? I never would have thought it was important enough to commit to memory, just like I don’t remember what shoes anybody was wearing, but if I try to “look at their shoes” in a conjured memory, they’re all in nondescript businessy/dressy shoes because that’s what I’d expected was most likely there.

    8. I should really pick a name*

      A lot of people don’t own printers these days irrespective of age.

    9. Dread Pirate Roberts*

      Yes I’m not younger and I don’t own a printer, but also I wouldn’t want to kill trees for something that in my career has been wholly unnecessary. I’ve had many jobs and have a decent interview-to-offer ratio, am in as senior a position as I want to be in, and have never taken notes or felt the need to, or brought a hard copy of my resume with me (except maybe in the 90s?)

      As a hiring manager, I’ve been given paper resumes a few times but they go into the shredder right after the interview because I’ve already used the electronic copy to screen who I want to interview and if I feel the need to consult it again I still have the electronic copy.

      If a company expressed their dismay that I didn’t take notes or bring paper resumes I would consider that a sign of an old-fashioned and rigid workplace that I’d factor into my decision about accepting an offer – so if I was screened out for not conforming to their norms then that would be the right outcome.

    10. Flowers*

      Yup I don’t own one either. BUT I always bring a copy of my resume – thats’ just what I was always told to do and I don’t see any reason to change that. And I’m glad I did because I realized that formatting can be totally off. When I interviewed for my current job, I saw the copy they had and it was formatted so oddly (I had applied through Indeed). I did give them my copy of the resume as well, and things did turn out alright (I got hired after all!)

    11. Lacey*

      And if you’ve printed out your resume for interviews before and the interviewers all reject it saying, “I have a copy here” and waving their ipad at you, well… you stop wasting your money at Staples.

    12. Milfred*

      Printed resumes are not perfunctory. Nobody reads them anymore (other than machines). They likely got all the info they needed from your LinkedIn profile. Paper resumes are in the same league as the old Japanese custom of exchanging business cards when two people meet.

      But, having said that, the Japanese do still exchange business cards, and it is considered a faux pas not to have a business card to exchange.

      And it’s a faux pas to go into an interview for a job that requires a college degree and do not have one or more copies of your resume to share.

      I hate to tell people to spend money, but if you’ve spent tens of thousands of dollars on a college degree, a grand for a laptop, and get tripped up because you don’t want to spend $40 on a cheap printer and some paper, it seems you are being “penny wise and pound foolish”.

      1. metadata minion*

        It’s probably better to just go to a Kinkos or something like that if you don’t normally need to print. Cheap printers are almost guaranteed to have mysterious Issues the minute you actually need to print anything, and that sort of thing often isn’t quickly fixable even if you go to print things out well ahead of time. Heck, even decent-quality printers often have mysterious issues if you don’t use them often.

        1. DataSci*

          Cheap laser printers aren’t as cheap as cheap inkjets (because they aren’t sold as a loss leader for ink cartridges), but they also seem to have fewer problems. We got one early in the pandemic to print out stuff for the kid’s Zoom school, and it’s still useful on occasion.

      2. TootsNYC*

        Back when I hired, I printed out resumes and put them in a file if i thought I might hire them later (I hired a lot of freelancers and also got a lot of people asking me for names of possible candidates). That way I could write a note more easily. And I could scan them visually. And I didn’t trust that I’d be able to find all the various electronic ones.

        It was faster to pull out a file and flip through the paper, with the font and some of the key words slipping past my eyes and jogging my memory.,

    13. AlsoADHD*

      I own a printer and I think it feels very dated to print resumes. Granted I’m remote and don’t plan on going to an in person interview well… ever again hopefully. But I wouldn’t take notes or have a physical resume for an online interview and in person I’d assume we’d have technology too. I might bring my laptop Most naturally! I’m not young but I do in a technical function so maybe that colors it. Paper resumes sound so…old.

    14. TootsNYC*

      It’s funny, but I sort of share the “you seem unprepared” feeling; maybe it’s just the old paradigm.

      I wonder why they didnt bring a notebook to write down any info they gleaned, but I guess they could remember that if they wanted to, and stuff like hours, salary, etc., is usually in the job description, so there’s not a practical reason for that either.

    15. Observer*

      With résumé copies, a lot of younger folks just don’t own printers.

      That’s actually the only reason I don’t have sympathy for in most cases. Because most young people DO know about the myriad places that offer printing one or two copies of something as a service.

      1. louvella*

        Eh I mean, I can go to FedEx or the library to print something but it’s easily an hour out of my day…and usually I have a slightly different version of my resume for each job so it’s not like I can just print a stack one time.

      2. Susannah*

        I tend to bring a resume mainly because I feel more comfortable having a sort of prop, something to hold. But I’d be awfully put off if my interviewer hadn’t looked at my resume and had it there to refer to if necessary. And I’ve never taken notes during an interview. The best interviews I’ve had have been conversations. If I had a notebook out on my lap, I’d feel like a police interrogator.
        I’m late BB, early Get X, and I find this absolutism on having a notebook with you just bizarre. I mean – you’d actually consider it unprofessional to NOT be taking notes? I think I’d find it really impersonal and off-putting if someone was scribbling while I was talking.

        1. Observer*

          I think you meant to reply to someone else. I tend to agree with you on the (lack of) necessity to bring a resume. Sure, it’s a good idea but I can’t think that not doing so is actually a red flag.

          All I’m saying is that while there are a lot of good reasons why someone might not bring one, “I couldn’t figure it out” is not one of them. And in the majority of cases, that’s what “I don’t have a printer” means.

    16. I Wrote This in the Bathroom*

      This. I do own one, but it’s expensive for a younger person (and takes space that they might not have) and I only need to use it a few times a year. The rest of the year, it just sits there collecting dust while the ink cartridges dry out.

      Another point I was going to make, I always used to bring a copy of my resume and note paper and then never get to use it. It is awkward to break eye contact and start writing when the interviewer is talking. It is more awkward still to start writing when I’m the one talking? And what will I write anyway, the information that we exchange is all pretty basic and at the end of the day I don’t need a hard copy of it (especially if the odds are low of me getting and accepting an offer anyway). So, I used to do it, but I cannot for the life of me explain why. Definitely would not give this advice as feedback to the applicants. It might be obsolete.

    17. louvella*

      In high school I was told to always bring a printed copy of my resume to an interview and I followed that advice until I was 22 and then realized no one had ever asked for it so I stopped. (I’ve never owned a printer.)

    18. Nina*

      Seconding, I’m not even all that young but for me getting something printed would be a trip to the library in the next town outside of my usual every-other-week trip, and a pretty major undertaking.

    19. Sam*

      I find it strange the LW admits they haven’t touched paper in years, but expects the interviewee to bring paper?

      I think an interviewee may be reluctant to take notes on a phone or computer out of concern they’d be distracted by incoming notifications or SEEN as distracted by the interview when they were genuinely taking notes — and if they too don’t regularly use paper, I don’t see the need for them to make a show of it just to have something in hand for an interview.

      1. bluepants*

        right! the idea that a candidate would refer to a tablet, phone, or laptop to take notes on during an interview is the thing i’m snagging on. i would feel awkward and disrespectful, not to mention be distracted, if i took out my laptop, opened it, and began typing (my laptop keyboard isn’t silent!) while someone is speaking to me from a company at which i want to work. (P.S. i’m a middle/elder millennial and in copywriting/libraries.)

  1. Panhandlerann*

    Rhetorical situations vary. Complete sentences are not always appropriate or expected. (In addition, the overuse of “I,” particularly at the start of successive sentences, is a problem. The reader knows the items on the resume concern the person whose resume it is! There is no need to announce that over and over and over.) I say this as someone who taught writing and grammar for many years.

    1. GammaGirl1908*

      This. A CV or resume is not a standard prose document. Different writing conventions apply. There are plenty of times full sentences are not required in writing, such as when you have bullet points (which you often do on resumes), in presentations, on slides, on posters and signs, et cetera.

      The reason we do this makes perfect sense to me — it’s because repeating I at the start of the sentence takes too much focus off what you actually want the reader to know, which is that and what you led, designed, taught, grew, or won. You also waste a lot of space on unnecessary words that you could be using on words describing your accomplishments.

      If LW is struggling with this, she can think of her name at the top of the page as the start of each point. So:

      Pookie Sparklepants

      *led a cross-functional project to identify, cluster, and describe visitors to Lucious Llamas Ltd. The resulting campaign delivered a 41% reduction in cost per llama groomed

      *won the 2017 Llama Groomers International New Groomer award

      *opened Atlantis’ first llama grooming academy in 2018

      1. An American(ish) Werewolf in London (ish) aka Pookie Sparklepants*

        I am changing my name at once (I am LW5)! As I said in the letter (I only sent it yesterday!) I realise it’s totally a low stakes question. I will consider changing it. But it will eat my soul.

        Regarding the ‘wasting words’ point, I’m not convinced that the word ‘I’ takes up all that much space but I see your point. Happily, I’m not currently job searching (there were redundancies where I work but I escaped…this time) so I have plenty of time to consider it. I do appreciate your thoughts though. Cheers!

        1. Varthema*

          Well, think about it this way – in prose and poetry both, writers use repetition to draw attention to the repeated word – it’s a rhetorical device with a very specific object. The same applies here, and “I” is not really the word you want attention drawn to because it’s redundant information. Good writing is more than just grammar, it’s all about crafting words in a way that has the writer’s intended effect on the reader, and in the case of a CV, the effect you want is for reader to able to see and notice certain important information while scanning it (since scanning is all they’re going to do, at first anyway) without distracting them (i.e. by breaking the conventions of the genre). :)

          1. DyneinWalking*

            And the problem with redundancy: It generally causes people to tune out, especially when they are reading with the intent to gather information… and let’s be honest, no-one is going to read a resume in a relaxed fashion. It’s essentially a list of information and as such it’s going to be skimmed a lot. It’s in your own best interest to make it easily skim-able and to remove anything that a reader would skip over, such as unnecessary redundancy. You don’t want to entice people to skip over parts of your resume, because with every skipping there’s the danger that they might skip so far they miss relevant information.

            1. justcuriousiguess*

              The other problem with redundancy is it generally causes people to tune out…

        2. Agent Diane*

          Hi Pookie Sparklepants!

          UK-based person here, and I can assure you leaving the I out on a CV will not cause major palpitations. Reframing it mentally as a list, like GammaGirl suggested is a great way to calm your inner grammar nerd.

          The one exception might be if you are applying to the UK Civil Service but that doesn’t use CVs anyway. For that you write examples of competencies and the sifting panel is looking for evidence of what the individual did. A lack of sentences starting with “I” would always make me annotate an application with “who did this?”! If it was a strong application I might bring them to interview but I’d be listening out for them using “we” instead of “I”.

        3. Magenta*

          I’m in the UK and do a lot of hiring, the majority of CVs I see omit the “I”. I wouldn’t penalise people either way, but would assume that a person who repeatedly used “I” a lot wasn’t very experienced in applying for jobs. There is nothing wrong with being inexperienced at applying for jobs as it can mean you have been in the same one for a long time, but personally, it is something I would avoid on my own CV.

        4. Clara*

          Honestly, from reviewing CVs in the UK, your daughter is correct on what the standard convention is. I’ve not seen any that have used ‘I’. Not the end of the world, but would seem a bit off, which isn’t your ideal first impression to make.

        5. An American(ish) Werewolf in London(ish)*

          I am not new to the job market – Gen Xer here (waves at other 60s and 70s babies) – I guess it’s just my personal quirk. I suppose I’ll have to bite the bullet and edit myself (or at least the Is) out.

          Thanks for all your input!

        6. Rainbow*

          I’m British in the UK and a massive stickler for grammar. The CV is a list of bullet points, not prose. From the point of view of those reading it, they want condensed information quickly, hence the bullets. It’s not about how much physical space “I” takes up, it’s about the physiological time taken for the reader. Hence, each point should be on separate lines, and conveyed with generally as few words as possible for the information given.

        7. Bagpuss*

          I’m in the UK and would agree with Alison and your daughter.
          Maybe think of it as a list of bullet points rather than needing to be full sentences, if that would make it easier for you if you have to write one or when reading those of applicants, if you are involved with hiring).

          you’re right that ‘I’ of itself doesn’t take up a lot of space, but in my experiences, CVs where the applicant uses full sentences everywhere tend to be much wordier generally, and as well as being harder to pick out the salient information they often seem to involve more padding and less material information. (I’m not saying that yours does, just that’s been my observation more generally!)

        8. NeedRain47*

          Think of it as the “I” being implied. They already know who you mean, it’s your CV, you don’t need to tell them many times. (this viewpoint based on my bachelor’s degree in linguistics.)

        9. fhqwhgads*

          Think of it this way: it’s redundant. It’s also a list, not a paragraph. It is a given that the resume list your work accomplishments/qualifications.

          When I go to the grocery store, my shopping list says:
          Bananas
          Bread
          Eggs
          Cheese

          blah blah blah, because it is a given that the purpose of the document is “I need to purchase these things”.

          It doesn’t say:
          I need to purchase bananas.
          I need to purchase bread.
          I need to purchase eggs.
          etc.

          Does it feel weird reading the latter example list? If so, that’s what the people reading a full-sentences resume feel like.

          1. Double A*

            This exactly! The entire point of the bullets is that each one is the continuation of an implied sentence. This is why the bullets need to have parallel structure and should NOT each be a complete sentence.

            You sentence starter is, “In this job, I:”

            This is why each bullet must start with a verb and writing something like “Responsible for a team of 30” would not work.

            It if helps, translate it from prose. You would write in a paragraph using a colon and semi-colons. “In this job I: Led a team of 20; raised revenue x%; and sold a million widgets.” Now just turn that into bullets.

        10. br_612*

          Complete sentences aren’t required in bullet points. It is perfectly grammatically correct to have a bullet point without a subject. In fact personally I roll my eyes when every single bullet point IS a complete sentence unless it has to be for clarity reasons, which is certainly not the case here.

          And I think one thing that gels with Alison’s usual advice is that you don’t want to stand out for something like having complete sentence bullet points that all start with I (as several commenters below have confirmed is outside the norm in the UK). You want to stand out for accomplishments.

          1. TootsNYC*

            even in prose, sentence fragments are acceptable.
            So make this a whole bunch of sentence fragments

        11. TootsNYC*

          it’s not about wasting space.
          It’s about wasting attention.

          What people care about is “what did you do”? That’s why starting with a verb, and choosing the right verb, is the convention.

        12. Observer*

          But it will eat my soul.

          That’s a fairly out-sized and rigid reaction. I imagine you avoid poetry, because that’s FULL of incomplete sentences. But poetry is not the only form that is full of these “fragments”. And when you are just doing stuff for yourself, you should do what makes you comfortable. But if you are this rigid when dealing with others or outward facing communications, that could be a problem.

          Are you this rigid about other things or is this just an idiosyncrasy?

          1. An American(ish) Werewolf in London (ish) aka Pookie Sparklepants*

            In all fairness, Observer, I was using hyperbole intentionally. I’m not even sure I have a soul to be eaten (or who would do the eating).

            It is, though, I suspect, an idiosyncrasy, apparently restricted only to me (judging from the comments). Perhaps a personal preference is more accurate.

            And I still (usually) put two spaces after full stops. Sue me. :D

        13. marvin*

          The key to surviving the job search process is to accept that your soul will be eaten sooner rather than later.

      2. KatChatz*

        Australian here, from now on I will only provide my resume on purple paper with 16 point font :)

        1. An American(ish) Werewolf in London(ish) aka Pookie Sparklepants*

          I already have. It’s scented paper too. Using Comic Sans. ;)

    2. *kalypso*

      I am South Australian, and the advice for ‘resume’ in Year 10 work experience was ‘avoid ‘I’ at all costs; it makes you look self centred’. It’s job, three dot points about PD or significant achievements, earlier or irrelevant jobs get 1 dot point or skipped so resume stays 1 page (unless field convention is otherwise, like listing postdocs or papers in an addendum in academia or specialist/accredited medical practitioners).

      You also need the extra characters for fitting that dot point in 2/3 page width.

      The only reasonable exception is if there’s no cover letter and you have to demonstrate language proficiency in excess of ‘knows conventions, has managed to graduate school’ (and here high school stays on the resume so they know that – if you don’t put it on there the first interview question is ‘what school did you go to’ even if you’re getting interviews at 70! It’s an Adelaide thing, because huge parts of our social networks are founded on who you went to school with and the perception of each school – I went to a country school and ‘You’re from the Limestone Coast? My sister’s cousin’s dogwalker drove through there and got a pie!’ is social grease equivalent to ‘how ’bout dem Packers?’ or ‘so, Patriots again, yeah?’)

  2. Beezus*

    1. “Does this impact you or why are you asking? Obviously I have had it approved, why would you think otherwise?”
    2. we all need to know less about each other.
    4. I genuinely hate bringing a notepad and resume to interviews but good to know! (I usually bring a print out of my resume and dumb folio for show. why should I need to take notes???)

    1. Sophie K*

      My dumb folio is definitely for show, but the notepad has my notes for the interview – things I want to ask about the job/employer, short notes to remind me of my prepped answers, and if I need a moment to think, it feels less awkward to “check my notes” than just sit there in silence until I come up with an answer. I will also make a note if they tell me anything I want to remember or look into afterwards, but those are relatively few compared to my prep notes.

      1. Beezus*

        that’s absolutely fair. my dumb folio I got as an alumni gift so I also am just like what’s your deal. I work in HR and wouldn’t judge either way bc by the time I do in-person interviews I HOPE I know who I’m talking with so if they need to take notes fine! if not, also fine.

      2. NB*

        Yes, this is what notes are for me- to remind myself of questions I have and salient information about my experience I want to highlight. No one has ever been upset about me taking notes or referring to them during an interview. And note taking isn’t like taking minutes or transcribing, it’s just occasional jottings to help the memory.

        I usually take a couple copies of my resume cause who knows if they’ve even looked at it (has happened a couple times) and helps me remember what’s on it. There are a lot of people who seemed shocked that others take/use notes for interviews, but maybe those people have great memories. I certainly don’t, and writing things down is what helps me keep track.

    2. KateM*

      I thought for a moment here that 1. was meant as answer to LW2.
      Which would maybe be an idea – “were you hoping to sleep with Bill and now your hopes got crushed – or why else are you so interested in his sexual life?”.

        1. KateM*

          And after that each time they still bring it up: “wow, you are still so salty about Bill not going to sleep with you?”.

          1. N C Kiddle*

            I can see how this makes a fun fantasy of putting them in their place, but in practice it’s just keeping the focus on the inappropriate conversations. An eyeroll and “why are we talking about this at work” is probably a better tactic for an on the spot shutdown.

      1. JSPA*

        ramping up the sexual talk (and thus escalating what is, in effect, sexual harassment as well as religious harassment) is NOT NOT NOT a good answer.

        1. Not Totally Subclinical*

          Agreed. This would be a great response in a group of friends but a lousy one for the workplace.

      2. Ellis Bell*

        I honestly think the more obvious piss-take is the fact that they are bragging about *not* being virgins. If I was inclined to take them down a peg, I would say “This isn’t high school, no one gives a shit if you have more notches than Bill.” or “Well done for having sex and everything, but no one needs to be picturing how pleased you are about that” or “please don’t compare yourself to Bill just so we know that you have sex”. That said, this is the kind of cruel, sad, childish, bullying shit that you really can’t allow anyone to normalise or claim that it’s joking territory. I’d probably take a more serious tone, instead of trying to join in. Something like: “No it isn’t funny, it’s pathetic”. Sometimes you just have to let your disgust show and let the joke clang on to the floor. It’s a stronger statement and you don’t have to say as much in the moment (usually people feel pretty sick in the moment around this stuff). Yeah, you’ll come up with cleverer answers later on in bed, but it’s actually stronger to just say “That’s disgusting and I’m speechless” or a simple “That’s really not okay”.

      3. Boof*

        It’s a good answer for a sitcom and I agree, and WTF what’s wrong with not having sex; that being said I feel like actually saying something like “I’m not sure if this is sexual or religious harassment, or both, but it’s not cool, and there’s nothing funny about it” etc. I don’t know if it’s worth pointing out that it’s effecting bill too or if that’d just make it worse though, depends on the flavor of jerk I suppose.

      4. Festively Dressed Earl*

        I want to be Bill’s anger translator for a week. Yes, it’s a bad idea, but a person can dream.

    3. JSPA*

      The reason to take notes is not nearly because memories are imperfect (though that too) but also for what we could call stagecraft and signaling.

      Referring to notes is a way to gather your thoughts without an awkward pause.

      Referring to notes is a way to cross check on a statement that seemed like it might have been a mistake but you don’t want to call it out as such, and end up in a debate over who remembers what.

      Referring to notes is a way to make it clear that you are comfortable using exterior memory aids to make sure that a task is complete (there is no bonus for keeping everything inside your head– in fact, in most career paths, making and keeping sharable records is a fairly essential part of becoming a functional adult who works well with others.)

      Consider that the whole concept of having an interview is itself largely about stagecraft and signaling that collegiality and adequate communication is possible… Otherwise we could hire entirely based on c v’s and a couple of email exchanges.

      1. L'étrangère*

        Let me point out though that nobody shows up to an interview without a phone, and that my encrypted notes app is all I ever need in a meeting

        1. Dr. Rebecca*

          Let me point out that a candidate fiddling with their phone for the whole interview is *extremely* offputting, regardless of the reason they’re doing it.

          1. too many dogs*

            I had a candidate TAKE phone calls during the interview, telling me, “I’m kinda in demand right now, so I’ve gotta take this.” Of course we didn’t hire him.

      2. Allonge*

        To be honest this may make sense to you and in your firld but it is still puzzling to me. How long are the interview questions we are talking about that one would need to refer to notes? What info is shared in an interview that HAS to be remembered exactly and xou cannot get it otherwise?

        I get asked a question, I note it down, fine. Absolutely nobody reasonable will believe that I am smarter if I say hmmm looking at my note of a sentence than if I look sideways for a moment. (nor the other way around).

        I have nothing against taking notes if if makes the person more comfortable or confident, or if they feel it helps! But I really don’t think that projecting being comfortable using an external aid is better than being comfortable using your memory if it’s reliable enough.

        1. NeedRain47*

          oh holy cow I’ve been asked interview questions that had three verses and a four part harmony. I mean they were multiple part questions. I always have to ask them to repeat it. Having to try to hold three related bits of question in my head while crafting an answer is stressful. Unfortunately I never realize I should be writing it down ’til it’s too late.

          1. Allonge*

            Just for the record, those are likely very bad interview questions. I know this does not help you, but commiseration!

        2. Some Lady*

          It’s more and more common for interviews to use behavioral interview questions – ones that ask about past experience (ex., “Tell us about a time when you [used relevant work skill, etc.]”), with the idea that past behavior is the best predictor of future behavior. Having notes about some of your key experiences to refer to can be helpful to some people, especially if you’re worried you might get nervous/freeze a little on the spot and not be able to think of something.

          That said, the reason I appreciate it when people have something to take notes is that it can show an interest in the job details if you are making notes about what you, as the candidate, are learning about the position/the organization–that you’re noting down answers to questions you had, or details that would help you make decisions around taking the job or preparing for it if you’re hired. Of course plenty of people can do that mentally without notes–we all have different learning styles–but I do think it’s a small but positive indicator when someone is prepared to be able to track information they’re receiving in this situaion.

          1. Weaponized Pumpkin*

            I definitely have my “behavioral” situations jotted down, because I do tend to freeze, as well as a list of questions I might ask. I’ll even mark up the job description with how my key points link to their key points. Somewhere in the world, someone can do all this without notes but it sure isn’t me!

          2. ClaireW*

            Honestly I would be horrified if someone decided that because I have confidence in my own memory I’m not ‘interested’ in a job that I’ve put time and effort into interviewing for. That seems ridiculous to me, like interviewing is not fun, I am not going to be there if I’m not interested and I don’t need to distract myself with writing down what you said (and potentially missing the next thing you say because I’m busy writing).

      3. Lexie*

        In my career I did have to create records that are considered to be legal documents. However, that was done after the fact from memory. Due to the type of work I did I was expected to be in the moment and engaging with the the client, not stopping to jot down notes. So showing that you didn’t need to write things down the minute they happened and could wait at least till you got to your car to make any notes could be a bonus.

      4. louvella*

        I mean my memories are imperfect but if I take notes by hand those are going to be imperfect too (because my handwriting is illegible to even me and I am not really capable of writing and listening at the same time).

        I only really ever take notes digitally but feels weird to pull out a laptop or phone in an interview.

      5. marvin*

        Not to be a pessimist, but if any harrassment or discrimination comes up during the interview (or any work meeting) it can be very helpful to have notes on hand to back up your memory of the events.

    4. pope suburban*

      We all do, indeed, need to know less about each other. Management/HR needs to be made aware of this, and in the meantime, it’s perfectly okay to say, “Can we not,” or, “I’m not interested in that,” or, “Let’s keep this work-related” whenever these people are loudly advertising that they weren’t raised right (wrt shaming someone at work, of course; their sex lives are their own business, as is their religion or lack thereof). I truly cannot imagine what would possess someone to act like this and I hope these people are all plagued with the feeling that they are about to sneeze, but can never quite manage to do so.

    5. Observer*

      2. we all need to know less about each other.

      True.

      But the problem here is not that they *know*. It’s what they are doing with the knowledge, and that’s gross.

    6. TomatoSoup*

      I take notes for two reasons:
      1) It keeps my mind from wandering
      2) I’m horribly bad at names, so I write down the names of the people I’m talking with or anyone they might mention. Otherwise, I have to make sure to avoid using anyone’s name in the conversation because it disappears from my brain if I don’t immediately write it down.

  3. ChrisH*

    I didn’t realize I was expected to take notes. I take notes like crazy when I actually start a job, but it doesn’t seem necessary at an interview. I’m willing to bet there are interviews who would declare note-taking to be unprofessional.

    1. Ask a Manager* Post author

      There might not end up being anything you need to write down, but there easily could be — like if they ask you to send a particular item as follow-up or to contact X at Y email address to ask Z by X date or so forth, or you realize you need to ask a follow-up question about something they mention and you don’t want to forget. You want to be prepared for that kind of thing (and to come across as prepared for it), just as you’d (hopefully) come prepared to jot down anything important at any other meeting you attended.

      1. Aussieb*

        Australian here – I think this may be an outdated/American thing. I was taught this is school but haven’t done this in 10+ years and haven’t seen it done by anyone I’ve interviewed (of all ages). People can note things on their phone if they need to buy copious note taking/note referral would look unengaged to me.

        1. ecnaseener*

          I hope you wouldn’t hold it against anyone at least? Some of us need to write things down if we want to remember them – interview processes can take weeks or months after all. I would hate to think interviewers are seeing me as un-engaged because I’m taking notes, of all things.

          1. Some words*

            Not to mention stress impacts memory. I’m not relaxed during an interview though I try to project that I am.

            I take a lot of notes while learning a task, too. On paper. And I’m a high performer. I tend to use my phone as a phone, not a computer.

            It sounds like there’s perhaps some bias going on.

        2. WantonSeedStitch*

          Someone taking out their phone in an interview to make a note there would look APPALLINGLY unengaged to me, OTOH.

          1. VI Guy*

            I often take notes on my phone and people have reacted well. I explain at the start that I’m using it to take notes. It helps that I’m clearly typing for a few seconds then put the phone to the side and ignore it. If you thought that my use of a phone made me APPALLINGLY unengaged then you probably aren’t the right company for me. Maybe it depends on where you are or the workplace?

            I have taken copious notes on my phone in some meetings and no one gave me a dirty look. Again, it helps to be clearly typing (I keep it on silent but my thumbs are moving quickly) and engaged with the discussion.

        3. Le Sigh*

          “People can note things on their phone if they need to buy copious note taking/note referral would look unengaged to me.”

          I think there might be a bit of a disconnect here. I use a small notebook for interviews, but I don’t take copious notes. I jot down the questions I want to remember to ask so I can refer back to them, as well as any notes or reminders (send so and so the April writing samples, make sure to find out what so and so meant by X). I’m very likely to forget if I don’t write it down. And I can’t do that on my phone. I use my phone constantly but it’s much harder for me to type efficiently — in an interview situation, I will focus more on dealing with crappy tiny phone keyboards than I will the interview. If I have paper, I can jot it down and stay focused.

          All of which is to say I think it’s important to focus less on the specifics (the instruments we use) and more on the behaviors (ie, is this person paying attention, are they more focused on notetaking and not listening, etc.).

        4. Starbuck*

          “People can note things on their phone if they need to buy copious note taking/note referral would look unengaged to me.”

          So interesting to see this difference! In the U.S. I think a lot of people would see the exact opposite – any use of a phone would be disengaged and impolite, while notes on paper are at worst neutral, at best you come across more conscientious and organized.

          1. Le Sigh*

            And I think this is where it would be good for people to catch themselves on this kind of assumptions. I agree a lot of people in the U.S. would see it that way, but phones at this point have evolved into multi-functional tools — to take notes, to set reminders, and even serve as disability aids. Using phone =/= disengaged.

            VI Guy mentioned above that they simply state at the beginning they’re using their phone to take notes — I think just being upfront about it is the way to go so there’s no cause for confusion.

      2. I'm Just Here For The Cats!*

        Couldn’t someone take out their phone and write a note there if something did come up? I am someone who takes a pen and notebook with me to interviews. But being that more people are used to electronic note taking I could see someone taking out their phone to write down instructions. Alison would this be seen as odd or wrong behavior for a candidate?

        1. Weaponized Pumpkin*

          I don’t think it would work to use the device to store your prep notes, but it works for those few reminder/to-do things you might get in the interview. Naming what your doing seems really important to signal why you’re using the phone the first time — “let me just jot that note down”

      3. louvella*

        Hmm, if that comes up I’ve always just pulled out my phone and said, just making a note of that real quick!

        1. louvella*

          (For the record I often have a notebook and pen in my bag, so would be able to write it down there at a job interview, but probably wouldn’t for that kind of thing because I will definitely forget to look at it later if I do that.)

      4. AlsoADHD*

        Wouldn’t you put that information into your phone? I’m not that young but if there were something actionable, I would find it more efficient to put it into my Trello or calendar s apps probably or some other actual system. I guess some folks use paper planners still, but I don’t see why that would be the standard in 2023. Are folks writing stuff down in a notebook to transfer later?

    2. GammaGirl1908*

      Someone pulled me aside at a very early job and gave me the advice that you should always try to arrive at a meeting with something to write on, that you risk looking unprepared or unengaged if you don’t, and that you would far rather have it and not need it than need it and not have it.

      An interview is a meeting, too. While “unprepared and unengaged” is a pretty big accusation to hang on a pad of paper, the end part certainly is true.

      1. Beezus*

        why do I need to arrive to a meeting to take notes this isn’t a university lecture. I’ll jot something down if needed on a sticky note but this kind of attitude is yucky.

        1. Miri*

          How else do you keep track of specific actions, details or tasks that are your responsibility, or assigned to you in the meeting? (genuine question!)

            1. Miri*

              Oh, interesting – I think many workplaces don’t have such a detailed action log that’s maintained for every meeting.

              1. Eldritch Office Worker*

                Can confirm. You’re responsible for your own action items in many workplaces.

            2. JSPA*

              I’m sure it works great if that’s how everyone expects it to be done! But it seems very workplace- and matter- specific. In a way that if you are in that sort of space…you know it to be the case. Just as most workplaces don’t require a radiation badge, but if yours does, you know that it does, but you also know that that’s unusual.

            3. The Other Dawn*

              The only way that would happen at my company is if there’s a formal project going on. And even then, there are always times when we need to takes noted because other things come up in a project meeting discussion, but aren’t meant to go into a tracker. It would be notes for me to check on something that could affect my department, or any number of things.

              There wouldn’t be any kind of tracker for a standard meeting. We’re expected to be prepared to take notes if needed.

            4. Sparky*

              How do they get into the project tracker though? My company takes notes during meetings with clients so that we can then add action items to our project tracker afterwards.

            5. TootsNYC*

              OK, so the project tracker says “Beezus to provide X,” and there’s convo that Joanna in HR is going to be your source, and someone mentions that a certain website or regulation might have info as well; does the project tracker give all that info?

              That’s putting a lot on your project manager!

          1. L-squared*

            How much stuff are you guys being assigned at interviews? Seriously, I don’t recall ever getting a task from a job interview.

            1. Princess Trachea-Aurelia Belaroth*

              Usually for me it’s the answers to my questions. If I’m interviewing more than one place and asking specifics about the job so that I can compare them later, it’s very likely I might conflate, confuse, or forget the answers from different jobs entirely. A lot of people have an over-inflated sense of the accuracy of their memories, because your mind fills in blanks. After lacking crucial info several times from forgetting it, I now take notes.

            2. Weaponized Pumpkin*

              I rarely leave an interview without writing something down. A lot of my interviews (senior level, consulting) tend to have an info-sharing vibe, not just answering questions to fit me in a slot. Sometimes the note is just for me, other times it’s a reminder to do something. Things I’ve written down:
              – Reminder to send them a work product related to something we discussed
              – A resource (book, article, person) they mentioned they thought I’d like
              – Something they mention that I don’t know and want to look up
              – Important answers to questions like PTO, salary, or reporting structure
              – Follow up questions I’ll want to ask later but don’t want to now
              – Instructions on how to email them / send a thank you note

          2. Anonymous 75*

            I have taken notes in 1×1 meetings but not in larger meetings or interviews. those have always had follow ups via email that were sent out shortly after.

        2. Ask a Manager* Post author

          Whoa, really? I agree with Anna — this is something every office I’ve worked in has always taught interns and junior staff very early. Walking into a meeting (especially with someone more senior than you) without anything to take notes on is a really common “I’m being cavalier about this meeting/my job/any needed follow-through” cultural signifier.

          1. Beezus*

            yeah bring a pen and a pad to a meeting but that is not an interview? what should I be taking notes on that I won’t get emailed

            1. Ridiculous Penguin*

              I don’t know what job you work in, but it’s not standard for everyone to be emailed the notes for a meeting that include minute details that include everything that applies to everyone individually. This is why people generally take notes.

              If you find yourself too good for that, I guess I hope that works for you in your career. For most people in most contexts, it does not and will negatively impact the way people perceive them.

              (Even if things are entered into a project tracker, there is almost always context surrounding a task; I am a project manager, and I’m way too busy to remind people why they are supposed to be doing things.)

              1. Miri*

                I don’t know what job you work in, but it’s not standard for everyone to be emailed the notes for a meeting that include minute details that include everything that applies to everyone individually.

                Yes, just seconding this!

              2. Emmy Noether*

                Thirded. Culture in my compnay is no official minutes, ever (except for one meeting that I lead where I instituted minutes because important decisions are taken and no one can remember the reasoning two years later when they come up again. Since I both lead and take the minutes, they are quite succinct, though). Everyone is expected to take note of their own action points and relevant facts.

              3. Susannah*

                Wow, that’s a little hostile.
                I would not expect to be emailed notes after a meeting or interview. I’m a journalist, so I’d have a notebook on me somewhere as always. But I can’t imagine anything I’d need to write down during an interview. Info about salary, bennies, etc. are indeed the kind of thing sent to you in an email or hard copy.

                And if I were interviewing someone who kept taking notes – unless it was something very specific that needed to be written down, like a date or address, I’d be really put off by it. I get that newspapers are not like other offices, but the experiences people are sharing here are 180 degrees different from what I’ve experienced.

            2. Michaela*

              I agree with Beezus at least for interviews – I never bring something to take notes in an interview. Don’t need notes to refer to, and if it’s just one or two follow ups needed, I can write it on my phone after.

              Late 30s and corporate middle management. People still hire me. Can’t believe that some will hold it against me.

              1. Fikly*

                I’m not disagreeing that making hiring decisions based on appearances is ridiculous, but I’m shocked that you are in your late 30s and in corporate middle management and cannot believe that people are judging you based on appearances and then taking action based on that.

              2. NL*

                It is fascinating to me that people who read Ask a Manager, who I’m guessing are more likely to read job searching advice than others, haven’t come across this. It’s such a common thing to be taught. Or at least I thought it was and now this comment section is showing me that it’s not.

                It must be industry dependent?

                1. perstreperous*

                  I don’t know if it’s industry-dependent, country-dependent or both – but I work in IT in the UK, have probably interviewed a couple of hundred people over the years and:

                  1. The number of people who took notes during the interview was about five.

                  2. The number of people who had pre-prepared questions was two or three.

                  On the second, the number who had searching pre-prepared questions was one. I remember them mainly because my co-interviewer was shocked at their effrontery … the cheek of it, making demands of the interviewer! They got the job, though.

                  I sometimes think that the whole process of obtaining a job, and keeping it, is simply taken more seriously in the US.

                2. Falling Diphthong*

                  It’s definitely context dependent.

                  I would view having a way to take notes as normal for a meeting after you have the job, but usually not needed at an interview. I can see it landing as performative: “See, I have my pen and paper and I just jotted down the thing you said.” Or like yesterday’s reference letter, it lands as “So can you spell teapots? T. E. A…” when you should be engaged with what’s being said, not with making sure your permanent written record of the conversation is as detailed as possible.

                  This is distinct from having your own questions in written form, which is fine, but some people do that by memory.

                3. lalala*

                  @perstreperous “I sometimes think that the whole process of obtaining a job, and keeping it, is simply taken more seriously in the US.”

                  As an American, I wouldn’t be surprised if that’s correct. (And the reason is pretty simple: health insurance!)

            3. Manglement Survivor*

              The interview committee is not going to email you notes from your interview.
              You should be asking questions at the interview and you may want to jot down some of their answers, ideas that come up that you might want to research later, peoples names and job titles, reminders when something is mentioned that you want to get back to you later in the interview, etc.
              You sound like you just plan to remember every single detail that happens during your interview perfectly. And if you’ve got that kind of memory, good for you. Many interviewers will just think you look unprepared.

              1. Master Procrastinator*

                This has to be a cultural difference. I worked in the UK not-for-profit sector for twenty years, with experience of being hired and hiring. I have never in either role been aware of an expectation of note taking in interview. I absolutely have had interviewers email me any follow up tasks and would do likewise for candidates if relevant (though I can’t think of any instances of this off the top of my head). I’ve had a few people take notes at interview, for example to remind themselves of my answers to their questions. Personally, I might write down all my thoughts immediately after an interview, but have never needed to do it there and then, unless it’s a video call where I can do it subtly. I’ve also never heard of people bringing their CV/resume to an interview! It’s the company’s responsibility to have it to refer to, surely.

                1. Abigail R.*

                  Yeah, I’m UK based too and this whole discussion is wild to me! I’ve never taken a notepad to an interview, never taken notes in an interview, never needed to, and never been concerned that peope I’m interviewing don’t do so. I have a memory that works, thankfully, I can recall afterwards if there is something I need to do. I prefer to give the interview my full attention – and it’s worked oretty well thus far (25 year career, multiple jobs, several promotions, now in senior leadership role at a major company you’ll have heard of).

                  Americans do things differently, in many ways. I’m sure it works for them.

                2. Ontariariario*

                  I’m Canadian and surprised by the strong opinions! I have my personal preferences but if someone else didn’t bring any notepaper and pen yet remembered what I had said then that works well for them.

                  When I hired someone I had a list of questions to ask and took brief notes of each answer on my laptop so that I could compare them at the end. I used the laptop because I type faster than I write, and I can share them with others more easily. I wouldn’t care if someone took notes or not.

                  I haven’t brought my CV to an interview in years and agree that it is the interviewer’s responsibility.

                3. Miri*

                  I’m in the UK and usually take notes at interviews. I think there’s a difference between expectation that you ‘have to’ and would be judged if you don’t, and the idea that they might be useful (I usually find them to be, if you don’t, that’s OK!)

              2. Lexie*

                Not necessarily remember it perfectly forever. But remember it for at least an hour or two and if I feel the need to write it down do so after the interview. But I have never been given any information in an interview that was so detailed or vital that I felt the need to write it down. It’s always pretty much been basic job duties, salary, and benefits if any.

            4. Green great dragon*

              Well, if I ask a team member to do something in a meeting, I expect them to do it without being asked twice. I’m not going to send an email afterwards just in case they haven’t taken any notes. That would not be an efficient way of working. YMMV.

              1. Green great dragon*

                Perhaps I should say that if I agree to do something in a meeting, my team members can also expect me to do it without them having to send me an email to remind me.

              2. Amy*

                Are you saying the interviewers are the ones required to take notes but not the person being interviewed?

                1. Green great dragon*

                  This was responding to Beezus’s more general point. But I do think even in interviews, while I wouldn’t require the interviewee to take notes, I wouldn’t be taking them on the interviewees behalf, so particularly if there’s a potential follow-up interview I would have thought it’s in the interviewee’s interest to take notes of key points. I don’t care at all whether it’s on sticky notes, laptop, phone or notepad as long as it doesn’t distract from the interview.

              3. Allonge*

                Totally fair for a meeting where people already wotk for you but how much of a to-do list is there after a job interview for the interviewee?

                I know thank you notes are a thing in the US and would want to take a note of names for that but what else do ypu expect someone who does not work for you to do?

            5. Yoyoyo*

              My experience is that the official minutes of any given meeting do not contain all the minute details. They are more likely to say “Yoyoyo to follow up” than “Yoyoyo to email Wakeen and set up a meeting by Wednesday at the latest.” That would be on me to remember to do, which for me, requires taking notes. I guess if you are able to just remember everything then that’s great, but it’s been my experience that many people who think they will remember in fact do not.

            6. Jamjari*

              It might just be my job plus my brain, but it’s not so much about tasks I might be assigned (which may or may not go into some tracker). I often get ideas in meetings for other things I’m working on, and I want to jot them down otherwise I might very well forget before the meeting is over. So I want some way to take notes, be it on my computer, a notepad or sticky notes.

              1. Chestnut Mare*

                Same. I’m wondering if this whole discussion is more note takers in general versus non-note takers. I always have some way to take notes, even in my leisure time. I want to jot down the name of the wine I liked at dinner, the restaurant my friend recommended, a reminder to look for cute shoes like that person over there is wearing, etc. I’m amazed that others don’t need to write all this life and work stuff down to remember it…I’m so envious!

            7. I Wrote This in the Bathroom*

              This is where I fall – in a meeting, we are either brainstorming/solving an issue together, or I’m going to come out of it with action items for myself, or both. What am I doing in an interview other than selling myself and assessing the company and the position I’m interviewing for? If it sounds great, I won’t forget. If it sounds horrible, I’ll remember too.

              Now that I think of it, mayyyybe I’d advise to bring a pen and paper for the technical/coding part of the interview, but I was always provided either a whiteboard, or a laptop, or a digital whiteboard for those.

          2. talos*

            I’ve never needed to unless I’m running the meeting – generally if there’s something I need to take from a meeting I can either write it down on my (work-provisioned) phone, or I can ask the person running the meeting to include it in the meeting notes that get distributed after.

            I would say it’s maybe 20% of my coworkers at most who bring anything to write on other than a phone.

            This is tech, across several companies.

            1. talos*

              I should say, what I ask to be included is something like “talos will do X task by Y date” and maybe a line or two of critical requirements; it’s incredibly rare that any more detail than that will be hammered out in a scheduled meeting – more detail always comes either via chat, email, face to face at someone’s desk, or in a smaller meeting (where I organized it so I am taking notes).

              Tech is weird, I guess?

              1. hbc*

                It’s pretty annoying to make sure that someone else properly documents your to-do item when you’re sitting right there. When I’m running a meeting and taking minutes, I try to capture that kind of stuff, but it slows down the whole meeting to basically take dictation.

                The minutes are a reference, not an exhaustive list of what happened in the meeting. Every individual present is 100% responsible for their action items, regardless of whether they made it into the record.

                1. Allonge*

                  I totally agree that people need to take note of their to-do items. But – why are there meeting minutes if they don’t by default capture to-do items (and decisions)?

                  And ideally, although I know this may be out of your control, the person taking notes and the person running the meeting are not the same, precisely so they can proceed without a lot of delay.

                2. talos*

                  I’m largely with Allonge here that part of having meeting minutes is that action items are captured, and this usually happens without me asking for it.

                  Wrt who takes meeting notes: I’ve always preferred to take my own meeting notes! Notes usually go in the agenda and I prefer to make the agenda for my own meetings. Maybe I’m weird.

          3. doreen*

            I think that depends – every actual meeting at my last employer had a folder full of handouts and a pad at every seat so not having one wouldn’t have seemed cavalier.

          4. Danish*

            When I worked at Major Tech Company this was not at all expected – the company provided pens and paper but only if you went to look for them, and it was never required or even implied. More like, if you’re some kind of backwards weirdo who still uses pen and paper, here you go.

            To be fair – everyone always carried their laptops everywhere, so note taking would have been possible, but most people just…didn’t.

          5. Observer*

            Walking into a meeting (especially with someone more senior than you) without anything to take notes on is a really common “I’m being cavalier about this meeting/my job/any needed follow-through” cultural signifier.

            Would you see it that way if they had SOMETHING to take notes with, even if not a pen an paper?

            But essentially, I totally agree with you. And it’s not just about action items that in some companies might wind up in a task assignment / tracking system. But information about how a project is going to be implemented, instructions on which process we us for the chocolate teapots vs the marzipan ones, or information on how to code certain items in the database.

            One of the employees who sticks in my head for how bad they were in a particular job was the person who showed up to a meeting where a bunch of people were going to try to give some guidance on some really important recordkeeping they had been struggling with. They were the only ones who had come in to the meeting without something to take notes on.

        3. Emmy Noether*

          If you are able to jot something down on a sticky note, then you did bring something (sticky note and writing implement) to take notes, or else they are provided. This counts as being prepared to take notes.

        4. rebelwithmouseyhair*

          Suppose I were to say “ok the interview went great, could you meet up with my boss next Tuesday? His schedule is open, let’s say 2pm?”
          I’m pretty sure I’d be expecting you to jot that down somewhere. If you had to ask me for a post-it, I’d look askance at you.

          1. amoeba*

            I mean, even though I usually bring a notebook and a pen (which I think I’ve never ended up actually using, but better to be prepared) – this kind of thing would just go straight into my google calendar (as in “sure, let me just save it in my calendar straight away!”). Much safer this way. Although of course normally I’d never get my phone out in an interview, but scheduling feels different? I mean, I’d probably check my availability first, anyway, so…

            1. Aussieb*

              and while I might check my availability and put a block in my calendar, I’d expect the interviewer to send a calendar invite. If they didn’t, it would be a not-green flag about the company

              1. Allonge*

                Yes – if your time and date are non-negotiable, I would expect an email/calendar invite with also the location mentioned (you need to put it in your obss’s canedar anyway, right)?

                If we are finding a time that is reasonably convenient to me, I need my calendar out, and that comes in my phone and I will take note of what we agree there. A confirmation via email is nice.

          2. Falling Diphthong*

            My tech skills are unlegendary, but even I a) carry a cell phone; b) can make notes on it if I encounter something I want to make a note of.

            I believe using that cell phone to check whether you are free at 2 pm next Tuesday, and then add that to your calendar, is a routine task.

          3. Yorick*

            I can definitely remember “Tuesday at 2pm” until I can write it in my calendar. If I did anything right then, it would be to use my phone to put it in the calendar.

          4. I Wrote This in the Bathroom*

            I’d put it on my google calendar right away, rather than on a piece of paper that’s likely to get lost and that would need to be transferred to my calendar anyway. I’d have to check my schedule on my calendar anyway, why not enter the 2PM Tuesday meeting at the same time?

            Sadly, in all my years interviewing, I never had this problem occur that they wanted to schedule the next one right away and I needed something to write the date/time down on. I’m just not that impressive, I guess? Usually it’s “ehhhh we’ll call you next week… or the one after… maybe.”

        5. Reality Biting*

          Oh dear god, this explains why some colleagues constantly outsource their memories to me. Being in a supervisory role, I’m apparently expected to remember everything for them and explain over and over again how to do things.

          For the love of god, just please write it down!

          1. Allonge*

            Gently, if you are in a supervisory role, you can in fact tell the people to write what you say down, expecially if it’s an issue.

        6. Starbuck*

          “arrive at a meeting with something to write on, ”

          “why do I need to arrive to a meeting to take notes this isn’t a university lecture. I’ll jot something down if needed on a sticky note”

          Well yes, that would be the sticky note you mentioned. Also, where I work it’s very few meetings that don’t have some kind of action-item or decision made that I’ll need to remember! Otherwise, why else meet?

      2. *kalypso*

        It’s a socially acceptable fidget for when you don’t have the information to risk disclosing a more specific one.

        1. CrazyJob*

          Nod. My notebook is a fidget. Yes it does have lists in it, but I’m so confused by taking notes in an interview. The interviewer usually quizzes you about how you handled blah blah , and you can ask a few questions, but you can only ask the ones you want answered ( pay, vacation, etc) on the second interview

          1. BenAdminGeek*

            That hasn’t been my experience- even in first interviews I’ve been able to ask questions about the job, team, etc.

        2. Yorick*

          Ok sure, but that’s a tool for the candidate and not something an interviewer should expect everyone to bring.

      3. L-squared*

        This is one of those things that, for people’s first jobs, I’ve heard that advice too, and would probably give it. Mainly because there are super judgy people out there who would make some wild leaps in logic, like OP is, if you don’t. But at this point in my career, I don’t feel the need to. And if a company wants to not hire me becuase I didn’t do some pointless old fashioned thing, probably not where I want to work anyway.

    3. ENFP in Texas*

      I’m interviewing now for some internal positions and I am taking copious notes – what the role entails, responsibilities, org structure notes, manager expectations, what sort of systems and software are used, etc.

      I can’t imagine NOT taking notes during an interview, because I’m going to learn new things about the role and need to make an informed decision about what the role actually is beyond what’s in the job posting.

      1. Allonge*

        To be honest it’s wild to me that a lot of this info (especially internally!) is not available to you in writing and/or via some informal channels.

        Manager expectations, sure, but org structure, responsibilities, what the role entails should not be something that is a mystery until the interview. Maybe I just worked at especially well-documented places.

        1. BenAdminGeek*

          I find it’s the nuance that you get in job interviews- “Ok, so the job is 80% llama-grooming right now, but we really want to grow Trainer X and Trainer Y to increase it to 50% llama-consulting by the end of next year. They’ll start reporting to you soon, but remember that….” That helps me understand the vision for the role. Of course things might change, but remembering the nuance is the hard part for me, so keeping track is good.

          1. NB*

            Yes, this is definitely where it helps. Just noting down phrases that jog my memory for those details is useful.

      2. Weaponized Pumpkin*

        I tend to get a lot of “This role reports to me, I am the overall manager of the department. But day to day you’ll be dotted line to Jane, who leads tech marketing and reports to the CTO but also on the team is Greta, a senior content developer mainly for the operations products and Bob who handles writing for applications but also the keynotes for the CTO. In the fall we all pitch in on X initiative…”

        I write that shit down.

    4. jamlady*

      I am finding this particular topic fascinating because I didn’t realize there were expectations around this either way. I am senior and never bring notepads to interviews. I’ve elevated in my career quite successfully and I have worked in varied industries, so it does not seem to be industry-specific. Thinking back, I believe most candidates I’ve interviewed came with just their person. I do remember one candidate a few months ago that was prepared with questions, notes, etc. and it was noticeable that she came with such items (not negative though). I’ve only ever had a couple of junior candidates bring resumes. I feel like this topic needs a geography/industry/age poll!

      1. Violet Crumble*

        I am intrigued by this topic as well. Similar to jamlady, I’ve worked in various industries and have interviewed hundreds of people throughout the years and can’t recall any candidate taking notes – most people didn’t seem to have anything on the conference table in front of them. I never took notes while being interviewed but did bring a notebook etc.
        Although I’ve never attended a meeting without a note pad and pen… This is for multitasking as much if not more than taking notes as I jot down things to add to my task list or ideas – totally unrelated to the current topic of discussion… I’ve worked for so many companies that have way too many unnecessary meetings and have spent way too many days in back to back meetings. A little off topic but a huge pet peeve.

      2. oranges*

        I really think age/generation is a big piece. I have kids in school, and their *entire* day is digital and all their work on screens. They don’t print anything, they don’t have paper notebooks or planners, and with the exception of math, they barely write anymore.

        I work in an office and would know to advise them to bring a notebook to an interview, but without my lived experience, I can see a world where it wouldn’t occur to them to go out and buy one.

        1. BadCultureFit*

          I also have kids in school and they absolutely write! They have computers too but only use them for specific programs (even math, though you seem to think math is only by paper). But both kids come home with stacks of papers!

    5. fluffy*

      I’ve been a software engineer for around 20 years and have been on both sides of the interviewing desk. Maybe it’s something specific to software, but I have never once been expected to take notes as a candidate, nor do I recall ever seeing a candidate take notes for later. For any followup-related stuff that might come up, people might jot it down in their phone or something, but that’s also incredibly rare.

      So, this could be hugely industry-specific advice.

      1. Roland*

        I’m a software engineer and in the prepandemic days where I interviewed people physically, a good amount had a notebook or something with them. They weren’t “expectee to take notes” like a junior person recording a meeting, it would have questions they want to ask and anything they asked me and wanted to record my answer. I would find kt pretty jarring if they pulled out their phone to take notes tbh. And I’m “young people” and was mostly interviewing other “young people” due to needing to have the same experience or more than the roles I was interviewing for.

        1. Observer*

          I would find kt pretty jarring if they pulled out their phone to take notes tbh.

          Interesting. Why?

      2. KatEnigma*

        My husband is Sr level software engineer who has worked in 3 different industries, and has always taken a notepad for notes. If for no other reason because things you are told in those interviews sometimes change later, and he wants his notes to refer to, to clear up any misunderstandings, or know to decline an offer from companies that can’t even keep things consistent during the interview and offer stages. He would consider having his phone out during the interview to be a potential distraction to himself. I don’t know if he expects interviewees to take take notes, but certainly he wouldn’t be taken aback by it.

    6. Ellis Bell*

      I never realised either, mainly because I have never been in an interview situation where note taking felt natural . I usually have a notepad and pen in a pocket of my bag (interviewing and shorthand were my job for many years) so if something I had to note down had come up, I would have been able to reach for them, but it never has. Weirdly, my memory is not great and if I’m in a meeting or training, I take notes to keep focus and my ADHD at bay. But an interview usually has the opposite effect; it’s often a fast moving conversation which has me so engrossed, that there’s no opportunity to take anything down. I actually do note down some of my own questions pre interview but I prefer to memorize them, like you would with a speech or presentation. I think I’ve only ever referred to my notes when I’ve been really nervous so this is super interesting to me that having some paper out is seen as more prepared.

    7. Phil*

      Same! I’m late 30s and got my interviewing knowledge from high school careers class, and that was never covered. Having said that, outside of retail etc in younger years, all my career-centric interviews have been at the one company, so not sure if the expectation is different when interviewing within… also I’m Australian so this could be an America thing (like the whole thank you notes business…)

    8. Madame Arcati*

      For meetings I would always expect to see a means of note taking.
      However in my world (U.K. government work, national level) you actually may not make notes. You may bring a few prompts as you will know the competencies you’ll be asked about (when they ask about leadership, talk about the South American research trip and mention the sloths) but you will be marked down if you are reading notes out.
      You may scribble a note of the question/competency to keep your mind focused while you answer but you may not take such notes out of the room. This is for security and to prevent the specific question being passed to anyone else not yet interviewed.
      I’m sure note taking in interview is advisable in many situations but it is not universal and I don’t think government work is very niche as an area.
      I do notice that LW says she doesn’t need a resume copy nor does she actually care if they take notes so…she just wants them to be holding something?!?

      1. doreen*

        Same in my world of state government. There’s always a pad in the interview room for candidates to use for note taking – candidates have to leave their notes behind and there are notebooks where tearing out pages doesn’t work well. The postings provide a detailed job description and the interviewers can’t answer questions about benefits or salary – not because they aren’t permitted to but because the answers depend on whether someone is a current state employee , and if so for how long and what is their current salary and grade ( a rare situation where the “new employer” knowing the salary helps – you are guaranteed a certain percentage raise for a certain number of grades ) , if they will be represented by a union and if so which one , and which of the many health insurance options they might choose. The only question I remember being asked by candidates that I could actually answer was whether the schedule was strictly 8:30 – 4:30 or was a workday starting at 8 or 9 possible.

    9. Moo*

      Honestly I’m really weirded out by #4. I’m at the point in my career that I’m now senior management and I have never brought a notebook and pen to an interview! And I have done so many interviews (many of them successfully!!)

      I’m not in the US so maybe that matters. Some years ago I switched to interviewing more than I was interviewed and I was surprised that anyone brought notebooks – moreover I saw people refer to notes they had made earlier to give answers. Still I’d say I’ve seen less than 20% of people bring notebooks to interviews.

      I do lots of prep beforehand, and when I leave I usually made a point of writing down everything I remember from the interview afterwards – usually the questions I was asked.

      It might be that I think of interviews as inherently different to meetings, there’s an element that’s more performative. It might be that I mostly experience panel interviews, and the US system seems to be a series of 1 to 1s or 2 to 1s. I expect to be doing most of the speaking as an interviewee and as I need to be very present, I suspect that notetaking or thinking about writing things down might be more distracting to me. I haven’t really had cause to write anything down in the interview, or wish I had done – as I said afterwards I write down the questions asked, but that would be a weird thing to do in the interview. So maybe its a feature of different systems, or maybe I just never thought about it before!!

    10. S*

      I had a job interview this morning for the same job I interviewed for last year. (The hiring manager reached out to me and asked me to apply again.) I wished I had taken better notes of interview questions at the time, to be honest.

      The other reason to write things down is that it makes it easier to write thank-you notes that aren’t carbon copies. If John talked about a background in llama research, I can refer to that.

    11. nonprofiteer*

      I take some notes at job interviews, but find it is a little awkward to to – it can be hard to pay attention and write down notes at the same time. So my notes are kind of scattershot because I’m not giving them the same attention I would if I was just an audience member to a conversation and not a participant. Still, it was very useful to capture names of people I interviewed with and their titles, as well as some details about the job – I also used it to write my questions in advance so I would remember them.

  4. Eric*

    #4, I wonder if part of it is that people don’t use paper much for taking notes anymore. I use my laptop for taking notes at virtually all meetings, but would feel pretty awkward, as a candidate, pulling it out to write stuff down in the middle of a job interview. leaves everything feeling awkward.

    1. OP4*

      This is something I’ve been thinking about as well, reading the comments. Not sure if y’all would agree, but it would be so strange for an in-person candidate to take notes on their laptop, but not so much so for a virtual candidate (if they told me that’s what they’re doing) or in any other type of meeting. That said, we interview by panel and take notes in a shared file, so we all have our laptops out and are typing periodically during the interview (we explain this at the jump).

      But if the candidate took notes with an electronic pen/stylus on a tablet, I wouldn’t think twice. As I’m writing this I’m seeing it’s about signaling preparedness, but in a specifically slower, thoughtful, non-distracted way. I want to chew on this more, as I want to understand for myself what other signals for the same trait I would more readily accept and why.

      1. OP4*

        Also want to add that taking notes on a phone (or a sticky note as obliquely mentioned elsewhere here) sounds completely WILD to me, but I cannot, for the life of me, articulate what the material difference is. (I could go down this rabbit hole forever…are crayons and construction paper okay? Buddha boards? I’ll stop here lol)

        A lot of this questioning is prompted for me in making sure that when I talk about professionalism with my direct reports and with more junior staff in my department, that I tie expectations around behavior to desired outcomes and firm goals rather than “what is typically done.” There are some things I can’t get around (think: illogical elements of business casual dress code that I can’t change) but I can say “bring something to write on and with because our work is complex and you missing even small details can slow us down and may impact quality/budget/schedule, etc”.

        In an interview, though, the stakes and dynamics are so different. It’s so interesting to hear everyone’s thoughts!

        1. Former call centre worker*

          As others have mentioned this may be a cultural difference, as I am UK based and have never needed to write anything down in an interview and can’t even think what I would need to write down – but if I did need to, I would use the notes app on my phone. I’m not going to bring a laptop to an interview or buy a notebook that will remain otherwise unused when I can just use my phone

        2. Beezus*

          to be clear I would not roll up with a stack of sticky notes for an interview and I think the interview vs meeting stuff got sidetracked. I do think there is still a bias you’re “not paying attention” if you take notes on your phone or a laptop (unless you’ve been designated to do so), even in some meetings, nevermind interviews.

        3. *kalypso*

          Part of it may be that ‘being on a phone’ and ‘writing on paper’ have different social expectations and assumptions attached to them – typing on a phone is usually taken to be texting or scrolling on social media, writing on paper is ‘taking notes’ and never assumed to be ‘drafting the next great romance novel’ or ‘doodling manga’ or ‘distracted’ unless someone can see the paper and doesn’t approve. So the reflex is to not send the wrong signal, not so much about the phone itself.

        4. Critical Rolls*

          I agree with *kalypso that the signal given off by messing with your phone is, in my experience, not a good match for the interview setting, no matter how innocuous the action is (with the exception of scheduling something). At the point of interviewing, the interviewer(s) don’t know you well enough to know if you are taking notes, checking texts, or even looking up something you should know! Your colleagues hopefully can say, oh, that’s just how Crit is, all notes on the phone all the time. But your interviewer(s) would have to take it on faith and generally you don’t want them to have to take anything on faith if it can be avoided.

          1. Observer*

            If the interviewee tells the interviewer what they are doing, why do they have to take it more “on faith” than if someone takes notes in a notebook? After all, it’s quite easy to scribble all sorts of stuff that aren’t actually relevant notes in a notebook.

        5. Observer*

          Also want to add that taking notes on a phone (or a sticky note as obliquely mentioned elsewhere here) sounds completely WILD to me, but I cannot, for the life of me, articulate what the material difference is.

          I think it might be worth your while to think about why you see it so differently.

          You mention signals of traits that are important to you. Why is using a phone for note-taking not a signal of preparedness? There are a lot of situation where a phone is likely to be a lot less intrusive than a laptop.

      2. Green great dragon*

        I’m hardcore about working off laptop not paper, so I can keep notes, search them, quickly and unobtrusively look things up in meetings etc. And I definitely get the odd mildly surprised look when I go to board-level meetings (nope, I don’t need a print-out of the slides for lo! I have them here, complete with access to the notes pages and working hyperlinks). But I think people will get used to it, and I’m willing to be an example.

      3. Catwhisperer*

        One additional thing to think about if this is mostly younger candidates: Some people in their 20s have only ever taken notes via computer or tablet. I’m in my 30s and even we had computers in classrooms, they’ve gotten even more ubiquitous since then. My nieces and nephews started school right before the pandemic and most of their work is done via tablet. It’s very possible that candidates in their 20s don’t want to bring a full laptop to an interview but have never used a notebook for note taking before.

        1. PurpleShark*

          Just to add, (high school counselor here) my students struggle to write manually anyway. When I ask for initials or a signature they are like little old people – it appears to be a cumbersome process. They do not know cursive so all of it is printed which can be slower anyway. If they did learn cursive they don’t use it enough to be efficient. Don’t get me started on them reading Clockface. I have a vision of them taking notes in this situation but it being a very slow action while the interviewer waits.

      4. Elspeth McGillicuddy*

        Laptop stand between people-like, their screen physically sticks up in the air. The others tend to lay flat on the table and are at least smaller obstacles.

      5. nnn*

        The “thoughtful, non-distracted” part is interesting to me, because I find taking notes on paper to be so distracting that I can’t follow the conversation. By the time I’ve got the thing written down, I’ve forgotten what we were talking about.

        Even in school, I’d sit there in the classroom with a paper notebook in front of me because that’s what you’re Supposed To Do (this was in the days before it was common for students to have laptops) and write nothing whatsoever, because I was too busy listening and learning and asking questions to understand.

        In contrast, if I’m typing on a laptop with an appropriately ergonomic keyboard, I can literally transcribe the conversation while also participating in the conversation. But people get weirded out in some contexts if you show up with a laptop.

        I find a phone is a happy medium for if I just need to jot a couple of things down, but people perceive looking at your phone as inattention.

        So it seems that the optics and the functionality are almost entirely at cross-purposes in this issue!

        1. foolish fox*

          I cannot take notes during lectures. I ended up coloring through all my law school classes because I absorb more that way. However, outside lectures, I carry a notebook with me all the time. It’s my external brain. Anything I think of that I need to remember gets noted. Tasks, groceries I need to buy, books I want to read, the neighbor’s kid’s activity I want to look up, a random idea for a halloween costume next year. It all goes in a daily running log and then organized later into to-do lists and various other notes.
          I just can’t listen/interact at the same time as writing.

      6. Kelly L.*

        It really does kind of sound like you want to see the notebook as kind of a ceremonial thing! I don’t mean that as a criticism, it’s just interesting to think about.

        1. OP4*

          I totally agree! I can’t explain to even my own satisfaction why it makes a difference. I don’t need these notes. To be fair, it’s not much of a difference but I do notice. This feeling is the crux of my question. Is there a practical, non-ceremonial reason for this convention? If not, I can work on letting it go as an interviewer. I’d still bring my own notetaking gear though if I were an interviewee.

          1. Observer*

            Is there a practical, non-ceremonial reason for this convention?

            Unless there is a lot of information about your field that you haven’t mentioned, I would have to say that no, there is no real practical non-ceremonial reason for this expectation in interviews. Especially if you are reacting negatively to the use of a phone.

        2. Master Procrastinator*

          I agree about the ceremonial element. In my consultancy work I often encourage leaders to unpack norms, traditions and “the way things are done around here” to check that they make sense in the workplace as it is now, that they are equitable/don’t present barriers to access, and that they’re not based on unconscious bias. I think the concept of “phone = unprofessional, paper = correct,
          laptop = somewhere in the middle ” is a great example of this. I write much more slowly than I type and struggle to keep up with the pace of conversation while writing, so I’d always take meeting minutes digitally. And as someone with ADHD, I find that any note taking slows down my interaction with the conversation, so it would trip me up at interview as a candidate. I would absolutely take out my phone to use my calendar if any scheduling was required, but can’t think of another reason to take notes in an interview (as opposed to after). I am baffled by the concept of not knowing salary range, benefits, hours or other crucial information before attending interview. And equally baffled by the idea of reviewing CVs at interview. Am I an outlier in only having ever experienced recruitment (both sides) where all the essential info is posted on the job ad, interviewers do their homework about each candidate before their interview and the specifics of the role will be confirmed when the offer is made?

      7. korangeen*

        I brought my laptop to an in-person interview a few weeks ago, and if any of the interviewers thought it was weird, they didn’t comment on it. I did ask the recruiter ahead of time if it was okay. But my thoughts were—for any other normal meeting, I’d be bringing my laptop. Why the heck can’t I for an interview? And obviously we’ve all been using laptops this whole time throughout virtual interviews. It seems ridiculous to have to make a trip to the library to print all my prep notes, and then be shuffling through papers at the interview. And I can type a lot faster than I can write. I don’t know where this concept came from that it’s so strange for a candidate to use a laptop during an interview, but I wish we would all kill that line of thinking immediately. Job interviews are stressful enough without having to worry about being harshly judged for using a note-taking/reading device that’s completely normal under other circumstances.

    2. Lacey*

      I always bring a notepad and pencil to job interviews because I know there will be stuff I want to remember and I’m certainly not whipping out my smartphone to jot down the notes. But in general, yeah, I don’t carry a scratch pad with me like I used to.

  5. Louisiana Jones*

    # 1 — Oh wow – that would annoy the heck out of me as well. I’m a private person about things that aren’t someone else’s business. If they needed to know, different story. But just being nosy? Nope.
    Don’t let it continue – use what Alison wrote. And deliver it with “a look” as if you can’t believe they’re asking you or why they’re assuming it’s their business.

    1. Pennyworth*

      I’d reply to ‘Does Boss know?’ with ‘Of course’, and to ‘Just checking’ with ‘I don’t need to be checked, thanks.’

      1. I should really pick a name*

        I don’t need to be checked, thanks

        There are definitely less antagonistic ways to phrase that.

        1. Putting the Dys in Dysfunction*

          The antagonism is coming from the coworker, and it seems appropriate to shut it down firmly.

          1. I should really pick a name*

            There are ways to shut it down firmly that aren’t going to make you look bad to other coworkers.

            If someone sees you react like that without knowing any of the history, it’s going to reflect poorly on you.

        2. Caliente Papillon*

          How is “I don’t need you to do that” antagonistic? It’s direct. The asker is being antagonistic.

        3. Susannah*

          Yeah, but the questions are so antagonistic it might be worth saying, at least once, to jar the nosy co=worker.
          And I get the sense from the letter that co-worker is trying to establish him/herself as “above” LW somehow – with the authority to monitor. I’d shut that down very bluntly.

      2. Colette*

        Honestly, the only people I can picture saying “I don’t need to be checked” definitely need to be checked. Something that hostile is just not something competent people say.

        1. Non non non all the way home*

          As a competent person, I think “I don’t need to be checked” can be stated in a way that comes across as assertive, rather than aggressive. It doesn’t work in written form, but it could be said in a conversation with the co-worker.

          But, as I can’t possibly know how all competent people behave or what they say in all situations, I will defer to your omniscience.

      3. Some words*

        I wouldn’t even tell them that the boss approved. It’s between the employee and the boss, period. I might feign ignorance and ask if there’s been a change in time reporting requirements. “No? Okay.” Then change the subject.

        I’ve worked with too many busy-bodies over the years who live to try to get co-workers into trouble.

      4. Kella*

        “I don’t need to be checked, thanks” honestly comes across as defensive, and I feel like would undermine your point more than emphasize it. It also sounds like it’s inviting further argument rather than shutting down the conversation or pursuing information from the nosy person.

      1. Emily*

        Yeah, I was thinking along the lines of “just ignore it” too. However, there may be reasons why that’s not the best route, and I think Alison’s suggestions are good ones, and I especially think there can be a benefit to directly addressing the pattern.

      2. Eldritch Office Worker*

        I’ve been working really, really hard on ignoring messages that I have no kind response to and think are out of line or unimportant. It’s going to send me into fits one of these days, but it has had positive outcomes.

        Not that I was getting into tons of petty arguments before or anything but yknow – every office has these passive aggressive interactions here and there and while Alison has armed me with tons of professional responses, ignoring is sometimes best.

      3. Palliser7*

        Yep, that’s what I would suggest too. It’s not relevant and none of the coworker’s business. I am guessing the coworker is using IMs because they are on the more casual end of corporate communication, and OP can use that. Strategically not answering IMs on irrelevant matters can train the colleague on what will/won’t get an answer. The truth is, the colleague doesn’t have the power to police the OP, and sometimes reinforcing that truth can be useful. Sometimes power plays are very necessary.

    2. Manglement Survivor*

      Yes, instead of saying, “why do you ask”, I might say, “why are you asking me this when you are not my superior?” But since it’s been going on so long already, and I’d be pretty angry by this point and would say “why do you keep asking me questions like this when it really is not applicable at all to your job? Work matters between myself and Boss have nothing to do with you.“

    3. Ellis Bell*

      I work in a type of field where it’s acceptable for a certain amount of emails to go unanswered because we’re away from our keyboard so much. It’s not explicitly intended, but it’s understood that queries get prioritised. This kind of nosy query from a peer would definitely go unanswered! I don’t know if OP can just let this stuff drop or radio silence her on chat, but if not I’d probably make it very much an ‘information diet’ answer as well as making her explain the query until it becomes a thoroughly unsatisfying habit. “I’m extremely busy right now and that’s a bigger question than it appears; do you want to let me know what the information is for and I’ll get the relevant details to you accordingly” or “That depends what you mean, can you tell me why you’re asking first so I can give a more precise answer?” or “I’m really confused; my information was that you weren’t playing a role in this project, do I need to double check that?” or “I’m going to need to check with boss because I thought she already knew this detail; did she say why she is asking you for a second follow up?” or “Am I missing something here? I thought everyone involved had been briefed on this by Boss?” She will probably respond to these questions with ‘just checking’ but you can shuffle these responses without ever giving her any meat, until she gets tired of saying that. Or you can make a big fuss over being asked at all: “Phew, glad to hear there’s no issue; I thought something was actually wrong then!” and just add no actual answer at all.

    4. Penfold*

      I’d be so tempted to either say, “why wouldn’t it be?” or “none of your beeswax”.
      One of my colleagues was incredibly nosy about things like that and when I had to do reduced hours for health reasons she would never ask directly why but say things like,
      Why are you leaving before me? – ‘Because those are my hours today’,
      You weren’t in yesterday – ‘Nope’
      Should you be leaving this early? ‘Yes’, and with that one I’d skip off home with a smile.
      Thankfully when she’d quiz our manager to try and get the goss she was always met with a brick wall.

  6. Beezus*

    American resumes are worse: “I led a cross-functional project to identify, cluster, and describe visitors to Lucious Llamas Ltd. The resulting campaign delivered a 41% reduction in cost per llama groomed.” should be

    COMPANY – DATES
    job title
    * delivered 41% reduction in cost for our propriety llamas
    *spearheaded social media marketing campaign

      1. Lacey*

        Yeah, the way Allison said to do it and the way Beezu is recommending are how I’ve always been told to format my resume.

    1. Snell*

      I can’t tell what you’re trying to say. If you mean that the first example you mention is the “worse” American resume style, AAM’s answer straight up says that’s not the standard convention in the U.S., and does seem to favor the second format in your comment. Also, the first example you mention (the “bad” one) is from a LW in the UK.

      1. An American(ish) Werewolf in London(ish) aka Pookie Sparklepants*

        “the “bad” one” … I feel seen ;)

        I don’t think, though, I’d go as far to say it’s ‘bad’ – it may not be how you’d do it or what convention ‘says’…but I’d like to think it’s not outright BAD. :D

        All food for thought though – thank you!

        On a related note, presumably using ‘I’ is fine in the personal profile/statement section?

      2. Hlao-roo*

        Beezus means American resumes are “worse” as in “worse for LW5’s sense of grammar.” The first example is from LW5, and the second example is the translated “worse” (briefer) American style.

        1. Snell*

          It’s still not super clear to me that your interpretation is what Beezus meant. They provide a correction (“___ should be ___”), but say only “American resumes are worse” and no other elaboration, so I read it as Beezus saying the first version in need of correction was the worse American style…except the only proponent of the first style in this situation is the LW, who is in the UK, presumably looking for advice that would be of use in the UK. The second example may be the American style, but according to the LW’s daughter, it’s also the style expected in Australia.

    2. Melissa*

      Americans don’t typically write like your first example. We do:
      -Promoted to manager after eight months.
      -Sold four thousand copies widgets

    3. Miette*

      Well I don’t know how it is in other countries, but in the US we are very constrained by having to try to fit everything onto two pages–goddess forbid you go to MORE THAN A SINGLE PIECE OF PAPER OH NOES. So yeah: that’s why the brevity.

    4. Another Remote Worker*

      I’m confused – the examples at the bottom are typically how American resumes are written, not with long sentences.

  7. FG*

    #4 I am late late Boomer / early GenX and even in the analog days I can’t say I ever specifically took something to take notes on. I’d have a purse so I’d have a pen, & copies of my resume (photocopied, in the earliest days typed on a typewriter). But it would never occur to me, even in the interview for my current job, that “OMG must have notepad & take notes.”

    1. Beezus*

      bringing a notepad feels weird and I have brought my resume bc not everyone has a printer these days and I still felt goofy bc when I’ve done hiring I know who I called to talk to IRL I wouldn’t feel weird if they took notes but mostly also I assume they know the job role/company they’re interviewing at by that point.

    2. Turquoisecow*

      I don’t think I’ve ever taken notes at an interview. They might give me a sheet of paper with a more detailed job description, or a copy of it, but I don’t usually see a need to add to it. If we schedule another interview, I can use my phone for that, and in fact would need to as that’s where my calendar is, so no need to write that down.

      Once you get the job, there’s lots of notes to take during training and etc, but in the interview, no

    3. anna*

      Nearly everyone I interview brings something to take notes on, whether or not they actually use it. Coming without a pad and something to write with looks unprepared.

      This is the first thing we teach interns—bring something to write on whenever you go to a meeting.

      1. Pink Flamingo*

        I now always take a notepad and pen, and ask at the start if they mind if I also take notes. I jot down key words in the questions so I don’t forget (especially if they’re two part questions), I can have all my questions for the panel listed and ready to go…and because my typical working style is to take notes in a meeting!
        No one’s ever said no, and honestly, half the panel are always scribing down answers on their matrix anyway.

      2. Bit o' Brit*

        An interview is very different to a meeting, though. Meetings generally have actions for me, interviews generally don’t. They have actions for the hiring team, so they’ll probably need notes, but all I need to know is “do I think I want this job?”

        I always have a notepad and pen in my handbag, so technically I’ve brought one to every interview I’ve ever had, but they never left my bag. Nor did the copy of my CV that I brought specially.

        1. Ellis Bell*

          Bizarrely, even as a non-note taker I would feel like I “looked unprepared” if I did not have a good bag with me. I usually interview with a more briefcase or satchel style bag; the kind of thing you could pull a tablet or notebook out of. Even though I typically don’t! I’m well aware this is a weird inclination though.

          1. Moo*

            in my last set of interviews I had this conundrum. My current bag was looking a bit worse for wear but I love it and its a great size and function, and honestly I was asking myself if I really was going to buy a new bag just for interviews. I settled on a sort of folio thing – it even had a little loop I could attach my keys to. I’ve carried it in but then never opened it – but at least my phone and keys were tidied away!!

          2. Eff Walsingham*

            While reading these comments, I’ve had the brilliant brainwave (well, it’s probably just the cold medicine making me overestimate its value) that the bag is the most important thing. It contains pen, notebook, extra resumes, breath mints, bandaid, spare tights… basically anything *I* need to make myself feel more relaxed and better prepared. But the other thing that’s important about it is that, if I’m making a good impression overall, the interviewer may suppose the bag to contain all the things *they believe* a Successful Business Professional carries. Which might be nothing more than a clean hankie and an Emotional Support Iguana – who knows? It’s Shrodinger’s Briefcase.

            The notebook does typically come out and sit on my knee / the table with pen poised. I probably note down an average of 3 words per interview, lifetime, so far. But I’m prepared to write more, should the need arise. As someone posted above, there has been at least one occasion where the interviewer mentioned an excellent resource to learn more about part of the job description where my experience wasn’t as strong. Naturally, I thanked them and made note of it.

            Is all this performative? Yes, of course there’s that side to it. Job interviews are Show and Tell for the professional world, and this stuff is part of the show that the audience may or may not expect, and the candidate won’t know if it’s made a difference.

            But empty handed is empty handed. Maybe the interviewer will be of the type to think it looks unprepared, like you forgot you had anything scheduled OR you’re trying to project Confidence! I’ve learned from today’s column that I *do* have a bias. Clearly some commentors are sure that they won’t need to write anything down, or they can use their phone or whatever. But just know that you’ve missed the chance to score a point with an interviewer who has a bias, conscious or otherwise, in favour of the notebook or the briefcase or the ‘dumb folio’ (loved this!). Should it matter? Of course not! But only the some interviewers will look beyond scuffed shoes or a stained tie, even when the candidate is otherwise excellent, and I think this quibble is in the same vein.

            I’m sure we all wish for the sort of colleagues we’d get if HR and hiring managers based their decisions on substance and not style. But in my (GenX)perience, one often gets the people who know how to showcase themselves the best, and sometimes they do well in the actual job, and sometimes they don’t.

            1. Joielle*

              Schrodinger’s briefcase! I think you’re onto something here. Interviewing IS inherently somewhat performative – you’re projecting the best, most professional, most competent, most well-spoken version of yourself. And carrying a professional bag, having a professional folio, etc are part of projecting that image. (Same as wearing a clean outfit, brushing your hair, being extra polite to everyone…)

              Certainly, there are jobs and industries where that specific professional image matters less, and maybe that’s why there’s such a stark divide in the comments on this. But for my field (government lawyer) it is often pretty important to have those types of professional signifiers in an interview, especially once you get beyond entry level jobs and should theoretically be aware of those expectations. Not everyone will care, but I personally would just play it safe and bring the notebook.

      3. Some words*

        That’s how I’ve always done it. I’d much rather have it and not need it than look unprepared by not having anything available when it’s needed. For meetings, I can usually tell ahead of time if notes may be required.

        Full disclosure: I mostly use my cell phone for calls & texts, not as a mini computer.

    4. Fran*

      I don’t remember ever bringing something to take notes (Canadian Millennial here). Guess I should?

      1. Spill all the tea*

        I’m in Australia and have never taken notes or brought a copy of my CV to job interviews. This week I’ve been interviewing people and no one brought anything in at all. It must be a US thing!

      2. Willow Pillow*

        Fellow Canadian millennial, I’ve always brought something to take notes (and I have taken notes at times). Even where someone doesn’t take notes, it feels like such an easy way to improve your chance of success.

    5. irene adler*

      Ditto.
      I remember the suggestion to bring something to take notes on “just in case”. They key thing was the extra copies of the resume. And separately, a list of references-in case they ask (never volunteer this).

      Regarding notes, this was also suggested: before hauling out the notepad and pen, one should ask the interviewer “Do you mind if I take some notes?”.

      These days, for every interview I do, I take notes. At the screening interview, where a lot of information is dispensed, I find this essential for reference later on as there’s usually a run-down of benefits, description of the work team, culture, circumstances necessitating the hire (brief info about new projects), specifics of the job not indicated in the job description, names of hiring manager and other personnel involved with the position, timeline for when to hear back.

    6. Zzzzzz*

      I don’t get why one doesn’t have a list of Qs for your interview, and space to write answers.

      1. Allonge*

        I have a list, but it’s in my brain because in the last 44 years I have learnt that my brain reliably holds that kind of information.

        How many questions can you ask as an interviewee before it gets weird anyway? Three? Five? I know that some people cannot remember that many and it’s perfectly ok for them to use notes to help with that but I can and I will not do a perfomative List of Questions TM to show I am Thoughtful.

    7. Dust Bunny*

      I think I had a notebook in my possession at my last interview but I am pretty certain I didn’t use it. If I did, it was for a page or two of minor things. It just wasn’t how the interview worked. (I did get the job.)

    8. Lime green Pacer*

      Late Boomer here. I don’t go to any meeting without pen and paper. I keep a micro-notebook and pen in my purse at all times. (Check out Wellspring Flip Note and Levenger Pocket Briefcase for two different sizes and styles.) I do this because there are some settings where they discourage electronics (e.g. doctors’ treatment rooms), because it takes too long for me to type on a screen compared to writing on paper, and because batteries die when you least expect it.

      I am questioning this habit, though. Up until recently, it took a lot less time to whip out pen and paper than to start up a device. With facial recognition and finger ID, plus faster tech, it’s a lot quicker than it was. However, I will never give up carrying paper and pen, as you can’t leave a note for someone with your phone!

  8. nnn*

    I’m very curious what kinds of things OP#4 expects candidates to be writing down?

    I’ve never been in a job interview where there was information conveyed that I needed to remember and wouldn’t remember unless I wrote it down (and my career began in the 20th century), so I’m wondering if I’m missing something or in vastly different interviews or something else.

    1. anna*

      Alison gives examples above.

      Sometimes I wonder whether this post going up in the middle of the night means the first commenters are European and don’t reflect American norms because the response on this so far is blowing my mind and maybe it is cultural. I interview potential hires nearly every week and most people bring something to take notes. It stands out when someone doesn’t do that.

      1. Beezus*

        I’m American and in HR and like what am I supposed to take notes on? they’re going to send me the benefits package etc in an email. I am being interviewed by my potential boss who presumably knows my experience or I would not be there.

        1. Ask a Manager* Post author

          Something they mention that you want to follow up on once they ask for your questions … something that’s unclear that you need to probe further … a detail that surprised you that you want to make sure you don’t forget to think over later … some info on logistics that you want to be sure to retain … there’s a ton! I’m surprised so many people here are saying they don’t bring anything to take notes on. Not that they don’t end up taking notes, but that they don’t come prepared to do it in case they need/want to. It does make you look less prepared; it’s a pretty typical expectation that you will be prepared to do it (as it is with most meetings).

          For the record: as with all meetings, you should go to an interview prepared to take notes in case you need to, y’all! Most people do, and there are indeed interviewers who will look at it unfavorably if you don’t (whether you think that’s reasonable or not).

          1. Pierrot*

            I’m a bit late to the game but I’m slightly confused as well. I carry a purse with stuff in it to interviews and I put a notebook in it, as well as a pen, my phone, etc. I bring the notebook because it has my preparations note on it so I can review while I’m waiting, but it never occurred to me to have my notebook out on the table during an interview. If something came up in the discussion where notes were warranted, I would take the notebook and pen out of my bag, but that has never happened.
            I definitely understand bringing notebooks to other kinds of meetings and I carry one with me in general, but with interviews, I’m just not sure what I would write down unless the interviewer said “can you send me a writing sample?” or something to that effect.

            Now I’m wondering if interviewers have thought that I was unprepared in the past because it wasn’t out on display! I just got a job so maybe they realized my notebook was in my bag ;)

            1. Hillary*

              I’m late gen x / elder millennial. I’ve done a lot of interviews where I meet with a lot of people over a couple hours. My notes will look like:

              1 – Dale – HR woodworker
              2 – Steve – boss golf
              3 – Tom John – peers (drawn in relation to me at the table)
              4 – John – grand boss art

              Who, in what order, what their role is, and any interests we talked about. I may also write down who else they introduce me to in the hallway. This lets me send good thank you notes and prompts reflection after I leave. If I didn’t like Dale (and I actually did, he was great), well, we won’t work closely so I don’t have to like his hair. I don’t usually write down their questions but I do write their answers to mine. If it’s multiple people above I may note what they asked and which story I used so I can use a different one for the next person.

              Phones or laptops feel off putting to me because you lose the flow of conversation and eye contact with them. Yes, I can keep that going with a laptop, but it’s much more challenging for me.

          2. Varthema*

            As someone else who hasn’t necessarily brought paper and pen to interviews, I can see what you mean about bringing it in future just in case, but there’s no way I’m able to take the kinds of notes Alison describes during a job interview. I don’t know if it’s ADHD or nerves but I take good notes only in meetings where I’m not on the spot and talking extensively. Best case scenario I can jot down very basic info like names and dates, but as soon as I start writing thoughts my brain switches modes and my ability to speak coherently and recall info is compromised. On my team the convention is that the person leading the meeting isn’t the one responsible for notes, so I didn’t realize this was unusual! Even when I’m the interviewer, I try to take more detailed notes, but because of the power differential it’s more acceptable for me to not speak for a few seconds at a time and let a silence happen, and tbh I largely have to flesh them out immediately afterward anyway because I still can’t write down much that’s coherent. No issue taking notes in meetings where I’m just occasionally contributing, though.

          3. Trina Wegley*

            Removed — you’re sock puppeting all over this post (using multiple user names to make it look your position has more support than it does, and claiming wildly different geographic locations — first you’re in the UK and now you’ve spent your whole career in the US?) and you can’t do that here. – Alison

            1. Chutney Jitney*

              I think you’re extrapolating that getting a job=no one cared.

              Unless you’ve asked, how would you know if they cared? Just because no one mentions it or you got the job doesn’t mean they didn’t care. Maybe it was less important than other things or maybe everyone else sucked and you were the least worst option.

              What you mean is *you* don’t care and therefore you believe no one else should either.

              1. Turquoisecow*

                I mean, if it truly mattered to the interviewer that someone take notes, they wouldn’t have hired someone who didn’t. So I think the extrapolation that they didn’t care (or didn’t care enough that it affected the hiring decision) is a fair conclusion.

          4. Australianobserver*

            you seem very fixed in your opinion on this one – it’s very interesting considering how flexible you normally are! (just an observation, not judgement)

          5. BRR*

            I’m in general not a huge note taker, but it’s not hard to imagine that there might be a possibility that I will need to write something down during an interview. So while I rarely take notes during an interview, I find it odd that so many people seem to be on the “no possibility of notes ever” side of things. It doesn’t seem like a stretch at all to say “I MIGHT need to write something down at some point.”

          6. CatHerder*

            I have interviewed hundreds of interns and dozens of full time staff members. Even interviewed up when we had openings for high level Directors and VPs as part of panels. Maybe two have brought notebooks and no one has ever used them. You are over applying your own experience/expectations and calling it a standard.

          7. Llama Identity Thief*

            “Not that they don’t end up taking notes, but that they don’t come prepared to do it in case they need/want to.”

            THERE we go. I’m in the super in-between camp of “always have my trusty legal pad on me to take notes if I need to, basically never do because the act of taking notes pulls me ALL THE WAY out of the flow of the conversation if I’m actively conversing.” It’s really more a last minute informational aid about what I researched about the position/what I need to ask, but even then I hate looking at the questions mid-interview because it’s a complete stoppage of my interview brain.

          8. Unfettered scientist*

            I’ve been on a hiring teams and I think it is typical for people to bring something to take notes. I know I do when I’m interviewing

          9. Cat Lover*

            I’ve never done this. However, now in the age of virtual interviews, I’m just typing notes during the process. But I’ve never physically written anything down during an in person interview.

            1. biobotb*

              What’s different that you only need/want to take notes now that interviews are virtual? What’s so different about the information being conveyed?

              1. PNWorker*

                Could be easier. I, for instance, can type miles faster than I can write. I would have trouble writing notes quickly and neatly and field interview questions.

          10. Critical Rolls*

            I’m fascinated by the commenters insisting that Alison, the professional management consultant with many years of experience, is wrong and overgeneralizing because *they* have personally had a different experience.

            It’s amazing to me all the people who claim that they are at advanced stages of their careers and have never ever needed to write down a single thing in an interview. But beyond that, I find the resistance to a very basic preparation — bring something to write with an on, in case you need it — unexpectedly emotionally strong in a way I don’t understand. What are the outcomes of following or not following this advice? Follow it: be prepared if you need to write something down, register as prepared if the interviewer is noting (ha!) that kind of thing. Don’t follow it: maybe nothing; rely on memory for everything that is discussed; maybe register as a little less prepared. On the balance, modest benefit to following, somewhere between no effect and slight disadvantage to not following. And yet people are arguing this like lives are at stake.

            1. Allonge*

              The no-stakes questions are often the ones that get argued about a lot.

              But even so, a couple of things for your consideration:
              – practically everyone carries something into interviews they can take notes on: their phone. But of course you cannot have your phone out during an interview because that has it’s own baggage. So bring a pen and paper, even if you never use them otherwise, and don’t expect to use them here, because… just because. This is not a winning argument and is bound to annoy people.
              – plenty of people here say that they have been interviewee and interviewer for years and in their field there was never a reason to jot something down in the interview – their memory serves them well enough, they don’t get actionable info in interviews that they cannot get otherwise etc. It’s a bit annoying to be told that nevertheless you are Unprepared TM if you don’t bring something you never needed or seen needed in decades.
              Do you always bring a painkiller to an interview? Suppose you get a headache or cramps, even if you never get them? Allergy meds? A bandaid? A map, in case your phone does not work? A bus pass, even if you are driving? An extra coat, if it gets colder? A sandwich? Why not?

              1. Critical Rolls*

                Here again is someone who is taking this very personally. None of the things you mentioned in your last paragraph are visible to an interviewer. I did not say I disbelieve the commenters. I simply find it surprising that there has never, ever been a follow up question, new piece of information, clarification, name, anything in any interview over decades that they as a candidate wouldn’t have benefitted from jotting down. You seem determined to insist that this is a fully arbitrary suggestion, despite the explanations as to its usefulness beyond appearances. We do any number of fully arbitrary things around interviews, but for some reason this one thing with actual function is a hill to die on, not just that it could go either way, but that Alison and those who agree with her are *wrong* and must be defeated.

                1. Allonge*

                  If I seemed to imply that I don’t think note-taking has a function or that there is anything wrong with it, I apologise, that really was not my intention.

                  I do find it pretty ridiculous that whether or not I take my notebook out of my bag has an impact on how people will perceive my preparedness for an interview. (Especially in my job where there will never be a meeting where I cannot take my laptop to). In general, on this board, there is a tendency I appreciate that calls out these “you are supposed to because you are supposed to” things.

                  In the end it does not matter – if someone feels strongly about this, I am best not managed by them.

                2. Susannah*

                  I totally agree that it’s best to come prepared – and I always have a notebook and pen anyway, since I’m a journalist. So.. maybe this is just my field, and the ways interviews go, but I have never, in several decades of work, needed to write down something during an interview. it’s just not the trajectory of that kind of an interview. Maybe if I were sitting down with the HR/benefits person.
                  Again, I totally believe people who are talking about interviews where they are being told a flow chart of management, what have you. But it’s never happened to me in my field – and I guess that puts me in the minority.

            2. Willow Pillow*

              Yes! “You are over applying your own experience/expectations and calling it a standard.” It’s her site, we get to read it because of all those people who do think Alison has the right to declare standards.

              Even then, this doesn’t mean that everyone has to do whatever Alison advises – there’s plenty of room for someone to have succeeded without bringing a notebook to a job interview. One can just do so without the forceful announcements, though!

            3. Spencer Hastings*

              I think people want to brag about how intelligent they are (don’t need to take notes because they can effortlessly remember every detail). I’ve known a few people who do legitimately have that ability (I am not one of them), but it is surprising that so many would happen to be reading this particular post.

            4. Master Procrastinator*

              I don’t think it needs to be personal against Alison, OP or anyone else. For me, the thing that sent me down a rabbit hole of poring over hundreds of comments (and making three of my own!) is the performativity of it all. I think it’s fine to point out that “whatever you think of x norm, it exists and is good to be aware of”. But firstly, I think it’s important to interrogate norms and traditions – e.g. “is it fair to judge an interviewee on whether they remove a notebook and pen from their bag for me to see, even if they don’t use it?”. And secondly, I think the controversy highlights that it’s clearly not a universal expectation. This is an American blog, popular around the world because Alison’s advice is great… But not everywhere is like the USA, and not everyone works in settings where widely held/assumed to be universal professional expectations and practices apply. So I guess those of us who have never encountered the idea that “not taking notes or bringing resumes/CVs in interviews looks unprofessional” want it to be known that there are other ways of looking at things.

        2. Environmental Compliance*

          A lot of stuff! I always take notes during interviews. It’s a two way street! I want notes on how they responded to questions I asked, any issues I note during facility walk arounds, if they talk about future sustainability projects, what HSE software they use (or don’t use), any regulatory issues we discuss, etc. I’m also bringing notes on public-knowledge compliance items I pull off of ECHO or anything I saw in the news. I’ll use those notes later to review my gut check + data points for evaluating if I want to work there.

          I realize these are very specific items for HSE compliance roles, but you can still write down notes quickly on anything that you thought was interesting OR problematic during the interview.

          (Shout out to the time I interviewed at a manufacturing facility and they mentioned, really briefly, a random chemical I didn’t know off the top of my head but sounded like it could be a TSCA/EHS/COI. It sure as heck was and when I got back, researched off my notes, and then asked a follow up about PSM/RMP and they were hella confused, that was a really good data point for me to have about compliance obligations and reviewing applicable regulation…. that plus some other things said in the interview pointed towards them looking for a clean up person that may or may not have any management support. I declined to go further in their process.)

        3. I edit everything*

          I’m dumbfounded by the idea of not having something with me to A) jot notes down and B) remind me of my own talking points and the questions I want to ask. I know I have a crappy memory, but surely other people forget things, too. At the very least, you might want to note an interesting topic or something that resonated with you that you’d like to bring up in your thank-you note. Having the right phrasing or just a couple words to jog your memory can be super helpful. Plus, the mere act of writing things down helps me remember.

      2. JanieGrrl*

        Totally disagree. I live in Seattle (so not European) and have interviewed hundreds of people over my career. I can count on one hand the number of people who bring stuff to write on. When they do, I think it’s weird. I’ve also never taken anything to write on to an interview. I remember the first time I interviewed someone who took out paper and wrote down a comment based on something I said. I was so surprised. It distracted me, and I kept wondering what they could possibly need to write down. It’s been at least three years since anyone I’ve interviewed brought paper to a meeting.

        Perhaps it’s dependent upon industry. I work in accounting.

        1. Nela*

          You had that reaction to someone jotting something down in an interview (something recommended in every ‘how to do well in a job interview’ primer I’ve ever seen)? People really be telling on themselves here today.

          1. JanieGrrl*

            Sure did. I’ve never seen it in any “primer” I’ve read. Nor was I told it was required or even advised at any of the numerous career coaching and interview prep classes I’ve been involved with, including the ones at the career center at the community college.

            It could also have been because she interrupted me in the middle of a question to haul her gigantic purse up onto the table, rifle through her things looking for her pad of paper, pulling stuff out and putting it back, only to start all over when she couldn’t find a pen.

            1. Miri*

              That’s a bit different than just “took out paper and wrote down a comment based on something I said” though, I would say! And to me reinforces the usefulness of having something easily on hand to take notes on (which a notebook and pen in a giant bag are not)

              1. elle *sparkle emoji**

                Realistically, how else are you meant to bring something to write with besides carrying a notebook and pen(cil), likely in a large bag? Plenty of others have said other forms of notetaking (laptop, sticky notes, phone) would be frowned upon. I get that maybe it should be taken out of the bag ahead of time but not everyone will remember that. This sounds like a standard scenario for someone taking notes in an interview.

            2. Insert Clever Name Here*

              That is so different from “the interviewee placed a small notebook and pen on the table at the start of the interview and at one point wrote down a comment” which is what Alison is suggesting and others are saying is beyond normal in their field that its disingenuous to use it as a hallmark of how weird it is to take notes at a meeting.

          2. RojjBlake*

            Yes. Most of them who seem to be clinging on to some idea that is based on @bexause you’re supposed” without pausing to consider why.

            Phones have Notes apps. I cannot remember the last time anything came up in an interview, on either side of the desk, that necessitated taking notes.

            The idea I would mark a perfectly capable candidate down for not having pen and paper is just ridiculous.

            I’m a VP of HR with 30 years’ experience multi nationally, and I’m flabbergasted at some of what I’ve read.

            1. Jane's brain*

              I’m flabbergasted that you’re flabbergasted. It’s not “because you’re supposed to.” It’s because important info is likely to be conveyed. I never would have thought I’d see a divide on this.

                1. The Ghost of Madeleine Wool*

                  I very much envy you your brain that remembers everything. I, however possess a delightfully whacky neurodiverse brain that quite often remembers things that are important, but not always, and not always accurately. It’s much, much less stressful for me to jot a few bullet points down – the act of writing also helps things stick in my memory.

                2. Myrin*

                  “And why can’t you remember something without writing it down?”
                  Because… some people just don’t have great memories? I have an excellent visual memory or regarding weird trivia no one else ever remembers but give me lots of verbal information and I’ll retain maybe half of it.

                  I don’t really have a preference either way with regards to interviewing in particular – I do bring a pen and notebook but that’s because that’s where the questions I prepare for my interviewers are, and I always like being prepared to jot stuff down even if I end up not needing to but I feel like being incredulous that someone might forget something unless they write it down is a bit unrealistic.

                3. I am Emily's failing memory*

                  And why can’t you remember something without writing it down?

                  Serious answer: Memory storage and retrieval is a complex cognitive process and it can be impeded by any number of things. Childhood adverse experiences, ranging from things as mild as not having a consistent caregiver in infancy or high levels of household stress during childhood, to more serious trauma, can produce executive brain function disorders in adults that typically include difficulty with memory. And high levels of stress that would be not uncommon for a lot of people without disorders to experience in an important interview can literally bathe the brain in chemicals that impair ability to form new memories.

                4. Pixx*

                  It’s honestly so upsetting to read comments like, “Why can’t you remember something without writing it down?” It just really illustrates how alive and well ableism is in the workforce (well, and everywhere), I think.

                  It reminds me yet again that the simplest, most innocuous things I do to help myself (like write things down in an interview, because my brain struggles with memory due to conditions entirely outside my control, i.e. ADHD and PTSD from childhood trauma) are going to be viewed with derision and judgment by some people.

                  The fact that some people will be taking really uncharitable, negative impressions of me based on something as very simple as jotting down notes, without even bothering to consider there might be really good reasons why I need to take notes as a basic aid, is so disheartening.

                5. Yorick*

                  I agree with other commenters that some people might want/need to take notes because they may not remember details. But since some people CAN remember those details without writing them down, why is it expected that you’ll take notes?

                  Honestly, in most job interviews most everything they tell me is pretty vague. I’d honestly feel silly making a big show of writing down that stuff.

                  That’s not to say I wouldn’t bring a notebook. But I might leave it in my handbag or backpack until I need it rather than making a big show of taking it out at the start of the meeting so they know I have it.

                6. BadCultureFit*

                  I’m currently interviewing (SVP/EVP level) and here’s some of what I might jot down in an interview:

                  -The team structure (since I’m usually operating for senior roles, there’s often a big team under me, and I’ll note how many folks, and what the breakdown of their specialities is — ie, 3 internal comms, 4 media relations, 2 designers, 2 social media specialists)
                  -The type of Intranet they’re using (and if applicable what version) and cadence of stories on there
                  -The name of any employee app or digital comms tools they use internally
                  -Their typical cadence of outgoing communications (2 press releases a month? 20 social media posts per day? etc.) so I can have a sense of volume
                  -The other internal stakeholders they work with and if there’s anything of note there (some companies’ comms teams work closely with marketing, HR, and IT; some have random other partnerships that I’d want to call out in my notes)
                  -Key themes that come up from the interviewer about the vibe of the team and company, and the comms challenges they might be facing

                  This is just truly off the top of my head. I’m not transcribing our conversation — I would say maybe about 8% of my time is taken up by jotting this down. I use my notes to then craft my thank you notes with specifics.

                  If it matters, I get told often how amazing my post-interview emails are (ie, thank you notes)!

                7. New Jack Karyn*

                  “And why can’t you remember something without writing it down?”

                  Wow. Just wow.

              1. Susannah*

                I really think this has got to differ according to field. When I’ve been on interviews to be a reporter, I’m asked about my favorite beats, how I develop sources, whether I’ve ever faced an ethical dilemma and how I handled it, where I see my journalism career going, whether I’d ever be interested in being an editor someday, or sticking with writing etc. I ask about how much people change beats – and if they do so dramatically (like news to sports), how they balance daily with long-term project reporting, etc. Of course I have a notebook with me. I have never needed to write anything down. The only thing they might have said at the end was the timeline for a decision – and I’d wait until I was in the lobby to make a note of it. Otherwise, in that case, I’d find it disruptive of the flow of the interview.
                Really – it sounds like people in other fields have more to write down, and I respect that. But to immediately dismiss people as clueless and unprofessional for not sitting down with notebook and pen in hand? A little judgmental.

          3. Pixx*

            Some of these comments are a bit distressing to read (especially the ones from UK hiring managers, as I’m American and live in the UK and am job searching right now), speaking as someone who always has a paper and pen on hand in any kind of meeting, including interviews.

            I have ADHD and PTSD, both of which affect my memory, and my memory gets a little frizzy when I’m stressed or nervous (like in job interviews!). It’s not that I’m incapable of remembering anything without writing it down, but boy it sure helps to just write it down and be safe rather than sorry.

            If an interviewer says something like, “Wow, that article you wrote sounds interesting; would you mind forwarding that on to Tina Smith so she can take a look?” I *NEED* to write that down, because otherwise my brain will either a) be too half-focused in the rest of the interview thinking “remember to send writing sample to Tina Smith, TINA SMITH, TINA SMITH’ or b) think I’ve got it all down fine until I leave the building and get round the corner and breathe a huge sigh of relief at getting out the door without tripping on my face, and then thinking, “Oh, [schnookums], was I supposed to send that to Tina Smith or Tammy Sawyer?! Oh god, now I can’t remember, and now it’s going to look like I can’t handle the simplest directives!”

            To think someone would judge me harshly for simply picking up a pen and jotting down, “Send writing sample to Tina Smith” is disheartening and anxiety-provoking. Frankly, it also seems kind of ableist!, but what do I know.

            1. I am Emily's failing memory*

              Fellow ADHD adult here. I truly think the people actively put off by/negatively judging note taking are an extreme minority. I think even among people who don’t take notes in interviews themselves, they still aren’t going to think, “can this person not remember their way out of a paper bag??” about someone who does take notes. Just because perfect memory is truly so rare and usually even people with perfect memory have figured out by the time they reach adulthood that they’re unusually good at remembering things compared to everyone around them. Very few of those people who have eidetic memory are walking around assuming everyone else does too.

              1. Princess Trachea-Aurelia Belaroth*

                Honestly the derisive tone of the anti-note people makes me think they are probably forgetting a lot and just don’t notice, or not asking questions about specifics. The human brain is pretty unreliable about recalling tons of details with any distance from an event, but will fill in the blanks. Read anything about the reliability of criminal witnesses.

                And I’m even on the side of the interviewees in the original question. If someone doesn’t think they need to take notes in an interview, I wouldn’t penalize them. Though, as with a good cover letter helping you stand out but not being universal, someone taking notes when they ask me detailed questions would probably impress me with how seriously they were taking it. I have literally had interviewers compliment me on my organization when I took out a notebook to take notes. (Little did they know it is a coping strategy for always forgetting at least one small but important detail.)

                1. Reality Biting*

                  Honestly the derisive tone of the anti-note people makes me think they are probably forgetting a lot and just don’t notice

                  Yes, this or in some cases I’m assuming people maybe just haven’t yet had the kinds of interviews that are high-stakes enough for it to really matter.

                  I remember early in my career—interviewing for bookstore jobs—I didn’t exactly whip out a notebook. I only started doing that once the details of the job started to matter more and I’d actually make a decision one way or the other based on those details.

              2. Orsoneko*

                Well said. As another ADHD-haver, I’m too baffled by comments like this (“Why can’t you remember something without writing it down?”) to be upset or disheartened by them. If you truly have an eidetic memory (assuming that such a thing even exists), then you’re not neurotypical. And if you don’t have an eidetic memory and exist as a functioning adult in society, then the general concept of needing to write certain things down in order to remember them shouldn’t be unfamiliar to you, even if you don’t personally resort to it often.

            2. Willow Pillow*

              Autistic + CPTSD here, I feel the same way. My need to take notes in order to augment my sieve of a short term memory has zero impact on anyone who doesn’t need notes, so why the hostility?

        2. Lilo*

          I’m a lawyer and my experience is lawyers show up to interviews pen and paper. But you’d never show up to an oral argument or hearing without pen and paper either.

        3. Lozi*

          This is so interesting! I’m American, in a more human-services centered field, and I absolutely bring something to write on/with to interviews, and would expect candidates to, as well. I don’t have a great memory, so writing things down just helps me process. I’m specifically thinking about I ask questions of the employer, I would jot down brief notes when they answer … both to remember it and to show that I value what they share with me. But it’s good to know some people might be weirded out!

          1. Ask a Manager* Post author

            Just to be clear: You can always find an interviewer who will be weirded out by some behavior — there are tons of outliers and tons of bad interviewers out there. But no decent interviewer will be weirded out by it; it is a very normal (and advisable) thing to do. (And indeed, in the example above, it sounds like what the person was really reacting to was the candidate’s disorganized behavior in other ways.)

            1. Anonymous 75*

              Well to be fair, there seems to be an awful lot of people on this thread and throughout the comments who are awfully weirded out and judgy to those who don’t take notes.

              Personally I don’t take notes in interviews but I don’t care if others do. Maybe no one should read so much into whether or not another person is taking notes and trust that that person has a handle on what they need to do for themselves.

              1. Eff Walsingham*

                Speaking for myself, what I was being “judgy” about was not whether a candidate takes notes or not, it was about them showing up empty-handed. Which, to me, looks unprepared. Admittedly, a candidate may have a notebook in their pocket or purse, and not take it out because they have an excellent memory for information received in conversation.

                I always have the notebook, and almost always pull it out at the start so as not to be distracting in the meeting. And I hadn’t consciously realized, until this topic came up today, that I do tend to wonder if the empty handed candidate has issues with overconfidence or unpreparedness. This suspicion has developed from my personal experience, of being involved in or peripheral to the interview process, then seeing which new hires flamed out. I know that my experience won’t be universal.

      3. Emmy Noether*

        I don’t think so. I’m European and I always take notes. I don’t think I’m unusual. That said, I’m the type that is always prepared to take notes anytime, and does so often. My brain remembers things much, much better if I’ve written them, ideally by hand, so sometimes my notes aren’t even to consult later, just to process better.

        1. Modesty Poncho*

          This, and I’m in the US. In school, I didn’t often refer back to my notes (and they often didn’t make sense after the fact) but the act of writing something down reinforces the idea in my brain. Even when I’m interviewing over Zoom, I like having a pad of paper to write on rather than switching screens on my laptop to get to the word processor and type. Definitely envious of those of us here who remember the interview perfectly after the fact!

          I’m also one who will write down my questions, with space to jot down the answers. That way it’s easy to see at a glance whether we’ve gone over everything I wanted to talk about when we get to the end, or whether something is still blank.

      4. rebelwithmouseyhair*

        I’m as European as they come and I’m also very surprised to read that people can turn up to an interview without being prepared to take notes. I’ll take notes of how they describe the job, especially if there’s any difference compared to what they put in the ad, I might make notes of their behaviour. I’m not trusting my memory, and I need to mull things over afterwards. Writing things down also helps you not to forget things by fixing them in your memory a different way.

      5. L-squared*

        I’m American too.

        And to me, while I received this advice when I first started job hunting, at this point it really just seems to be something to do for appearances sake. OP even says if they never take a single note, she doesn’t actually care. So its not about taking notes, its all about signifying some random thing that it may not really signify. The fact that for some people, I could literally be writing a grocery list on paper, and they’d think I was engaged, but if I do nothing but remember everything, that it looks like I’m not, shows the problem here.

        So yes, I agree that it is a common thing to do, but I disagree that anyone should be judged if they don’t.

    2. Miri*

      Notes I’ve taken in job interviews include:
      – information about the team and organisation
      – names and information about other members of the team I might be working with
      – any key dates or busy periods for the role
      – answers to my questions around what the role will be like, what challenges they see, etc
      – short notes to myself before answering a competency question (like “new team – ED – presented to PM – 15m – YOY 35m”), so I don’t forget to include a key detail
      – timeline for expecting follow-up
      – full names and job titles of interviewers if I didn’t have these already
      – any follow-up actions I took (for example, to send a copy of my professional qualifications to HR to help justify the salary I was asking for)

      Partly these are so I don’t forget to do or say something, partly it’s so that if I do get the role, I don’t have to go back over the intro again and can get myself up to speed more quickly. Or just help me remember specifics about the role after the interview. I’m in the UK public sector

    3. GammaGirl1908*

      In an interview, I write down people’s names, new information about the job or organizational structure, keywords to remember, details to discuss in my follow-up notes, and, most importantly, details about which I want to ask questions so that I have something to ask at the end of the interview.

      1. allathian*

        Yes, this. I have a horrible memory for names and I definitely won’t remember them unless I see them written down. The best way to ensure that I remember a name is to write it down by hand myself. There’s something about the action of writing, rather than typing, that helps me remember stuff, and particularly names, even if I never look at my notes again.

        Every interview I’ve been to the interview panel’s also been taking notes. The fact that I’m also doing it focuses my mind on the idea that it’s a business meeting where everyone’s trying to see if the candidate’s a good fit for the job and the job’s a good fit for the candidate. If I didn’t bring a pen and a notebook to an interview, I’d feel like I was back in college taking an oral exam and questioned by a panel of examiners who had the power to pass or fail me.

        1. rebelwithmouseyhair*

          You’re quite right, writing things down helps to remember them, it’s been proven scientifically, it’s not just a quirk of your brain.

      2. Agent Diane*

        Likewise! I’d note the names of the panel interviewing me at the very least!

        I can also write faster with pen and paper than I can type on a phone/tablet. I’ve been trying for over a decade to switch to paperless working but I simply can’t. My brain needs the physical act of handwriting. Plus I can write something legible for me without looking down whereas I have to focus to type. So I always have a notebook and pen with me.

    4. Alton Brown's Evil Twin*

      I write down particularly salient things that I wanted to include in thank-you notes or followup communication. Not to mention things I gleaned from one interviewer that I could discuss with the next person I saw that day – specially answers to my questions!

      I also write down technical information (I’m an engineer) that I want to look up to be more prepared for further interview rounds, or to be ready to take the job. Plenty of parallel things apply for other careers – local legal/accounting/building/regulatory standards & practices, particulars of competitors & suppliers, etc.

      1. JobHunter*

        Like allathian and Alton’s, I also suffer from lapses in memory during interviews or jot down things I want to look up later (usually). I brought a notebook and pens to my most recent interview but ended up following the lead of my interviewers and left the writing materials in my bag. I did take notes during the first Zoom interview though.

    5. Samwise*

      My career began in the last century as well. I’ve always taken notes, mostly when we’re at the my-questions part of the interview. I have a really good memory, but I *know* I’m not going to remember everything.

      I don’t judge anyone for not taking notes during an interview. Taking/not taking notes doesn’t seem to correlate with behavior after being hired.

      That said, I do judge new hires for not taking notes during training….if I’m the trainer, I will directly say, “You’ll want to take notes on this”. If I’m the mentor, I’ll say, “Take notes during all of your training sessions.”

    6. Butterfly Counter*

      I haven’t done an interview in a while, so sometimes I use my notes to write down multi-part questions. “Tell me about a time you struggled with X, what you did to correct it, and what, if anything, you then did to prevent it in the future.”

      When I’m in an interview, I get nervous and I talk when I’m nervous. I can sometimes answer the first part of a question and forget the second or third parts by the time I’m done. In my field, longer and detailed answers are expected, so I’m not that unusual in over-, vs. under-explaining myself.

    7. SchuylerSeestra*

      When I was interviewing it was company specific notes, like questions about the role, hirinf manager insight, comp, etc.. Sometimes I would write any questions that came to mind up:

      When interviewing at multiple places it was good to take notes to keep track of each company. I thoroughly recommend candidates taking notes.

  9. Limdood*

    LW2
    You should report it, for Bill’s sake as well as to work to stomp out this kind of behavior wherever it rears its head…

    …BUT, it sounds like your organization may be toxic and/or dysfunctional not just at the rank and file level, but probably at least ONE level up. While that isn’t likely to impact how effective taking a very valid legal discrimination complaint straight to HR is, it probably has a good chance of *cough* *cough* “accidentally” being let slip to your and your coworkers immediate managers and likely, your coworkers.

    To that end, you should thoroughly document how your work is treating you now, and be prepared to document how it changes (if it changes, HOPEFULLY this is unfounded, but prepare for the worst and be happy when it doesn’t happen, right?) so that you have a leg to stand on if you need to subsequently talk to HR…or a lawyer…about retaliation.

    You and Bill have a right to not be harassed over religion, but you ALSO have a legal right not to suffer retaliation for a report made in good faith, which this most certainly is.

    Good luck, and I hope my advice is unnecessary!

    1. I'm Just Here For The Cats!*

      “…BUT, it sounds like your organization may be toxic and/or dysfunctional not just at the rank and file level, but probably at least ONE level up.”

      Where are you getting this? I have read the letter 3 times now and I see nothing about anyone else but the COWORKERS being jerks. There is no mention of any other toxic issues or dysfunction and the OP doesn’t say anything about management. There is nothing to indicate that the OP or Bill will face backlash because they report coworkers discriminating against Bill.

      1. Chutney Jitney*

        Because it’s so prevalent that everyone has heard about it, even the target. There’s almost zero chance that a manager hasn’t heard it too.

      2. xylocopa*

        If people are going on with childish bullying like this even after being asked by a coworker to stop, SOMETHING is wrong. Someone hasn’t set a professional tone, and that’s on management.

        1. Malarkey01*

          I’m also wondering if this is an unusually young group. I’m trying to think of my office of late 20 year olds to 65 year olds and hearing that someone was a virgin might get a “hunh?” Reaction out of me but it definitely wouldn’t be something mocked. Who is or isn’t having sex just isn’t a normal joke (unless people in the office are sleeping together but then it’s not a joke just gossip).

      3. I GOTS TO KNOW!*

        The letter doesn’t explicitly say management already knows. BUT. If it isn’t being done in front of Bill but Bill still knows about it, there’s a good chance at least one person one level up knows about it and has done nothing to shut it down. If they know about it and it hasn’t stopped, they are either directly or indirectly condoning the behavior.

        We have no way of knowing, obviously—they may truly be oblivious—but I agree with Limdood that it’s prudent to prepare for the fact that some management may know and not care and get more upset by LW2 rocking the boat than clear discrimination. Hopefully that isn’t the case! But better safe and prepared than sorry.

    2. Chief Petty Officer Tabby*

      Exactly. Why are these toxic people so pressed about a guy who decided he wants to wait til he’s married? It’s not funny, and it’s not something they need to be concerned about because it doesn’t affect his job.

      1. 2 Cents*

        And it certainly doesn’t affect them in any way, shape or form. Leave the guy alone. These people sound like morons.

  10. Catgirl*

    #2’s coworkers DO suck, I’m as atheist & sex-positive as they come and what they’re doing to Bill is appalling.

    1. Fran*

      Yep, it is appalling and I feel so bad for Bill. Maybe the LW could be more firm with their responses or reach out to Bill more so he knows not everyone he works with sucks

      1. WoodswomanWrites*

        What awful behavior. What’s also appalling is that this has apparently been going on for *months* and management hasn’t yet shut it down. LW, thanks for writing about what’s going on and your concern for Bill. Reporting this to HR or comparable high-level manager is important.

        Fran, I had the same thought about reaching out to Bill if I were a peer, and that can be tricky. I wouldn’t approach Bill at all if I hadn’t spoken up for him with management already. In a past job, I did speak up on someone’s behalf to HR when they were a target, told them that I did, and they appreciated it. I knew in that instance that the conversation with my colleague would be welcome, but it depends on the person and relationship in each situation.

        Given that Bill is private about his personal life at work and the sexual nature of the gossip, he might be horrified to have a co-worker reference it to him in any context even if the intention is to be supportive. If the co-workers are good friends, that’s one thing. If not, this could come across as one more cringing instance where Bill is having to deal with his religious beliefs at work, and then it would be best to report it and not say anything to him about that.

        1. UKDancer*

          Definitely speak to HR. Bill shouldn’t have to put up with this and it’s a horrible thing to torment him about. Also if it’s not something he’s mentioned in the workplace then they must have gone looking for it, found the information and then weaponised it against him which is a really unkind action to take against a colleague.

          I think personally using information from peoples’ personal lives to make their work life a misery is contemptible.

        2. LTR FTW*

          There’s nothing I hate more than people that say sexist/racist/cruel things, and then when they get called on it, they act like the *person calling them out* is the problem. Being told to “lighten up, it’s just a joke” is infuriating.

        3. I'm Just Here For The Cats!*

          But does management even know??? Bill hasn’t said anything and if the coworkers are only doing it when bill or management is not around then they may not know. Or they may have heard something but they don’t know what is going on?

          1. Bagpuss*

            Yes, and it’s also possible that if HR aren’t very good / Management aren’t fully aware of their responsibilities that they may mistakenly think they can’t do anything unless Bill complains. Which makes them poor managers / HR but doesn’t mean that they won’t deal with it appropriately once it is ‘officially’ brought to their attention.
            So another good reason for OP to raise it and to make clear that it is religious ./ sexual harassment of Bill and *also* that it is making OP uncomfortable

        4. WoodswomanWrites*

          In case it wasn’t clear, I absolutely agree that LW should report what’s going on to HR. My comment here is in response to Fran saying one possibility is expressing support to Bill directly, and that conversation with Bill might or might not be welcomed.

          1. Fran*

            Good point that it may not be welcomed! The LW will just have to judge what to do based on their relationship with Bill

    2. GammaGirl1908*

      These people are TRASH.

      That is especially because Bill is not the one walking around the office announcing his religious affiliations and beliefs about sex. He has rightly tried to keep that out of the office, and someone is dragging it in against his wishes.

      Then they are pretending this bullying and humiliation is a joke. One of the fastest ways for me to lose respect for someone is for them to say something mean, and then go, “ha ha, just kidding, not really, but yeah,” or to accuse other people of not having a sense of humor for not laughing at the cruel thing they said. (Obviously it’s better to shut up instead of bullying your colleagues, but if you must say something mean, at least have the guts to say it and stand by it.)

      He’s allowed to be a virgin. Everyone was a virgin once. He’s not bothering anybody with it.

      By now I hope would have said something much more like, “No, I won’t lighten up, because this isn’t funny. I’m not laughing. It’s bullying and humiliating and abusive to be dragging this poor man’s religion and sex life into work. He never did a thing to you, and I don’t think this is acceptable.” Either that, or I might have gone with the annoyed/baffled look and “I really don’t get why it’s appropriate to laugh about this. I’m going to need you to explain what is so funny about embarrassing your colleague about his sex life to me.”

      1. Emmy Noether*

        Yes! I am known for my dark and occasionally cutting sense of humor, but this is clearly way, way over the line. Ribbing someone is only ok if they consent to it (joyfully, not out of conflict avoidance) and participate, which Bill clearly is not. This is just bullying.

        Also, if basic human decency didn’t already preclude it, “so-and-so is a virgin” isn’t much of a joke, come on. It’s only “funny” because it’s mean, and also because any mention of sex is funny to a certain type of immature person.

        1. Dr Sarah*

          I think it’s also a toxic masculinity thing; part of that is the expectation that men will be trying to have as much sex as possible with women and that not doing so is somehow ‘unmanly’. It sucks big time.

          One of the bits I love in the novel ‘The Gilded Ones’ is that this behaviour actually gets addressed; there’s a scene where one soldier is teasing another about being a virgin, and the man who’s basically the love interest/main male character (the main protagonist and secondary characters are female) steps in *immediately* to shut this down by saying that he’s also a virgin and doesn’t see why it’s so laughable to want to save your first time to be with someone who’s special to you. (It then turns out that the person doing the teasing is also a virgin. And there’s a really nice little conversation among the group with the message of ‘some people choose to stay virgins, some choose to have sex, why is it a problem either way?’)

          1. Mitchell Hundred*

            As an asexual man, I believe very strongly that the dominant social narrative about men and sex is tied to toxic masculinity. It’s a bigger topic than I’m capable of covering (especially in this forum), but I’ll just say that being a man who doesn’t care about sex makes one very aware of society’s thoughts on the matter.

            1. fine tipped pen aficionado*

              Yeah. I’m an asexual/aromantic cis woman and the compulsory sexuality of our culture is so hurtful in so many ways, but there is definitely The Gender of It All going on here. Solidarity, comrade. It’s tough out there.

                1. Mitchell Hundred*

                  ⠀⠀⠀⠀⠀⠀⠀⠀⠀⠀⠀⠀⠀⠀⠀⠀⠀⠀⠀⠀⣶⡀⠀⠀⠀⠀⠀⠀⠀⠀⠀⠀⠀⠀⠀⠀⠀⠀⠀⠀⠀⠀⠀⠀⠀
                  ⠀⠀⠀⠀⠀⠀⠀⠀⠀⠀⠀⠀⠀⠀⠀⠀⠀⠀⢀⡼⠁⠹⡄⠀⠀⠀⠀⠀⠀⠀⠀⠀⠀⠀⠀⠀⠀⠀⠀⠀⠀⠀⠀⠀⠀
                  ⠀⠀⠀⠀⠀⢠⡄⠀⠀⠀⠀⠀⠀⠀⠀⠀⠀⠀⣿⡁⢰⡆⣻⠀⠀⠀⠀⠀⠀⠀⠀⠀⠀⠀⣄⠀⠀⠀⠀⠀⠀⠀⠀⠀⠀
                  ⠀⠀⠀⠀⠀⡞⢳⡄⠀⠀⠀⠀⠀⠀⠀⠀⠀⠀⠛⣿⣿⣿⣥⣤⠀⠀⠀⠀⠀⠀⠀⠀⠀⣰⠟⢧⠀⠀⠀⠀⠀⠀⠀⠀⠀
                  ⠀⠀⠀⠀⣾⠡⡄⠙⢦⠀⠀⠀⠀⠀⢠⣆⠀⠀⢸⡏⠉⠋⠉⢸⠀⠀⢀⣦⠀⠀⠀⢀⢰⠃⢠⠈⣇⠀⠀⠀⠀⠀⠀⠀⠀
                  ⠀⠀⠀⠀⠻⣥⣽⣦⡼⠁⠀⠀⠀⠀⡞⠙⣆⠀⠈⢷⣀⡇⠀⣯⠀⠀⡼⠈⢷⡀⠀⠈⢻⣄⣾⣦⢞⣀⠀⠀⠀⠀⠀⠀⠀
                  ⠀⠀⠀⢠⡶⣿⣿⠿⢿⠇⠀⠀⠀⣰⠃⢠⠈⢳⡤⢾⡿⣇⠀⢸⠦⢼⡇⢰⡀⠹⣄⠀⢈⣿⠿⠿⠿⢻⡆⠀⠀⠀⠀⠀⠀
                  ⠀⠀⠀⠘⣇⠀⣀⠀⣾⡤⠔⠒⠋⠹⣦⣼⣶⣼⡃⠀⣿⠸⣆⣼⡆⢈⣳⣤⣷⣼⣋⠓⠺⣧⢠⠀⠀⢸⠁⠀⠀⠀⠀⠀⠀
                  ⠀⠀⠀⠀⣹⡆⣻⠀⣿⠀⠀⠀⢀⡶⠿⠿⠿⠛⡇⠀⠈⠛⢛⠉⠀⣼⠛⣛⠛⠋⣹⢦⡀⠹⡿⡀⠀⢸⢦⣀⠀⠀⠀⠀⠀
                  ⠀⠀⠀⡼⠋⡇⠈⢷⢿⡀⡆⠀⠘⣧⢠⠀⠀⢸⠁⠀⠀⢠⠇⢰⠀⠸⡄⢸⠀⠠⡇⠀⢳⡀⢹⡇⠀⠸⡄⠈⠳⣄⠀⠀⠀
                  ⠀⠀⣸⠁⠀⠷⣤⠤⠞⠃⡇⠀⠀⠸⣿⡄⠀⢸⠀⠀⠀⠸⡀⠘⢦⡀⢹⠈⡇⠀⡇⠀⠀⠇⠹⠦⠤⠔⠋⣇⠀⢸⡆⠀⠀
                  ⠀⠀⡇⠀⠀⠀⠱⣄⠀⠀⠙⠲⢤⠀⣿⢳⡄⠈⣇⠀⠀⠀⣹⠀⠀⠃⢸⡄⠁⠀⣹⠀⠀⠀⠀⠀⠀⣀⡤⠿⠀⣸⠃⠀⠀
                  ⠀⠀⠙⣶⠲⡄⠀⠘⢦⣀⣀⣀⣸⠁⢛⠦⠤⠞⠃⠀⠀⠘⠁⠀⠀⠀⠀⠛⠒⠒⠋⣀⣀⡤⠤⠚⠉⠁⠀⢀⡴⣯⠀⠀⠀
                  ⠀⠀⠀⢸⠀⣇⠀⠀⣠⠷⣄⠀⠀⠀⠀⠙⠢⣄⣀⣀⠤⠖⠦⠤⣀⣀⣀⣀⡠⠄⠀⠀⠀⠀⠀⠀⠀⠀⢾⠉⠨⣿⠀⠀⠀
                  ⠀⠀⠀⣸⠀⠙⢦⠼⠁⠀⢸⡀⢠⠖⠋⠉⠒⠢⣄⠀⠀⠀⠀⣀⣀⣤⠾⢦⡀⠀⠀⣰⠊⠉⠓⠦⠤⣤⣞⣥⠾⢻⠆⠀⠀
                  ⠀⠀⢸⠉⠉⠓⠦⣄⡀⠀⠈⣧⠼⠁⠀⠀⠀⠀⠈⢧⡀⠈⠉⢡⠞⠁⠀⠀⠙⠦⠞⠁⠀⢀⣠⠤⠞⠋⠁⠀⢀⡼⠀⠀⠀
                  ⠀⢠⡇⠀⠀⠀⣀⠀⠉⠑⠒⠦⠤⢄⣀⣀⡀⠀⠀⠀⠙⠲⠖⠋⠀⠀⠀⣀⣠⠤⠔⠒⠋⠁⠀⠀⠀⠀⣀⣀⡼⣿⡀⠀⠀
                  ⢀⡼⠳⢶⠞⠉⠉⠙⢦⡀⠀⠀⠀⠂⠒⠚⠿⢍⣙⣶⢶⣶⣶⣶⣖⣋⣉⣁⣠⠄⠀⠀⠀⢀⡴⠦⠤⠞⠁⠀⠀⡇⠙⢧⠀
                  ⣿⡇⠀⠈⢧⡀⠀⠀⠀⠙⢦⡤⠴⠒⠒⠲⠤⣄⡀⠀⠀⠀⢀⡽⠛⠯⣍⡉⠀⠀⠀⠀⣠⠟⠀⠀⠀⠀⠀⢀⡴⠃⠀⢸⡇
                  ⠈⢧⠀⠀⠀⠙⠢⣄⡀⠀⠀⠀⠀⠀⠀⠀⠀⠀⠉⠳⢤⡤⠞⠀⠀⠀⠈⠉⠓⠒⠒⠋⠁⠀⠀⠀⣀⡤⠞⠁⠀⠀⢀⡾⠁
                  ⠀⠈⠑⠦⣀⠀⠀⠀⠉⠓⠲⠤⢄⣀⣀⡀⠀⠀⠀⠀⠀⠀⠀⠀⠀⠀⠀⠀⠀⣀⣀⣀⡤⠴⠒⠋⠁⠀⠀⢀⣠⠖⠋⠀⠀
                  ⠀⠀⠀⠀⠀⠉⠓⠲⠤⣄⣀⣀⠀⠀⠀⠉⠉⠉⠉⠉⠉⠉⠉⠉⠉⠉⠉⠉⠉⠁⠀⠀⠀⢀⣀⣠⠤⠖⠛⠁⠀⠀⠀⠀⠀

            2. SimonTheGreyWarden*

              Solidarity, bro. My cis male partner and I are both on the ace spectrum (I’m ace, he’s demi) and he’s struggled with the same kinds of expectations.

      2. Buu*

        Re 4
        I bring notes to interviews, but I don’t think I have needed a printed resume for at least a decade. If I have anything printed at all I’ll look at it pre interview while I am waiting but then put it away.
        The more stuff you have, the more you need to move or worry about. I’m also going to suspect a lot of people’s notes now are in the cloud. So they may not even bother with pen. I’d not get my phone out in an interview and would make notes after.

        It may be age but it may also vary by
        country and industry. If you are interviewing graduates and it is the norm for you then I would consider adding it to any interview info.

      3. Tupac Coachella*

        I wondered if OP had been a little too gentle with their protest based on the “lighten up” comment, but I can’t even stand by that, because the insinuation that a comment made at work comes across as mean should be enough to shut it down, gentle or not. Even in my own head I can’t put this on OP for not being firm enough. Yep, go to HR. Your colleague is being sexually harassed, and technically so are you because you’re being unwillingly subjected to commentary about Bill’s sex life (which he never brought into the office in the first place!).

        Just to cosign: TRASH.

    3. Francie Foxglove*

      I’m wondering how they happened to find this video. Were they specifically looking to dig something up on him?

      1. Millennial*

        Eh, might have been on his public Facebook or something (if someone is giving a testimony in church they feel proud of or could be beneficial to someone, it’s not unusual to share that) – could have even turned up on the “people you may know”.

        Sometimes I Facebook stalk coworkers out of curiosity, just to see what they’re like out of the office. If I do this, it’s usually a “huh” or a “neat”, I move on with life, and usually forget about it.

        1. Clisby*

          Plus, it’s not uncommon where I live for churches to have their own FB pages – sometimes just for general social-media type stuff like “Don’t Forget Our Giant Rummage Sale This Weekend” and other times for livestreaming services or posting video of a sermon. I could see a church posting a religious testimonial.

          1. Random Bystander*

            And I could see something along the lines of Bill following his church’s page, and these awful co-workers traipsing along the breadcrumbs to find something about Bill posted on the church’s page, and then using what they found. Because it does sound like Bill is pretty private at work about his beliefs.

            (I don’t use Facebook myself, because due to history of being stalked by now-ex–this was serious enough that he ended up doing time, I am averse to anything that attaches my actual name, but my children are comfortable enough with the privacy settings to have pages, so I have a vague sense of how it all works.)

      1. Miette*

        I was thinking the same thing.

        What would concern me is that OP reports them, but Bill takes the hit if/when HR does anything about it. Would it benefit the situation for OP to warn them that what they’re doing is literally sexual harassment on top of religious discrimination and they should stop for that reason?

      2. Totally Minnie*

        Exactly. It’s the sexual harassment/religious discrimination double whammy, and OP should definitely report it.

    4. MEH Squared*

      This was my thought as well. I’m areligious and sex-positive, and I want to tell off OP#2’s coworkers. OP#2, you are a stand-up person for telling your coworkers to STFU and for sticking up for Bill. Bring it up the chain if you feel comfortable doing it because this is definitely religious harassment of Bill–and you if you are in earshot.

    5. Office Gumby*

      I don’t know why people see virginity as a thing to be mocked. Bill has chosen sex on his own terms. It’s not like he’s sworn off it completely. He knows when and how he’s going to have sex; not everyone gets that luxury.
      So what if he’s chosen this path because of religious reasons? If anything, yes, this does need to be reported as religious discrimination. Be sure to mention the word ‘illegal’.

      1. ecnaseener*

        And of course, if he *had* sworn off it completely, that would still be 100% his own damn business and not deserving of mockery.

      2. Chirpy*

        Even if he had sworn off sex completely, that is a valid choice and he shouldn’t be mocked for that either. Asexual people and people who choose to be celibate (for many reasons) do exist.

    6. Synergy*

      I’m an atheist too and I feel like this borders on sexual harassment. People really need to stop worrying about what other people do with their junk.

    7. learnedthehardway*

      The OP should raise this to HR and management and should point out that it is BOTH religious harassment AND Sexual harassment, and is explicitly defined as a “hostile work environment”, (not to mention piss-poor management to not address bullying, in general).

      How does the company feel about human rights complaints? Might give management some incentive to shut this down and discipline offending employees.

    8. nonprofiteer*

      FFS. Your coworkers sex lives are none of your business unless they are sleeping with your significant other or their boss/direct report.

      I bet they also try desperately to get their teetotalling colleagues drunk. what a bunch of a-holes.

    9. April*

      Yeah. Believe me, anyone who’s an adult virgin knows what people think and why it’s weird, and has gotten the lecture on why we’re Doing It Wrong. There’s a reason no one talks about it even beyond the religion issue – my reasons for waiting aren’t religious and I still would be happier to talk about my surgical history.

      But the people who make it an issue are being jerks.

      1. SimonTheGreyWarden*

        This. I was a virgin until 31 (not for religious reasons but because I’m ace and never saw a need for it) and while I’m a cis woman (so there was not the added pressure of toxic masculinity on top), I still would have rather had emergency dental surgery than talk about it.

  11. Anonychick*

    LW5:

    It may help if you consider that the way the resume is read is likely to be with the assumption of the pronoun “I” at the beginning of each section, like this:

    I…
    – supervised a team of five Assistant Llama Analysts. This team’s work was instrumental in increasing the maximum number of llamas groomed per day by 15%.
    – led a cross-functional project to identify, cluster, and describe visitors to Lucious Llamas Ltd. The resulting campaign delivered a 41% reduction in cost per llama groomed.
    – was responsible for compiling all data required for compliance with the International Association of Llama Groomers.

    1. Anonychick*

      Oops: apparently, GammaGirl1908 had the same thought I did, and expressed it (IMO) much better.

  12. Heidi*

    So the people LW4 is interviewing don’t even bring a phone? I think as long as you have a phone, you can write a note or email a resume quickly without carrying around a pen and paper.

    1. Lilo*

      I think taking notes on a phone isn’t a great idea because it can be seen as distracting. People also generally can write notes without extensively looking down while typing on a phone isn’t the same.

      I’d heavily recommend most people doing interviews have a basic interview binder. You keep extra copies of relevant documents in it and it has a pad of paper and pen holder.

      Worst case it gives you a positive looking fidget outlet.

      1. rebelwithmouseyhair*

        Yeah if a guy started doing something on his phone I’d be wondering whether he was texting his GF rather than thinking he was taking notes, unless he specifically said “ah I’ll make a note of that”. Very distracting. Phones are used for too much nowadays!

        1. L-squared*

          I mean, I could be writing a grocery list on my pad of paper where you think I’m taking notes.

          What is the difference. You don’t know either way what is being written/typed. Why do you assume the worst when its a phone, but the best when its paper?

          1. Lilo*

            I guess the reality is that interviewing is a bit theater anyway.

            From a personal standpoint I absolutely wouldn’t be using my phone because I’d personally find the notifications distracting.

            Interestingly my particular org hires almost entirely over video interviews now so we wouldn’t be able to tell. But if I’m coaching an intern for an in person law firm interview, I’m going to teach them to bring paper and extra copies of everything. It’s been less than ten years since I did law firm interviews and was asked for paper copies of transcripts/writing samples/resumes.

        2. metadata minion*

          Smartphones are incredibly useful tools. Why not use the best tool for the job? I don’t like to take detailed notes on my phone because I’m still a really slow typist on touchscreens, but I vastly prefer to be able to save things digitally than on paper because then it’s much harder to lose and is searchable.

          1. Willow Pillow*

            That would mean a smartphone isn’t the best took for the job, though… There are other options, I bought a Rocketbook when I went back to school a few years ago and it’s erasable and scannable, for instance.

            1. metadata minion*

              Sure, it isn’t the best tool for the job *for me*, but it is for tons of people, and yet we consider it “not paying attention”.

              1. Willow Pillow*

                In an interview, your job is to convince your would-be employer that they should hire you. I have an ADA-level disability and I use my smartphone copiously to help me function, but it is also a frequent distraction. I would say it’s a common enough source of distraction that it’s not the best tool for making a stranger feel like you’ll put their organization’s interest first.

                I’m sure there are places where that isn’t the case… but those people have no need to write AAM for advice.

        1. Lilo*

          I mean a lot of interviewing is performance. Alison’s goal here is to help people get hired and typing notes on your phone is potentially going to alienate people. It’s okay if you want to personally screen out employers that feel that way.

        2. Elspeth McGillicuddy*

          But job interviews ARE performative. Like, unavoidably. Because the interviewer can only judge you by the way you present yourself and the answers you give.

          1. Empress Matilda*

            This is a really good point. Some aspects of the interview process are performative by nature, and it’s probably better to lean into it than to fight against it.

            The problem is you don’t know your audience. Is your interviewer the kind of person who will judge you for taking notes, or for not taking notes, or for taking notes but doing it wrong? Or are they the kind of person who doesn’t care about your notes either way? There’s no way of knowing ahead of time.

            So my takeaway would be that candidates should take notes – or not – in whatever way feels best to them. It would suck to lose out on a job because of this of course, but you can’t control for everything. Maybe the interviewer has an irrational bias against people who write with blue vs black ink, or on a laptop vs a tablet, or who knows. Do what makes YOU the most comfortable, and hope for the best otherwise.

    1. Portia*

      It’s so distressing. Bill was just going about his life and work, and suddenly he’s being made miserable about something just incredibly personal. And how can he even respond without being forced to talk about his sexual choices at work?

      I’m glad you’ve stood up for Bill, LW. I hope there’s a way to put a stop to this without embarrassing the poor man further. It’s just incredibly awful.

      1. Empress Matilda*

        Seriously. Who even are these people? Are they twelve? (With no offense intended to twelve year olds, of course!)

        LW, in response to “it’s just some fun,” or “lighten up,” you could try responding with “it’s not fun for Bill” and deliver a lecture about impact vs intent. But honestly, I would report it to HR at this point. This has been going on for MONTHS, and you’ve already asked them to stop and gotten nowhere – it’s time to escalate.

        Good luck, and thanks for standing up for Bill!

    2. lucanus cervus*

      Same. And my world view is wildly different from his, but NOTHING would make me treat him like that. What the hell is wrong with these people?

    3. goddessoftransitory*

      Me too! This guy has suddenly descended into Asshole Town and he just wants to do his job.

  13. Biff Chippington*

    I would have a hard time not bringing out my obnoxious sibling training for #1.
    “Why do you ask?”
    “Just checking.”
    “But why?” (Repeated ad nauseam to any reply)

    Or:
    “Why do you ask?”
    “Just checking.”
    “Okay.” (Followed by subject change)

    I suspect you would have to be extra upbeat and guileless-seeming to get away with that at work, though.

    1. KateM*

      Someone in another forums once said that their go-to answer was “when do you need to know?”.

    2. Snow Globe*

      I think “but why are you checking” is an excellent response, and I’d really love to know how the coworker would answer that.

      1. Shirley Keeldar*

        Right! “I don’t understand; what exactly are you checking? What other information do you need?”

    3. All Het Up About It*

      I was thinking something like door two. Just answering okay and turning back to your computer screen or asking her and actual work question.

      You could also throw out “Did Boss ask you to check?” and then when the answer is no, revert to that “Okay.”

      I’d also recommend taking a minute to look at this person and see why you are assigning this particular motivation to their questions. I had a co-worker doing similar things once and I think my gut instinct was similar to yours. But then overtime, I realized I was 100% projecting her motivation and instead she was just more of a motherhen/just likes to be helpful/organized type. It was still annoying, and I still used the sort of answers without answers to respond, but the fact that I knew she wasn’t coming from a place of implied seniority/authority definitely helped. Once I understood that I could shrug off or redirect her annoying actions/questions a lot easier, because her motivation wasn’t as threatening.
      NOT saying that’s the case here. Your co-worker might be giving you lots of other clues to show she is trying to be a second boss to you, but wanted to throw it out.

      Also, I did end up asking my boss about her behavior. I framed it more as “Hey sometimes co-worker seems concerned I’m not performing task in x manner. Do I need to shift my approach?” My boss assured me they liked having two team members who handled tasks differently because they had different strengths. So I was able to relax about any fears that my co-workers comments reflected my boss’s thoughts. That might or might not be applicable to your situation. I was the new person on the team, so I wanted to make sure I wasn’t inadvertently forming bad habits. But you might just want to keep this solely as a problem you solve/ignore.

      1. Edinburgh*

        Yeah I also think a lot of commentators are assuming really negative intentions to the coworker. If a coworker asked me if our supervisor had approved my change in hours, I’d assume it was because she was thinking of doing the same and wanted to be aware of the process. Do you have regular catch ups as managers to share info and make sure you’re applying processes consistently? It seems to me like maybe she just feels like she’s operating in a bit of an information vacuum and would value more information sharing with peers.

        1. Philosophia*

          Perhaps this is a midwestern (U.S.) thing, but in my workplace a facially intrusive question would be preceded or immediately followed by an explanation of the intrusion:
          “Did our supervisor approve your change? because I asked for mine on Thursday and haven’t heard back,” for instance, or “Would you mind telling me how you got him to say yes? I haven’t had any luck yet.”

    4. Global Cat Herder*

      That’s what I was thinking. Like making people explain inappropriate jokes (“I don’t understand. Why is that funny? No really, why?”) until they realize they’re inappropriate, make her explain why she’s asking, until she realizes it’s inappropriate.

      “Why do you ask?”
      “Just checking”
      “Why?” then when she can’t explain “but why do you think you need to check?”

      Get in touch with your inner 3 year old and ask BUT WHY until she stops.

  14. Brain the Brian*

    Re #2: It’s obviously a religious discrimination situation, but is it also a hostile workplace situation? I can’t imagine wanting to know anything at all about my coworkers’ sex lives or lack thereof, and if that were a constant topic of conversation, I wonder whether it would cross the legal line.

    1. Brain the Brian*

      Well, duh. I’ve just reread the advice, and Alison does indeed note this. I swear I can read… sometimes.

    2. Lilo*

      It’s important to understand that legally (in the US) hostile work environment doesn’t just mean “workplace that is hostile” but rather that it is a pattern of behavior based on illegal discrimination. So if the coworkers were making fun of John for a non discriminatory reason, say, John really liked the Yankees and everyone else was a Mets fan (and the mocking of John didn’t go into illegal conduct), that potentially wouldn’t meet the legal definition (in the US) of hostile work environment.

      So the reason John is being harassed here (religion, sex) is relevant.

      1. Lilo*

        I should note that this is not legal advice and I am not suggesting harassing coworkers based on sports teams is a good idea or couldn’t essily fall into another illegal category of behavior.

          1. Lilo*

            Yes, however, bullying someone for a non-discriminatory reason may not meet the legal definition of “hostile work environment” as used in the US.

              1. Lilo*

                From the very specific viewpoint of “is this something someone can file an employment lawsuit over”, yes. Not all bullying is going to run afoul of employment law.

                1. *kalypso*

                  See, where I live it actually does, and there’s a dedicated process for it in addition to the general OHS implications.

              2. Stay-at-Homesteader*

                I mean, being a jerkface is generally legal, as long as you’re not venturing into specific and legally-protected territory (race, sex, religion, etc.). That doesn’t mean companies should or need to tolerate it, though. You can absolutely be fired for being a bully, you just can’t be criminally prosecuted for bullying per se.

              3. RagingADHD*

                Yes. There is no federal law against “bullying” if it is not related to a protected class. Furthermore, verbal teasing or insults at a place you are both obligated to go every day would not meet the standard of criminal harassment in a lot of areas (if anywhere).

                Obviously, there should be workplace policies that address it, and management should not tolerate it. But there are a lot of actions that good business policy should prevent, or that are firing offenses, that you couldn’t call the police over or sue someone for.

      2. Brain the Brian*

        Yep. This situation came across to me as meeting the criteria for exactly such a pattern, and having missed that Alison called it out, I wanted to ask. But of course, my reading comprehension wasn’t great originally, and it turns out that she *had* mentioned it.

      3. Yorick*

        It’s a hostile work environment for the LW because she is having to hear all these sexual comments.

        1. What’s in a name*

          I wonder if this falls into the category of sexual harassment-sounds like they’re talking about sex an awful lot and I’m sure that makes people (including the OP) uncomfortable.

          1. Warrior Princess Xena*

            I feel like at any point if the conversation veers into a person’s sex life, the person says to stop, and the conversation does not stop, the boundary for sexual harassment has been officially crossed.

            1. PlainJane*

              I think the question here is that it’s someone else’s sex life being discussed, with OP being an unwilling witness to it. I think that should count as well, but it may not fit a legal definition, which makes it harder to stop if the atmosphere is toxic.

  15. Quizix*

    I find it a really bizarre expectation that you would want candidates bringing paperwork with them. I’ve never been a note taker, made it through 3 degrees without taking notes in lectures, and wouldn’t dream of taking notes in an interview. Why would I need copies of my resume? I’d already have submitted it electronically, and it’s not like I need a refresher on what my own work history is.

    1. Fikly*

      Two main reasons to bring a resume:

      It’s sadly common for interviewers to walk into an interview and not have looked at your resume, and not have a copy with them. Having a copy with you is a good way to make sure they have it to reference.

      Second, many people tailor resumes to each application, or at a minimum, type of job they are applying to. Bringing that particular resume with you helps remind yourself what you included on it, and what you didn’t. There are often good reasons you might not want to mention things you didn’t include, or it can just be confusing to bring up something you didn’t list on the resume you included with your application.

      1. Darren*

        If the interviewers need a copy of my resume and haven’t read it or have a copy with them my next step would be thanking them for their time and leaving.

        I’m not going to work for a place that is that disorganised at best, likely horrendously overworking their staff at worst.

        1. YetAnotherAnalyst*

          There’s disorganized and then there’s “life happens”, though. I’ve been asked to step in and interview when the original interviewer had to call out sick or got pulled into an urgent meeting. HR hasn’t always gotten me a copy of the resume in time for the actual interview when that’s happened, but fortunately the candidates have had paper copies with them.

          1. Darren*

            At my work HR provides interviewer packets with everything you need to know. If you need the resume (because your interview is covering that) you’ll have the resume in there, similarly questions raised from previous rounds that would be covered in this interview, technical questions you are expected to ask (and examples of good/bad answers), paper and a pen for the interviewee if they are expected to need writing material while answering questions (they don’t get to keep the paper afterwards), etc.

            If someone is sick they give you the packet for the sick person instead. Not having such a packet is what I mean by disorganised. It could just be HR but HR being disorganised or overworked is how people end up not getting paid, or paid incorrectly. I have never been at a point in my career where I had to work with companies like that.

    2. Hlao-roo*

      made it through 3 degrees without taking notes in lectures

      Based on this phrase alone, I’m going to assume you have a much better memory than most other people. I have a fairly decent memory, and I had to take notes in all of my lectures to pass my classes and earn my degree. For me (and many others), the act of writing something down helps me remember it even when I do not look at my notes later.

      1. Lilo*

        I actually used to get paid for my notes through the ADA office. I had to type my notes and make sure everything was intelligible for a third party. They kept it completely anonymous so I didn’t know who got my notes and they didn’t know who they got them from.

      2. Princess Trachea-Aurelia Belaroth*

        I actually almost never took notes in school and graduated in the top 10% in high school and cum laude in college. And I religiously take notes in job interviews.

        In school, you often don’t need to remember specific details, and if you do, you will be practicing them ad nauseum for the next several lessons (and in classes where that’s not true, you might get an “open-notes” or open-book test).

        In job interviews, you might ask the same question in 5 different interviews, in the middle of an active conversation each time, and you will want to remember which one answered how so you can compare them later and pick. I don’t want to rely on my more holistic-type school learning style when choosing where I’m going to spend hours every day for the next few years of my life. I want data.

      3. PlainJane*

        I actually would get very distracted if I tried to take notes, and end up just doodling on the notebook and not paying attention. I had one math teacher who used to grade notebooks on how accurately we copied down her notes from the board. I used to fail the notebooks–which ended up in doodles, story fragments, and thoughts from other classes–but get in the high 90s on the tests. Taking notes is just not universally useful.

    3. Zzzzzz*

      It is not bizarre to bring a list of questions and write down answers.

      It’s more bizarre to me that thru/out this whole thread, that ppl don’t get this concept. YOU are interviewing this company as well, you often meet more than one person in the process (from HR phone screen to Zoom grp interviews), receive conflicting info from various, need to follow up on pretty much anything… what is so hard abt this concept of taking notes?

      1. Princess Trachea-Aurelia Belaroth*

        Especially, I can even see their side. I was on the side of not judging the candidates in the original question. But the tone of “What could I possibly want to take notes on?” is very bizarre to me. Like, maybe you don’t need to take notes, but you can’t even imagine someone wanting to? Either preparatory or during?

        1. Reality Biting*

          I had a similar series of reactions.

          No shade, but I’m also guessing that some folks may not be in a time of their life where certain details may be very high-stakes and detail oriented. Like, if someone can remember the specifics of PTO policies across eight different companies from interviews they had a month ago, good for them! I can’t do that, so I have to write it down.

          1. Michaela*

            But as both interviewer and interviewee, I’m only after a candidate’s approach to situations and the reasoning behind it – I don’t want details, and details are generally confidential.

            That being said, I’m very well thought of in my company, where it’s very detail oriented, and a grad recently commented on my complete lack of attention to detail in responding to a team building exercise, and how I could be in my line of work, as I got the pre-reading wrong multiple times – I told him the legit reason – I didn’t care enough to pay attention.

            1. Princess Trachea-Aurelia Belaroth*

              If it works for you, fair enough. But I also want to assert that a candidate’s approach and reasoning is in fact a detail. There are some things I’m good at maintaining a holistic “feeling” of, and some I am not. If I interviewed more than, say, four people, I would lose track of which name went with which interviewing experience unless I took a few quick notes.

              Also, what details that you learn in an interview are confidential to the point you can’t write them down for your own reference?

          2. Darren*

            The specifics of PTO policies across eight different companies are going to be in the offers/contracts, your notes you took in a random meeting aren’t necessarily going to be accurate anyway.

            Also there is a bit of luxury there I live in Australia. The mandatory is 20 days of leave a year, 10 days of paid sick leave plus all public holidays. I only need to remember PTO policies which are more than that which are few and thus not exactly difficult to remember.

            Basically there is nothing binding said during an interview (by either side), it all comes down to the written details anyway. The only thing an interview can be used for on the part of an interviewee is to assess whether or not you want the role. Which comes down to any red flags, and whether it’s interesting or not. Neither of which really need notes.

            The answer was either yes, or you wouldn’t have continued the process anyway. I literally walked out multiple interviews for that reason.

            One after the first half once it was established that the role as described didn’t have any of the interesting components that it could have had so after a couple of pointed questions I thanked them for their time so far but this wasn’t the role I was looking for and I wasn’t going to waste any more of their time and left.

            Another was a business ran out of someones house once I established I’d be expected to work in that house with the owners dogs running around I left in like 5 minutes.

            1. Princess Trachea-Aurelia Belaroth*

              I have never in my life been sent a formal offer/contract before taking a job. Different places, industries, companies, and levels are different.

              I would also like to think over the possibilities before getting to the point of job offers. I’m not good at processing things on-the-spot, either during the interview or when I receive a (verbal) offer. Different people in different types of careers approach job searching differently.

              There also might be multiple jobs of the same objective quality that I must choose between. I once applied (and later received offers) for the exact same role at two different public schools in the same district. There would have been zero differences in any kind of official offer letter, benefits, or hours, because the position was “standard” across the district. The only differences were in how the supervisor utilized the role, the location/environment, and the student body. Things that are hugely important to me, and which I took notes on.

              I’m not saying everyone needs to take notes. If not taking notes works for you, do that. I admire and envy you for your conviction and decision-making skills, and your country’s standard minimum benefits. I, on the other hand, take a few notes. I find utility in it.

  16. D.C. Paralegal*

    LW#1: These four little words will either resolve the situation or bring it to a head: “Don’t worry about it.”

    On one hand, it’s a mostly polite way of saying “It’s none of your business,” without explicitly saying so. There’s also just enough edge in it to communicate that you don’t really appreciate the question. It would work particularly well in this situation, as your coworker is apparently trying to demonstrate seniority over you, and by refusing to engage, you’re disproving her very premise. Say it enough and she’ll either get the point or become frustrated and give up asking these questions.

    But it’s also possible that she won’t appreciate this response and push you to answer her question, at which point you’ll have an opening to discuss why she feels entitled to this information in the first place.

    1. learnedthehardway*

      This is one of those situations in which I would have a quick check-in with my manager to ask if they had requested that Co-Irker supervise my work (just to give the manager the heads up that Co-Irker is up to mischief). Upon them saying “No, of course not!”, then I would have a direct conversation with Co-Irker about the situation. As in, “I notice you frequently are checking up on my work, attendance, etc. This is not appropriate. We’re peers. Would you feel comfortable with me questioning your work and attendance? Didn’t think so. So, please do me the courtesy of not questioning mine.”

      ie. return awkward to sender.

      1. Easy does it*

        Respectfully, I find this (and other similar phrasing) really aggressive. Not every situation requires the internet’s strongest boundary-setting language and it’s really off-putting when deployed indiscriminately. I wouldn’t do this to anyone with whom you need to keep a collegial relationship.

    2. Picklecommentator*

      This is good advice. I’ve told my 9th grader that the person who asks the question controls the conversation, and that intrusive questions are mostly about power, so that you should never answer a question with a statement from someone who you think is hostile — instead respond with your own question. While it could be a defense question (“Why do you ask/care?”) it may be better to deflect back onto the the questioner’s own experience int he subject (“Have you had issues with vacation requests before?”). Then you can follow up with either another question ( “oh I’m sorry to hear that, was it resolved? or statement that reflects your opinion on their experience – “I’m lucky that didn’t happen to me). Keep the spotlight on them, their experiences and disclose nothing of yours.

      Or if the question is highly intrusive, simply say “What a weird thing to ask” and move away from the conversation.

  17. JSPA*

    #1,
    follow up “just checking” with
    “why??” for a few times.

    If that’s not either discouraging or illuminating, move to,

    “why on earth, though???”

    or,

    “well that’s odd.”

    or,

    “they say that the first step in breaking a bad habit is recognizing it.”

  18. Not A Real Manager*

    I am both an interviewee and and interviewer. As a candidate myself, I’ve never taken notes and as the interviewer I’ve never seen a candidate take notes either.

    I do take notes when I’m the one conducting the interview though.

    I’m not against the candidate taking notes, I’ve just never heard this and didn’t know some assume it’s common knowledge.

    1. Reality Biting*

      If you see this… would you be willing to say what types of roles you’ve typically hired for and at what level(s)? No one in these comments is putting any context around these kinds of statements and I think the details of industry/role/level are key in this issue.

      1. NotARealManager*

        Mostly hiring for entry level roles in manufacturing and office work. But most candidates do have prior work experience or higher education (if that makes a difference to your calculation).

        For me personally, I have a graduate degree and am mid-level moving towards senior-level.

        1. Reality Biting*

          Thanks! I do think these details make a difference.

          So I’ll just speak for myself here. When I was interviewing for entry-level or early-stage jobs in the service sector, I did not take notes during interviews. I don’t think that would have served much purpose. That’s because at that stage of life, I mostly just took the jobs I could get, and a lot of how you do things in early-stage jobs is essentially however they train you to do them. So I chose jobs much in the way I think most people did: based on extrinsic factors (e.g., which one is closer to home) or fairly basic factors such as whatever pays the most or looks like an entry point into a desired career.

          I am now at a fairly senior level in my career in educational publishing. Choosing a job is likely to be based on a much more complex calculation of factors: exactly how the retirement benefits package works, PTO policies, specific procedures for how they accomplish tasks, how many direct reports are expected, projected major projects, advancement opportunities, and so forth. If you are interviewing with multiple companies—each of which requires 2 or 3 or 4 interviews each over the course of several weeks—these are not details you can easily keep in your head. People take notes, I think, because they are going through a rather complex process.

          Having said that, I acknowledge that perhaps some people really can keep track of all those things in their heads. Good for them! But it doesn’t really surprise me that not everyone can. I certainly cannot.

          So all in all, I think the type of job can make a big difference as to what is considered normal.

  19. Feral campsite raccoon*

    Here’s poor Bill, doing what a lot of us wish more Christians would do and keeping his religion to himself, and he gets made fun of by your coworkers. Indeed, they suck.

    1. Stay-at-Homesteader*

      That is an excellent summation of the extra-sucky nature of it. He sounds like he goes out of his way to not discuss these things! (Which, as a non-Christian, I actually don’t think he needs to do to the extent it sounds like he is! There’s nothing wrong with being open about being a believer in a casual and non-proselytizing way. It makes me wonder what other messages the coworkers or office might be sending about religion…)

      1. Not Tom, Just Petty*

        It’s all in fun…
        Yeah, whose?
        OP should say, “you’re right. Bill would enjoy this. Let me go get him. He loves a good joke.”

    2. goddessoftransitory*

      Right? And apparently the original shitty coworker went looking for this video? Like, I assume Bill didn’t have it playing on a loop in his office.

      To make fun of someone’s choices about their sexuality is just so incredibly gross.

  20. Millennial*

    In Grade 10 we were told to bring a copy of our resumes to interviews. My first *real* interview was when I was in my early 20s, so I did just that after emailing it to them with the application.

    Never got asked. Sat in on interviews at my current company; never came up.

    Is this actually still a thing?

    1. Anonymous Poster*

      I sometimes have used copies of my resume in the interview. It’s been when the person conducting the interview had the only copy of the resume, and other folks were in the room and didn’t have copies of it for whatever reason. It made me look prepared, and didn’t burn interview time with someone running out to make copies.

      It’s definitely a, “can’t hurt” kind of thing.

      I have a folio because I write down questions I really want the answers to and take notes on the answers, like stuff on time off or my impressions on the overall office culture. Also helpful if I have multiple interviews so I can better rack and stack what I find out.

      I dunno, I think taking notes in an interview is really helpful later on when making a decision! But I’m a heavy analyzer.

    2. Donner*

      I still bring resumes to in person interviews. Here are situations where it’s been useful:
      someone is grabbed last minute, someone just never got around to reading my resume, someone read my resume but didn’t bring their copy with them.

      1. All Het Up About It*

        All of the above and gave me a last minute reminder of what I sent this particular team.

        The varied and emphatic feelings on this topic are fascinating. I’m very middle ground in that I always bring my resume in the folio, (and sometimes my cover letter and copy of the job description) and a pen. Sometimes I’ve also jotted down specific questions I want to ask or items I want to highlight. Rarely to I actually add to or take notes, but yes, it helps me feel prepared. I don’t know if I ever worried about how it comes across.

        I also don’t think I would particularly notice if someone showed up withOUT these things though. Honestly when I think back on past in-person interviews, I can’t remember what candidates did…

    3. Reality Biting*

      I was told in driver’s ed always to wear a seatbelt. I’ve been driving for 10 years and I’ve never been in an accident. Is wearing a seatbelt even necessary anymore?

      You don’t bring a resume because it always comes up; you bring it because it could come up, and if it ever does, you’d rather have it than not.

      1. Emmy Noether*

        Yes, and I don’t really see a downside in bringing a copy just in case (one can leave it in one’s bag or at the back of the notepad or folio and get it out only if needed). Although it’s true that when one doesn’t have a printer at home, that can put it on the side of too much work for the outside chance of needing it.

  21. Fintech*

    Early in my career, I would always show up to interviews with a pad of paper and a pen. This was sometimes useful for sketching diagrams or quick notes on a scenarios, but most of the time I found it had been totally unnecessary. I’m in a technical role, and typically the complex technical questions would be handled as a whiteboard exercise, and the behavioural questions were simple enough scenarios that note taking wouldn’t be needed.

    I could imagine that being very different in other roles – for example, perhaps in HR interviews candidates get given a more complex histories of theoretical employees and asked how they’d proceed in that situation… I’ve never done an interview for an HR role though, so I’m just speculating.

    I also noticed that when I interviewed over soon during the pandemic, I took more notes that I normally would have… which is to say “some” rather than “none”

    On the reverse side, when interviewing candidates, I always make sure there’s a few sheets of paper and a pen available to them, in case they want to diagram a solution or make their own notes.

    I once interviewed a candidate that had brought something like 30 pages of notes with them. Every question we asked was followed by “let me check my notes”, and a frantic riffle through the pages to find something relevant. I don’t think the notes were prepared answers, more like details of all the tasks they’d done in previous roles, and how they’d been done. It was really off-putting though. I was left with the impression that this was a hard worker, who prepared well, but seriously struggled to think on their feet or recall stuff they’d done previously.

    1. Agent Diane*

      I had someone bring a lever arch file of “evidence” to an interview once. It did not go well.

      1. House On The Rock*

        Years ago I interviewed a woman who insisted on showing the interview panel her giant binder full of “projects” – aka made up business cases/analyses that didn’t relate to our work or industry.

        I thought it was a giant red flag, especially since she totally derailed the planned interview format, but fa higher up was impressed with her gumption and offered her a job (that he didn’t manage!).

        It will surprise no one that she 1) required way more training and support than she should have and 2) was a nightmare for all who worked with her.

    2. Weaponized Pumpkin*

      I had an interview a few years back where I went way too far on notes — it was only about 4 pages, not 30, but still far to much to rifle through on the fly. I spent time looking for my questions on those pages, when it should have been short, scannable crib notes on a page or two tops. I felt awkward and scattered, and I’m sure I looked that way too.

  22. Cookies For Breakfast*

    #4 – interesting, makes me wonder if I’ve been messing up something that always felt normal.

    I always have pen and paper with me, but never thought of having them out during the interview itself. My concern is slowing down the conversation by stopping often to write, when in fact I want to appear engaged in listening and answering as the questions flow. I have very good memory, so taking time right after to write things down still means things don’t get missed. Which I realise I can’t expect interviewers to know.

    Now, my most recent interview experience was online during the pandemic, which was different. Easier to take notes on screen while talking, so I did it. But I did wonder whether others would get the wrong impression (unengaged / reading from a script) if they spotted I was typing or not looking straight at the camera. At my old job, I was often told that the sound of me typing gave the impression I was multitasking during meetings, but typing is in fact my most effective way to capture details, so that got me feeling self-conscious about something completely normal like note-taking.

    1. Princess Trachea-Aurelia Belaroth*

      I usually only take notes when it’s my turn to ask questions, or if the interviewer pre-emptively answers something I was going to ask, or says something very unexpected about the job.

      I also bring a copy of the job ad, which I sometimes take notes on as things are clarified.

  23. KR*

    I know that when I’m in interview mode, I’m focusing all of my energy on just appearing to be a competent qualified not-awkward and definitely-not-nervous human. I couldn’t possibly jot down useful notes and also appear to be engaged in the conversation. My preferred method is to do the interview and then write down everything that I can remember as soon as I’m out of it. I don’t think not having a note pad or anything is unprofessional, per say, I think it’s just as likely to be your candidate trying to be as present and focused as possible on the interview.

    1. SageMercurius*

      Yup, this is where I fall on it. Especially if I have to do a presentation as well, so need to remember to bring the correct USB… More power to them that do take notes, but I’m not one of them.

    2. YetAnotherAnalyst*

      For what it’s worth, trying to appear competent and non-awkward is why I bring a notebook and pen! I don’t think I’ve ever actually had anything I *needed* to note down, but having a pen gave me something to do with my hands, and taking a second to write something down was reasonable cover for not speaking while my brain was catching up. I also like to go into interviews with several questions written down ahead of time. They always ask if I have any questions, and my mind goes blank every time. If I have a notebook with a list, I can at least take a second to scan my list and cheerily reply “No, I think we’ve covered everything! What are the next steps in your hiring process?” or something similar.

      The notebook is a prop and a prompt for Interview!Me to be much more together than Unscripted!Me could ever be.

      1. UKDancer*

        Definitely. This is why I have one. It helps me feel more organised and ensures I make structured points and don’t forget anything and gives me something to do with my hands. Mostly I hardly use it but I feel better for having it.

        I also have the questions I’d like to ask written down.

      2. Lilo*

        I’m fidgety and having a pen in my hand provides a spcially acceptable outlet for the fidgeting.

  24. Bizhiki*

    Hold on… not only am I allowed to bring something for note taking to an interview, it’s actually encouraged?? I’ve just been white knuckling it through my interviews for the past 25 years, thinking that taking notes was somehow unprofessional or distracting.

    I’ve even been an interviewer on many occasions and had this view reinforced by seeing how rarely others were taking notes. It wasn’t until the pandemic that I realized how freeing it was to be able to discretely take notes during interviews. This is an absolute game changer for me, I’m so excited!

    1. Melissa*

      Haha yes! I love my little notebook and my cute pen, so I take any excuse to use it, and that includes job interviews. Also occasionally jotting down a note can relieve some anxiety for you (I mean, don’t stare at your notebook the whole time or anything, but having a pen to hold in your hand and a place to write can be soothing).

    2. Milfred*

      I usually bring a folding portfolio that contains a pad of paper, a pen, and a few extra copies of my resume.

      Most of the time I never open it.

      It does come in handy if they give me a document to take with men (which has happened a few times). Occasionally I’ll make a note as needed, but for the most part I’m talking or listening.

    3. Princess Trachea-Aurelia Belaroth*

      I’ve been specifically complimented by interviewers for having a notebook, pen, and resume copies. Also for asking good questions, which I only remembered to do by writing them down ahead of time and checking my list when it was my turn to ask.

      I think part of the disconnect in this comment section is that it’s kind of like having a good cover letter. Most people don’t bother, so they don’t even know it can be useful. (Not to say everyone has to–if you are satisfied with your interview experience and takeaway without taking notes, and you get jobs with a generic cover letter, more power to you.)

  25. Zarniwoop*

    “ It feels aggressive to answer such a simple question with…”

    What she’s doing is aggressive, sneakily so.

    1. Peanut Hamper*

      Yep. After three or four times of “Why do you ask?” I would start responding with “I’ve got it covered. You can stop asking.”

    2. Not Tom, Just Petty*

      It IS aggressive. OP has responded, yet co-irker continues.
      Trust your instincts, OP. You are under friendly (frenemy?) fire.
      You’re flight or fight instincts are kicking up. You are feeling like it’s an overreaction, but
      She is crossing your boundaries.
      And you are responding.
      And yet she persists.
      Listen to your gut. Throw that awkward back at her.

  26. UKgreen*

    British, mid-40s.

    I’ve never taken a notepad and pen out in an interview as an interviewee. I have never taken a paper copy of my CV to the interview, and would see it as a huge red-flag if the interviewer didn’t have a copy in some format.

      1. oirishgal*

        Irish..late 40s .. working across Ireland and UK and have always brought paper copy of CV, notepad and pen with me…and used them frequently. Have also used in Zoom interviews.

    1. londonedit*

      British, early 40s, and I always bring a notepad and pen to an interview. I also usually bring a couple of copies of my CV, though I don’t currently have a working printer, so I might not do that if, say, I had an interview tomorrow. I see it as being and looking prepared – I also write notes for myself before the interview, just little examples and pointers for things I could mention in response to the usual questions, or lists of my achievements etc, and I like to read through those on my way to the interview so that they’re fresh in my mind. I probably wouldn’t actually look at or write in my notepad at all during the interview itself, but having it there on my lap makes me feel more prepared, and it’s there just in case I do need it.

      I’d expect my interviewer to have a copy of my CV, but I don’t think it would be a huge red flag if they didn’t.

      1. UKDancer*

        Also British and 40ish. I bring a notepad and pen to an interview because I feel more comfortable with one. I also tend to have a few key points to cover jotted down so I can make sure I tick them off and get them in. I don’t usually write anything down (except the names of the interviewers so I don’t forget them) but I feel better having the pad to hand.

        I don’t tend to bring my CV / application with me because I assume they’ve got it electronically.

    2. Cat Tree*

      But don’t you come with a list of questions to ask the interviewer? Do you just have a super memory, or do you not care if you forget to ask about something important?

      1. JobHunter*

        I sometimes have two or three specific questions prepared, just in case none organically arise from the conversation. I have emailed a question to an interviewer afterwards, too. It depends on how I gel with the interviewers. Mostly I use the same stock questions:

        “What metrics are used to measure success in this role?”
        “Which of my KSAs interest you most? Is there a specific problem I can help you solve?” (I have a technical background.)
        “Can you tell me how you plan the target achievements for this role? Is it a collaborative process with quarterly revisits, for example?”
        “What training platforms does the company have in place for continuous improvement of employees?”
        “Where in the company could I go from here?”

        I have never been bold enough to bust out the ‘magic question’ but I can usually get the interviewer to chat with me a bit. I then get a feel for how coffee break conversations will go. If we can’t share a mutual enthusiasm for culinary experiments gone sideways then it’s a no-go.

        1. Princess Trachea-Aurelia Belaroth*

          Your first question pretty much gets to the main importance of the “magic question.” The best part of the magic question is that it kind of breaks everyone out of the usual mode and makes them look at the role in a new way, but essentially the actual question is “what are the most important skills (or soft skills, or attitudes) for the person in this role.”

  27. Roland*

    OP1 another option – what if you just ignored those questions? Alison’s advice is good, but if you get that far and she keeps doing it… What if you just didn’t respond to those oversteps? Not a rhetorical question, maybe it wouldn’t help either and she’d ignore the ignoring, but maybe she’ll stop if she doesn’t get anything back ever.

    1. yala*

      I feel like that wouldn’t really be as helpful as just using Alison’s responses and if that doesn’t work, straight up using your words. To me, ignoring people who are asking you a question is *very* hostile behavior (low-key bullying and has happened to me more than once), and while yes, OP’s coworker is being nosy, I think ignoring direct questions would likely just escalate the situation AND would seem childish. We’re grown-ups. We use our words.

      1. Lana Kane*

        I wouldn’t want to use my words in email, honestly. Too easy to misunderstand tone. I’d be ignoring the emails as they seem to be either coming out of nosiness, or a power play (as the LW suspects). Let the Coworker ask me in person if my boss approved my time off – then I’d be happy to use my words. (And no, not aggressively, I mean that in this scenario I would feel more comfortable talking it out with them.)

      2. Roland*

        It’s not bullying to ignore someone asking you questions that are none of their business after you’ve already had a conversation about it. “We’re grown-ups. We use our words.” We’re grown ups, we don’t have to indulge people when they’re regularly overstepping.

        1. yala*

          I’m not saying that it would be bullying in this instance, but it is a common bullying tactic, and it could unnecessarily escalate the situation and would also just be passive-aggressive (in a face to face situation, not so much in an email). Telling someone out loud that it’s none of their business isn’t indulging them.

    2. The OG Sleepless*

      That’s what I do. I have a coworker who asks that kind of thing. I don’t think she’s trying to manage me, she’s just the kind of person who had to put in her two cents about *everything* whether it’s her place or not. I just ignore her. That makes her have to decide whether to drop it or ask again, and even she can discern that if she’s having to dig that much maybe she should just drop it.

    3. One HR Opinion*

      I like this advice when it is an email. You have no obligation to answer an email just because it was sent. If she follows up, maybe something like, “oh, I assumed it was a rhetorical question”

      1. Not Tom, Just Petty*

        I was thinking that, too.
        “If you have questions about my schedule, just send me an email.”
        Which I will ignore (if you even send one), but yeah, I am not going to follow up, because this little power game can’t have a record.

  28. Helvetica*

    LW#3 – this does depend on the company/culture/circumstance, so Alison is right to be wary but I’ve successfully done what you asked.
    In my case, at the time when I applied for the first job, there was a possibility the second job would become available, and they even asked me what I wasn’t applying to do that (to which the answer was, of course, that there was no such post to apply for :) ). And a month before I was due to start in 1 st job (in Europe, so different timelines apply), they opened the next competition, which I actually wanted and was qualified for.
    So, I wrote to the HR manager who hired me and interviewed me, as well as to my at then presumed future boss, to let them know I would like to apply since this chance was truly once in like 5 years, and also referred back to my interview. They were okay with it, I applied, and due to said timelines, I started 1st job on 1st of the month, had an interview for the second one on 15th of same month, and then switched about two months later.
    Key is to explain your reasoning, be very understanding that this is hardship for them, and be ready for them to tell you “absolutely not”. If it’s an organisation where everyone knows it is hard to get an offer – like mine also is -, I would say they may be more open to you trying this.

    1. justcuriousiguess*

      I feel like a better strategy, if you feel like you have a decent relationship with your boss early on, is to bring the job posting to them and say : I realize I just got here but this is the type of role I’d like to grow into over the next year or so. I’d appreciate any help you can give me in preparing for this type of position.

      1. Jamjari*

        This was my thought as well. You’re signalling interest without signalling job hopping.

      2. Venus*

        This was my thought too. It works so well! It doesn’t mean that they will be supportive and encourage you to apply every time, but it opens up the option in a way that allows for a discussion while not hurting the relationship.

        “This role really appeals to me, and if there is any specific education or experience that I can do in the next couple years to make myself a better candidate to Company then please keep me in mind.”

        1. Venus*

          Reading it again, I feel like this should be worded more strongly.

          “This role really appeals to me, so could we please discuss what education and experience would make me a better candidate to company.” Maybe even add a line or two about current qualifications if that makes sense.

          This discussion should happen as soon as possible, in case the boss wants to recommend you for the position. It’s unlikely, but if the position is hard to fill or LW3 is already qualified then it can’t hurt to talk soonest.

  29. Harmonie*

    In regards to letter #2, I am a 33-year-old virgin. Not due to religious reasons but due to the fact that I haven’t met someone that impressed me enough yet. Also, I was prioritizing school and friends a whole lot more, which I don’t regret.

    I think there are a lot more 30-somethings and up that are virgins than people think. And that’s ok. I hope someone tells Bill that.

    Your coworkers are doing not only religious harassment but also sexual harassment and I hope they step on Legos for the rest of their lives.

    1. allathian*

      Yes, this. There’s a lot of unnecessary shame attached to when, or whether, people become sexually active.

      1. I'm Just Here For The Cats!*

        I think it’s worse for men than it is for women. I think if Bill had been a woman and did the video the coworkers wouldn’t think anything of it (or at least as much) because there is such a stereotype that men have to have X many partners.

    2. Chrissssss*

      As a society, we tend to give more importance to the concept of virginity than it really has. No one does such a fuss about skiing ore scuba diving.

      It’s a shame that OP#2’s coworkers don’t see how ridiculous their behavious is.

      1. Pucci*

        As a society we view first intercourse as the loss of virginity, when in reality losing virginity is a gradual process. Holding hands, kissing, sex are all “losses of virginity”, but none are a “Do it once and are now an expert” sort of thing.

        1. I should really pick a name*

          At the end of the day, virginity isn’t a thing.
          There is nothing “lost” by having participated in sexual activity.

          1. I Wrote This in the Bathroom*

            +1000 these folks are harassing Bill for something that 1) does not physically exist that 2) they would never be able to confirm or deny even if it did.

            As a woman who has, like the rest of us, experienced side-eye from casual acquaintances for literally everything I did in my personal life (married too late/no wait, married too early/had kids too late/no, early/too many kids/not enough kids/left husband/too many boyfriends/too few boyfriends/too single/not single enough and the list goes on and on and on…) I am ready to rain fire and brimstone on Bill’s colleagues. Why the hell is his personal life even any of their business? What about their own? also up for scrutiny? because I’d be happy to have a go at it and find something wrong with each of theirs. Forget the legos, can they step on live pirahnas for the rest of their lives?

    3. GammaGirl1908*

      ALLLLLLLL THE LEGOS.

      Agree and I support you, Harmonie. I am as sex-positive as they come, and… having had PÍV sex versus not is A) not an accomplishment or trophy, B) nobody’s business, C) A decision you are more than allowed to make as an individual and/or as a couple, D) REALLY not Bill’s coworkers’ business, as this office is not a frat house, E) REALLY not weird or shameful or sad, and F) really not for discussion at work, as it involves not just you, but also your partner.

      I am so disgusted by these people.

      1. UKDancer*

        Yes agreed. Also I prefer not to think about my co-workers love lives. I mean I accept they probably do have sex but I try not to think about it because it’s none of my business and really weirdly intrusive. I can’t understand why Bill’s co-workers are so wrapped up in this when it’s none of their business and shows a disturbing preoccupation with Bill’s private parts that is weird in an office setting.

        1. Grizabella the Glamour Cat*

          This reminds me of a scene from Murphy Briwn (God, I’m dating myself!) where Corky said she liked to think of the men she worked with as Ken dolls! X-D

          That’s pretty much the page I’m on. I don’t want to think about what ANYONE I work with has in their pants, much less what they might or might not be doing with said pants contents.

    4. Lurker*

      Extremely agree. I suspect this should be sexual harassment in addition to religious harassment.

      I’m gray-asexual. Not having sex is as valid a choice as having sex, regardless of your reasons!

      It’s extremely gross to mock people for their sex lives, and even grosser to do it at work.

    5. yala*

      Same, only 37 over here. It’s partially religious, but primarily, as you said–haven’t really met anyone that impressed me enough yet. (Also the whole ace thing, y’know)

      I would be furious if I heard coworkers mocking someone over virginity. Granted, I would also be furious if I heard them mocking someone for having lots of sex. It’s wildly inappropriate to talk about a coworker’s sex life, and even moreso when it’s tied to their religion. wtf.

    6. fine tipped pen aficionado*

      Literally same! I’m asexual/aromantic and letter #2 hit me where I live. How much sex you do or don’t have: 1) says nothing about you as a person despite what literally everyone who has ever made a joke in America seems to think and 2) is absolutely not a workplace appropriate discussion.

    7. Mischa*

      Thank you for sharing this. My childhood and teenage years were spent in the evangelical Christian church during the height of purity culture; I’m just starting to unravel the intense shame and anxiety I still feel around my lack of sexual experience (let alone dating experience). I’m 30, almost 31. Your post really hit me in a really good way, Harmonie. Might have to go cry some tears of relief.

      I would be absolutely devastated if my coworkers talked about my sexual history like this. It’s mean, it’s nonconsensual, and it doesn’t belong anywhere, let alone the workplace. Bill has made the best choice for him. He’s not forcing his views on his coworkers; he’s made a choice to live in a way that aligns with his values and beliefs. That choice deserves to be respected. I hope these coworkers step on legos and four-sided dice for the every day.

    8. lucanus cervus*

      For real. We do not shame people for sexual choices that harm no one. It does not matter what Bill does or does not do in bed. No one at his workplace should care.

    9. goddessoftransitory*

      Anyone’s sexual activity status seems like such a NOT ANYBODY’S BUSINESS area that this letter astounded me way more than it should have.

  30. oirishgal*

    OP 5
    You’re daughter is correct. As interviewer it’s preferable (visually and cognitively) for me and my peers to scan a list than read a descriptive essay. I’m looking for your skills not your ability to write complete sentences.

  31. It’sAlwaysSunny83*

    LW5: UK based. I have been doing my CV your daughters way for over ten years now. I was taught this was the correct way to do it now.

    1. londonedit*

      Also UK-based and as far as I can remember my CV has always been formatted in the way the OP’s daughter suggests, so that’s 20 years now.

    2. Enni*

      LW5 – I’m also from the UK, and used to oversee CVs at my last job. Your daughter is 100% correct here and I would advise you to listen to her! I hear you that it feels weird and incomplete to you, but it’s an industry standard and it will make your CV stand out in a bad way to do it differently. No, you probably won’t lose out on a job over it if your CV is strong, but why risk it?

  32. coffee*

    Re LW #5 – the daughter’s advice is the common format for resumes in Australia, at least in my experience.

  33. Akili*

    LW 4: I have ADHD and have never been able to take notes. It’s actually causing me issues at work (in my own head, my work itself is fine) because I never learned how to take notes. I actually can’t actively listen AND write at the same time (auditory processing issues).

    That being said – I do still usually bring something to an interview, but since I’ve mostly been interviewing in the public sector in Canada recently, I’m bringing my answers to the questions we got in advance, and most of the time a notepad and pen is actually provided by the hiring panel.

    I just wanted to point out other possibilities – they know they can’t take notes because of something specific (see above) or expect it to be provided so they don’t think to bring anything. Like I said, I still bring a notepad knowing I can’t use it, but I’m an elder Canadian millennial desperate to look “adult” so I think I picked it up somewhere lol

  34. Teapot Wrangler*

    LW5 – I’m in the UK and, as far as I know the standard convention is bullet points and drop the ‘I’s – sorry!

    1. Keymaster of Gozer*

      Yup, both my CV and all the ones I review are bullet points and written in the impersonal ‘wrote training guidelines’ style.

      I wonder if it’s an offshoot of our self deprecating humour which discourages saying anything positive about yourself directly.

    2. An American(ish) Werewolf in London(ish) aka Pookie Sparklepants*

      The jury has definitely ruled against me! I do have bullet points…but use complete sentences within those bullet points. I fear, however, a revision session is ahead of me.

      Thank you!

  35. Testerbert*

    LW4: Personally, I’ve never felt the need to take notes in an interview (but have brought copies of my CV just in case); if I’m in a face-to-face, I’ll have seen the job description and had at least one screening phone call (where I’ll have had an opportunity to ask any questions not answered by the job description). By this point, I’m going in to answer any questions they have about my prior experience, actually see where I’d be working, and generally have a nice chat. If something is of such importance it *must* be noted down in the interview, I should either have asked about it prior, or it should’ve been part of the advert/job description.
    This is from a UK, IT-adjacent perspective, so it won’t apply to everyone.

  36. Richard Hershberger*

    LW5: Many years ago, I had the same notion. My resume pushed hard on my writing skills. I thought that one way to demonstrate command of the English language was to write in complete sentences. I was advised strongly against this. The argument at the time was that no one would take the time to read my resume if it had those extra two words over what would be in a bullet point. I eventually came to understand the issue rather as one of genre. A complete command of the language includes command of the conventions of different genres. The genre of the resume calls for bullet points and sentence fragments, so there you go.

    The same person talked me into changing the word “effect,” used as a verb, on the grounds of the common wisdom that “effect” is a noun and “affect” a verb. I pointed out that there are in fact standard uses of “effect” as a verb and “affect” as a noun, and that I was using the word correctly. My advisor pointed out that I can’t count on the reader knowing this, and I might be rejected on this basis. This too was wisdom. The genre of the resume includes a strong lowest common denominator element.

    1. Melissa*

      Absolutely agree, especially with using words like “effect.” A resume is not the place to use unusual verbiage. Keep it as simple as possible, and make sure sentences will be understood easily by any reader.

    2. Emmy Noether*

      Mmmh, I definitely see the difference between “effecting change” and “affecting change”, and I’m not even a native English speaker (to be fair, it being a foreign language to me may make me more, not less, attuned to spelling, (near) homophones, etymology, and whatnot).

      That’s really just an argument for writing in “simple English”. I would avoid those words entirely when in doubt, definitely not purposely use the wrong one!

      1. Richard Hershberger*

        I rewrote to take out the word. At this point I don’t recall what I changed it to.

      2. gyrfalcon17*

        I have seen both effect and affect used incorrectly many times, where the other one was needed. I take a perverse delight in reading these sentences with the meaning of the actual word used, rather than the intended meaning.

        You can’t count on people to know the difference between and usage of these.

    3. Zee*

      I pointed out that there are in fact standard uses of “effect” as a verb and “affect” as a noun, and that I was using the word correctly. My advisor pointed out that I can’t count on the reader knowing this, and I might be rejected on this basis.

      Fun fact, I once got a B on an English paper in high school where the teacher marked my usage of “effect” as a verb was incorrect, when it was in fact correct. There was nothing else on the paper marked wrong. I took away the same lesson as you – avoid less-common usages of words.

      1. New Jack Karyn*

        I would have gone pistols-at-dawn on that English teacher. Possibly your B was due to their envy at your writing skills.

  37. Turingtested*

    OP 1 I’m on the non confrontational side and I would put everything back on her. “Did boss approve your schedule change?” “Oh, do you need help requesting a schedule change? I can explain our procedure.” Just pretend that she’s asking for herself and give no personal information.

    1. Stay-at-Homesteader*

      Oh I like this. Slightly petty but with enough cover that it’s completely defensible as just being helpful (not that you need cover to return the awkward to the coworker, but if you want it).

    2. Captain dddd-cccc-ddWdd*

      I almost wondered if she is “asking for herself” in the sense that, for example, her request for a schedule change had been refused but then OP says they are coming in an hour later or whatever from now on prompting her to ask “did Boss approve that? [because they didn’t approve my own, similar, request]”. If she isn’t generally known as being a busybody I think that is a potential explanation.

    3. Bagpuss*

      I like this response a lot. It assumes good / neutral intent on her part rather than that she is a busybody, so it’s much harder for her to (plausibly) accuse OP of being aggressive or obstructive, it turns the tables if she is in fact trying to promote herself over OP, and if we are charitable and assume that she is actually trying to figure out how OP has managed something that she wants but can’t do, it lets her get that information!

      And if she is trying to promote herself, it may work to get her to stop doing it because if she wants to think of herself as senior / in a supervisory role, then having it interpreted as if she were a junior person asking for help is likely to be off-putting and may result in her asking less frequently. And f she challenged it it opens the door for OPs response to be on the lines of ‘Well, I knew that you couldn’t be asking just to check on me, because of course neither of us has that kind of authority or supervisory responsibility over the other, so the only explanation I could think of was that you were unsure and needed help, and I’m happy to provide that. ‘

  38. Irish Teacher*

    Honestly, reading the the first letter, I think the “just checking,” if that is a direct quote, would bother me more than the questions themselves, because to me, that sounds like they are saying they are just making sure you are doing the right thing. “Just wondering” or “just making conversation” has a different feel to me. “Just checking” sounds like something a boss would say. It’s…not really her place to “check” whether you have OKed your hours with your boss.

    I like the idea of responding to that with “but why?” or “I’ve got it covered.”

    LW2, your coworkers really are behaving like a bunch of teens. How on earth does it affect them if he waits until marriage before having sex?

    1. irene adler*

      RE: #1- yeah, I’d be bothered by this too. For the same reasons you cited.
      If this behavior repeats frequently, I’d be inclined to ask the boss why co-worker was monitoring me. Because if there’s a problem with how I’m doing things, I’d prefer the boss bring that issue directly to me so I can make things right.

      (More than likely boss knows nothing about the co-worker’s inquiries.)

  39. Dread Pirate Roberts*

    For LW2, if you feel comfortable with this it could be a nice gesture to find ways to check in with Bill in a friendly way. Not to say “wow, sucks that people are mocking your virginity, huh?” but to show him that not everyone he works with is a glassbowl or feeling too awkward because of the glassbowlness around him to be an ally.

    1. goddessoftransitory*

      I think this is a great idea. Just one professional, friendly face could mean the world to Bill.

    1. learnedthehardway*

      Agreed. Some people have phenomenal memories, and really don’t need to take notes.

      Personally, I have an absolutely lousy memory, and while I might bring paper and pen to an interview, it doesn’t mean I’m a better candidate than someone who doesn’t. In fact, despite my lousy memory, I don’t really have the time to take notes in an interview – I’m too busy focusing and answering questions. If I’m taking notes, I’m not really analyzing – I’m writing. These things are rather separate for me.

    2. OP4*

      This is…a lot. I’d even go as far as to say that this a pretty unfair characterization of my letter! I tried to be clear that expressly don’t want to operate in a way that overvalues tradition or professional norms that don’t serve me or my team well. And that also bringing or not bringing a notepad is not anywhere near the top of my list of things I’m evaluating in a candidate, because literally, why would it be.

      I was remarking on a trend I’ve seen in my experience and wondering about the convention behind the expectation. I’m not upset in the slightest, just curious about my own reaction and wanting to seek out other opinions on it. The judgment you’re reading in it is simply not there.

      The personal comments about the kind of manager I am are just not necessary, especially when, at the end of the day you and I don’t even disagree about one of the potential usefulness aspects of the practice. Your use case is what I’m assuming most candidates would be thinking!

  40. I should really pick a name*

    LW#4

    I was reflecting on why it bothered me so much and I realized that so long as they bring something, I don’t really notice or care if they take their own notes.

    That’s a pretty big sign that your expectations are based on “this is how things are supposed to be done” and not any real need on your part. This might be one of those cases where by writing out the question, you answered it yourself.

    You also specifically mentioned them not bringing copies of their resume.
    If the interviewers don’t have copies of the resume already, that reflects poorly on the interviewers, not the interviewee. It shows that they didn’t prepare properly.

  41. Falling Diphthong*

    A generational anecdote:

    On a family trip this Xmas, my 21-year-old child asked if anyone had paper and pen. I did, and gave him the pen and small pad of paper from my purse. He thanked me, but asked in genuine surprise why on Earth I was carrying these things? (I think some sort of theoretical math quandary had come up, possibly requiring a diagram?)

    I am someone who would likely have pen and paper to take notes in an interview. (And if I hadn’t brought them, could clearly produce them from my purse in miniature form.) But I am surprised at the level of professional signaling being ascribed to it here, where I definitely don’t view it for myself as more than an “I like taking physical notes on paper” quirk.

    1. Peanut Hamper*

      I love technology and work in a (mostly) paper-free environment. (My last job was also 95% paperless.)

      But when I really need to think (especially creatively), there’s nothing like paper and a pen or pencil. Typing is not the same. I need to make each and every character myself, or my brain just does not kick in the way I want it to.

      1. ProRata*

        Can’t cite the article at the moment, but I believe research has shown most people have better recall from hand-writing notes v. typing….how the writing process imprints the neural pathways more efficiently than typing.

    2. Eff Walsingham*

      I am That Person With The Pen And The Paper among those I hang out with. I am GenX, and a lot of us were early adopters (who else had a Palm Pilot? Lol) but I later came to feel that pen and paper works better for me personally.

      During the high pandemic era, when people didn’t want to touch each other’s stuff, more pens were carried, I found; but now everyone’s given them up again. Fortunately my husband’s work is an endless source of promotional pens, because frequently someone I’m with suddenly needs one. And I like to tell them to just keep it, so that then they’ll have it for next time.

      My sister in law does this all the time, even in her own home! “Does anyone have a pen?” And I’m getting fed up. They’re not expensive. They’re not heavy. I’m thinking of getting a t-shirt made up that says “I am not your Bag of Holding”! It’s like she wants to feel all paperless and unencumbered, but still get to use these things on demand.

    3. Don't Call Me Shirley*

      My 10 year old literally always has paper and pencil to jot down notes or sketch. She says it’s because her hands get bored.

  42. Team PottyMouth*

    To apply, I upload my resume, then spend 20 minutes filling out proprietary forms with data that could be pulled directly from my resume. Then they ask me to bring a copy of my resume to the interview, they take it and set it in on top of the copy of my resume that they printed out and then ask me questions that indicate that they didn’t read my resume, and then they end up hiring someone whose resume had more experience, which they knew before the interviews even started, and they want to reject people who aren’t note takers?

    1. Telephone Sanitizer, Third Class*

      Exactly. I still think it’s useful to have something to take notes with but it shouldn’t be a make or break.

    2. bighairnoheart*

      I think this comment has actually made me realize why there are so many people up in arms DEFENDING their right to take notes/not take notes. Interviews are fraught and emotional for a lot of people because your livelihood depends on being employed, and the whole situation feels unbalanced for many reasons, so the convention of bringing the right things and performing the correct actions in an interview probably riffles a lot of feathers.

      But OP specifically said that this is “not a deal-breaker” for them, so that last point isn’t relevant in this case.

  43. Peanut Hamper*

    LW#5:

    Of course your CV is about you. Which makes the use of the word “I” completely superfluous. Who else would you be talking about in your resume/CV? Dave? Jim? Nope, it’s about you and just you. ;)

    Don’t use the word “I”. You don’t need complete sentences on a resume/CV, and for space reasons, shouldn’t have them.

    1. Slow Gin Lizz*

      Agreed. Bullet points are your friend. They absolutely do not need to be complete sentences.

    2. Samwise*

      Yeah, that “I” is annoying. Get to the point. I’m not reading a memoir, it’s a resume.

    3. learnedthehardway*

      Agreed – Recruiters can scan a resume in less than a minute to determine if they want to interview someone. Don’t make it any more difficult than it needs to be. It looks out of touch and it’s going to cause recruiters to think you’re not presenting well.

  44. Astrid*

    #4 I had no idea that it was the norm for candidates to take notes. Frankly, I think it would signal some sort of cognitive limitation if a candidate could not retain information from a standard 30-45 minute interview (to jot down post-interview).

    1. Hlao-roo*

      Oh, wow! Are you an interviewer? If you are (and even if you aren’t), I really hope you read through the comments here and reconsider your stance that taking notes is a signal of cognitive limitation.

      Two points I want to highlight for you here (though there are many other arguments in favor of taking notes):

      1 – People are often nervous during interviews, and that can make memory worse. It helps a lot to jot down a few key words/phrases in the moment, so the candidate can refer to them later without worrying they forgot to retain a key piece of information.

      2 – About half of my interview experience has not been a single 30-45 minute interview. Instead, sometimes my interviews are 4 back-to-back 30 minute interviews (2 hours total, talking to 4 different people one-on-one). That is a lot of information to retain without writing down anything.

    2. I should really pick a name*

      That’s an odd assumption to make.

      Maybe some people take notes, some don’t and it doesn’t provide any insight into their character?

    3. Angstrom*

      People who are highly stressed often have poor or incomplete memory of what’s been said to them. This is why it’s often recommended that one not go alone to a medical appointment if there’s a serious issue to be discussed.

    4. Meghan*

      I mean, there is usually a ton of information that results out of a (halfway decent) interview. I would definitely not fault people for writing a few things down.

    5. deuceofgears*

      Beyond the issues of stress/nerves raised by other commenters, I imagine it would depend on the role? If the role requires remembering information and being able to process/regurgitate it under stress, then sure. But if the role is one where “refers to notes to get the job done” is fine, why not?

      I do have some cognitive limitations caused by medications. I take assiduous notes any time I’m talking to my agent, film agent, editor, client, etc. on the phone or a Zoom call. It’s never been a problem, and also, my job (novelist/freelance writer) is one where “needs to refer to notes to do the job” is completely fine. It’s not like my editor expects me to verbally produce my entire novel out of my head in person! When drafting, I refer extensively to my outline and notes, and no one cares because all that matters is the final product.

    6. FashionablyEvil*

      This is a bizarre interpretation. I often jot down notes when I want to make sure we circle back to a topic, it’s something I want to ask HR or another interviewer about, it’s a key part of the job that wasn’t clear from the job description, etc. I’m not sitting there taking verbatim notes.

    7. Alton Brown's Evil Twin*

      Wow – how do you handle a day of back-to-back-to-back interviews? That would be a pretty impressive feat of memory to recall at 3 pm what you talked with somebody about at 9 am.

      1. Valancy Trinit*

        This is really all that needs to be said. I’ll give no concession to this bizarre and frankly cruel take.

    8. Environmental Compliance*

      Aight, for one, what a completely bizarre take on this. Two, some of us might have things like ADHD or memory issues from PTSD. Three, only 30 minutes? None of my interviews have been less than four hours. 30 minutes is a phone screen. Four, for highly technical or detailed roles, this isn’t going to work for a lot people either.

      1. Eldritch Office Worker*

        Or even just interview anxiety! it’s normal to have a foggy memory of a high stress situation and whether or not we like it, candidates are human beings who experience stress when put on the spot in a high stakes situation like a job interview.

    9. Elspeth McGillicuddy*

      Yes, it’s cognitive limitation called “having a normal brain”. Most people can carry like, 2-3 pieces of information in active memory without having to constantly go over them, and even that requires effort. And for me at least, things that I need to remember to do have to stay in active memory.

    10. metadata minion*

      Yes, I do have a cognitive limitation relating to working memory. This is why I take notes. It’s a wonderful adaptive technology available since about 6000BCE. Unless the job requires retrieving memorized information on the fly with no access to notes (in which case I wouldn’t have applied because *I am bad at that*), this isn’t a limitation that’s generally problematic.

      1. I have RBF*

        This. I have memory issues. If it isn’t written down it might as well not exist. While I seldom take notes in an interview, I try to write stuff down afterward. I really should start writing down names, at least, because that’s my worst point.

    11. The Gollux, Not a Mere Device*

      That’s not a “cognitive limitation” unless being unable to solve differential equations in your head, or only speaking three languages fluently, are cognitive limitations.

      Most people’s memories aren’t as good as they believe them to be. That’s a problem if I think I should send a message to Jane73@example.com and the correct address is actually Janey3@example.com. People write down things like that, or that instead of Astrid interviewing them as scheduled, they talked to Fergus because Astrid was unavailable, and they should send any follow-up questions to both of you.

      Unless you’re hiring people for a role where remembering several possibly unrelated things precisely and accurately for 45 minutes, while answering questions, is a job-related skill, expecting everyone to have that kind of excellent memory is unreasonable.

    12. Chirpy*

      I have always found that the act of taking notes (by hand) makes me remember information far better- even if I never look at that note again. It’s a kinesthetic learning process.

      1. Lime green Pacer*

        Yes! Also, somehow it helps me to focus in the moment. Pen and paper are often better because my device has a zillion possible distractors. (I use Freedom on my laptop to avoid distractions there.)

    13. LilPinkSock*

      Wow. Do you truly believe that someone has a cognitive deficit if they write down a few details during an interview?

    14. biobotb*

      I guess you don’t provide interviewees very nuanced or detailed responses to their questions, then. Or think they may not be interviewing at other places that they want to compare later?

  45. Meghan*

    Absolutely amazed that people don’t take notes in interviews (Millennial here). I mean, I’m not transcribing the entire thing, but I definitely jot down things that I’m told about the job that aren’t on the posting. Also before the interview I’ll write down some of the questions I have for the interviewers and I might write a word or two next to them when I get their answers. Also my interviewers names, because I’ll immediately forget it in the craziness of an interview.

    People just… remember everything about their interviews?

    1. Meghan*

      Also an addendum, I’ve been in the situation where I’ve been interviewing multiple places at once and sometimes its hard to keep stuff straight post-interview (who said what where), so its nice to have something to look back on. They dont always keep the job posting up, so its even more important to keep your notes around.

    2. Vax'ildan is my disaster bicon*

      I’m equally amazed and baffled! I definitely take notes, both bringing my important questions and jotting down information I want to remember/follow up on. It helps me to remember the distinguishing factors between different organizations–sometimes when you’re interviewing a lot, they all start to blur together. Not to mention the fact that the physical act of writing something down helps me to remember it, whether or not I actually refer back to my notes. Do people really remember everything from their interviews, even when they’re stressed?

    3. FashionablyEvil*

      I am also baffled by the number of people who seem to disdain notes (elder Millenial here). Interviews are stressful or sometimes the conversation doesn’t flow in such a way that gets you all the things you want to ask about. Notes are very helpful for that! Different strokes, I guess.

    4. Lacey*

      Yeah, I was wondering about that as well. I have to take notes because otherwise I will come away remembering NOTHING that was said except for the one dumb way I responded that made me want to crawl under the table.

      1. Blanche*

        Another Millennial, and I have never taken notes during an interview (and I don’t bring anything with me to take notes). I also don’t bring along paper copies of my resume. There hasn’t ever been a time I’ve needed to take notes during an interview.

        1. Just Another Zebra*

          Middle Millennial, and I agree. There’s never been an instance where I needed to take notes. Someone taking notes, to me, is not engaged in the conversation. Isn’t that why people are told NOT to write down their answers to common interview questions? Because it breaks the engagement and flow of the conversation.

          1. Meghan*

            For the most part, I’m not writing down answers beforehand. I might write down particular instances (i.e. “the Sarah story”) that I think might be helpful in answering a potential STAR question, but the above is about all I’ll write.

            I’m taking information down like names, and answers to my questions. It feels like a lot of the non-note takers think that people are constantly writing, whereas (at least I am) its more of a word or a phrase jotted down on the page.

          2. Cruciatus*

            I’ve seen a few people write this now, that it doesn’t look like you’re engaged because you take notes. I don’t get that at all. I think a person could look MORE engaged because they are taking notes. I’m taking it in. I’m not writing every word down, but things that seem important either about the hiring manager, or about the job. It can help me, even in the moment, realize “they’ve brought this up a couple of times now, I should ask a question to dig deeper”. It might depend on how one does it, but I try not to take up time and keep everything moving along. I don’t think I’ve ever been hindered by note taking, and, well, if I have, then I guess I wouldn’t want to work there anyway.

          3. RagingADHD*

            Taking or referring to notes is not the same as staring at your paper the entire time or reading a script verbatim. It’s more like:

            Interviewer: “So, did you have any questions I may not have addressed?”
            Candidate: (Briefly looks at paper, sees note that says Travel?) “Oh, yes, how much travel is expected in this role?
            Interviewer: “About 15%.”
            Candidate: “Oh, okay, great.” (Nods and writes down 15%.)

            It’s a checklist.

          4. bighairnoheart*

            Wait, what? I swear, Alison actually advocates for interviewees to write out (or at least outline) their answers to common questions and/or practice saying them out loud before interviews to help them practice and get those responses clear in their minds. Am I making this up?

            1. Ask a Manager* Post author

              You are not making that up. I don’t advocate for bringing those written answers into the interview with you though, which is what I think the OP you’re replying to was talking about.

        2. Lacey*

          Yeah, I’m a millennial too, I just like to be able to jot things down that I want to remember.

          I totally understand not printing your resume though. I haven’t been to an interview in years where they didn’t have it up on their ipad or, once, projected onto the wall!

    5. RagingADHD*

      I know, right?

      You know, there is always a question from time to time about people who freeze up in interviews, or blank out on their behavioral examples, or they can’t remember the name of the interviewer to send a thank you, or they got confused about something regarding the job, and when Alison or the commenters say “bring notes / take notes,” they act like they’ve never heard of it.

      It’s a really useful coping mechanism, and just generally a good idea. I understand people pushing back on the idea that you “have to” take notes. But I really don’t understand why people are acting like it’s totally unnecessary, ever, or being scornful that it’s “outdated,” etc.

      Perhaps they are interviewing for jobs where nothing unexpected ever happens, so they can ask exactly the same questions every time about every role, every job pays the same, and they don’t need to be prepared for contingencies.

    6. JustAnotherEmailMarketer*

      Right?? I’m stunned coming to the comments and reading how many people think taking notes is a bonkers idea and the judgment for people who do! Granted, I haven’t had an in-person interview in 3+ years, but I always took a notebook and a pen. I would jot down notes on things like the team structure — how many people is it? who does the team report to? — things that were definitely not just listed in the job description.

      I would also jot down answers to the questions I asked at the end of the interview — obviously I would also not already have those answers either! I’m not sitting there transcribing the whole interview — it’s probably going to be a maximum of half a (small) page with bullets.

      Truly did not expect the commentariat to be so polarized on this

      1. Lacey*

        Or maybe different people function differently?

        People who take notes aren’t wasting time – they’re doing it while people talk. If you’re a note-taker it’s nothing. Obviously you aren’t one, but that doesn’t mean other people aren’t functioning.

      2. Willow Pillow*

        I am autistic and this perception feels so dehumanizing. My short-term memory is poor because my brain takes in so much more information from my surroundings – I’m functional because I write stuff down (or use my phone). People aren’t all the same, and there are multiple ways to accomplish things. No one is forcing you to change if what you’re doing works for you!

        1. NB*

          Exactly! Taking notes is how I stay functional. No one needs to insult other people for not having an excellent memory.

      3. RagingADHD*

        Again, the disdain. Why?

        Some people have to rush from one interview immediately to another, or squeeze an interview in between other intense meetings or tasks. Some people just have an awful lot on their minds or going on in their lives that they are juggling. Some people get nerves. Some people get engrossed in the nonverbal or conversational dynamics and forget details (this kind of person is usually very good at establishing rapport, actually). Some people aren’t going to get an opportunity to debrief “in couple of hours.” Maybe not until the next day.

        “I have a memory,” geez. Everybody has a memory. Some people use tools because it makes them perform better. It’s great that you can carry your entire life around in your head without ever using an external record. But it is not “dysfunctional” to do otherwise.

        I mean, do you think grocery lists are “dysfunctional”? Meeting agendas? Syllabi? Calendars?

        Why be so disparaging? It’s really gross.

      4. FashionablyEvil*

        I mean, a huge part of my job is qualitative research (aka interviewing people). I am great at it! Do you know what I’m doing the whole time? Taking notes (even when we have a note taker or are recording/transcribing the interview). Why? Because it helps me to focus on what they’re saying, process what they mean (often different than what they say) or remind me to follow up on something they’ve said in passing.

        You show you’re actively engaged by listening to their questions and responding thoughtfully. The notes don’t really come into it!

      5. New Jack Karyn*

        This is deeply unkind, and I request that you consider how your comments come across to other people.

    7. Allonge*

      I don’t need to remember everything about the interview.

      1. Companies I interviewed with were capable of sharing information in writing. Information I could only get from the interview itself was usually very limited.
      2. Training for the job starts after I start the job. I never needed to know org setup or team structure just based on the interview, although of course it can be useful.
      3. My memory in fact is pretty good. I could tell you at least half the questions I was asked during an interview 11 years ago. Remembering what they said about the job right after is not an issue.
      4. I love stationery and I would most likely bring a nice pen and paper anyway, but being judged on whether I take them out of my bag or not rankles, and this is not even touching on the phone question.

      I learned to work with my brain and I very much appreciate everyone who has done the same with theirs. So I am perfectly happy to have anyone take notes, but please trust that I don’t need them.

    8. Bit o' Brit*

      My memory is terrible, but I can’t think or listen at the same time as writing, so I never take notes if I can help it. All I need to remember from an interview is whether the job sounded like one I would want. I don’t need the details, because anything “hard facts” will be part of the offer letter/contract, I just need the impression I had and whether that was positive or negative, which is very easy to remember for long enough to record it after the interview (most likely in a text to my partner).

  46. Marketing Unicorn Ninja*

    I have a great memory for faces and a terrible one for names, so I always jot down a seating chart with people’s names/titles, so that if I get a second interview and I see the same person and I remember their face but not their name, I can refer back to my notes to jog my memory.

    (I bring the same portfolio and my previous notes to a second interview if I get one.)

    1. Alton Brown's Evil Twin*

      I used to do this exact thing with business cards at meetings w/ lots of people I’d never met before.

    2. JustaTech*

      Oh, this is such a good idea!
      I’ll have to use it the next time I’ve got a big roundtable.

  47. Not A Manager*

    “Why do you ask?” – “Just checking.” – “Okay, but why are you checking?”

  48. Lifelong student*

    Have read most but not all of the comments. Many people suggested making notes on a phone. It would be a huge turn off to me if someone pulled out a phone during a professional meeting- including interviews. In addition, if you have a phone turned on- how do you prevent it from making noise during the meeting? I suppose you could turn off sound.

    1. Falling Diphthong*

      As a very low-tech person with an iphone: To prevent it making noise, I would have flicked the button on the upper left of the phone down to turn off the sound. I would have done this before the meeting, so if I needed to use the phone to verify or note something, sound would already be off.

      I am a paper-and-pen person, but I view that as a personal tactile quirk and am astonished at the Great Signaling being ascribed to it in this thread.

      Do you look down on people who check the calendar app on the phone to see if next Wednesday at 11 would work for the follow-up interview, because they didn’t print out their calendar and bring it in paper form to the interview?

      1. JustaTech*

        Several people have suggested elsewhere in the comments that if you need to use your phone in an interview or other important meeting (to do something like email your resume or check your calendar) that you explicitly name what you’re doing *before* you pull your phone out – “Would Tuesday at 3 work for a follow up?” “Let me check my calendar” (pull out phone).

        Because smartphones can do so many things and from the back of the phone all those things look the same (checking calendar vs checking texts vs playing candy crush), it can be important to clearly state what you’re doing with the phone so that an interviewer doesn’t leap to “farting around” (even if not all interviewers would do this).

    2. Milfred*

      Either leave your phone in the car or power it off.

      Unless it was job related (let me show you the mobile app I wrote) I would be put off by someone working on their phone during an interview–even if it is to take notes.

      I suppose the stereotype of people with their heads rudely buried in their phones at the most inopportune times resonates too much with me.

  49. FashionablyEvil*

    Yeah, I don’t think there’s a good solution here. Plenty of people do take notes on their phone, but I agree it doesn’t look good.

    As far as the sound, that’s what the “do not disturb” function is for.

  50. curio*

    LW1: A potential different perspective–coworker could be checking less to see if *you* are doing what you’re supposed to, and more to see if *boss* is treating everyone fairly. By which I mean, maybe boss approved your schedule change, but never seems to approve theirs. Lord knows I’ve had times where I’ve kind of wanted to ask my coworkers things like that (I haven’t, because it would likely just get me in trouble, and it is, as you say, more than a little nosy. But when it seems like everyone else is getting better treatment, you kind of want to “just check.”)

    That said, still isn’t appropriate, and if your instincts are telling you it’s less that and more her trying to act like she’s some sort of sub-manager, that probably is the case (zebras/horses), and Alison’s script is best.

  51. curio*

    LW2: Oh PLEASE do report them! They’re already making Bill feel terrible, and you never know who else in the office falls under that umbrella for whatever reason. I’d be pretty angry if I heard folks making comments like that. Also, aside from the religious harassment, they’re talking about a coworker’s sex life! In what world (outside of VERY specific jobs) is that remotely appropriate?!

  52. JelloStapler*

    #2 speaking of things that are none of their business. Wow, how heartless, immature and cruel.

  53. EPLawyer*

    #3 – what you can do is use this as a learning opportunity. Ask your boss what you need to do in order to be competitive for that position in the future. Make it clear you aren’t jumping now, but you want to move up. You should be having these talks with your boss anyway but you can bring it up now.

    #2 — Talk to HR NOW. Today. This is not acceptable. Bill’s religion is NO ONE’s business. His juvenile coworkers should not be allowed to keep picking on him about it.

  54. HonorBox*

    OP2 – I’d suggest the following:
    1. Say something kind to Bill. Let him know that not every one of his coworkers sucks pond water. You don’t need to call out the specifics of why you’re being nice to him. But let him know that you’re “Team Bill.”
    2. Report this ASAP. To your boss. To your boss’s boss. To HR. This is bullying. This is discrimination. This is harassment. And based on both sex and religion. Your company is being opened up to a HUGE lawsuit. Get out ahead of this by reporting it.
    3. Maybe mention to Bill that you think he’s being harassed…legally speaking. Let him do with that what he thinks is right, but this is the kind of thing that is absolutely worth tearing things down for.
    4. Might not be a bad idea to also mention to your coworkers that what they’re doing is both juvenile and illegal. I don’t think I’d search out the opportunity, but if you hear something being said, in front of Bill or not, you should jump in and say something.

    Your coworkers are absolutely awful, OP. I’d consider looking elsewhere, too, given the fact that this has gone unchecked for such a long period of time. Your workplace doesn’t seem overly supportive. Remember… Bill hasn’t brought this up in the workplace. Someone from your workplace found it and decided it was water cooler fodder. That makes this even worse.

    1. Coco*

      Yes LW2, please follow these steps. Please say something kind to Bill. You don’t have to mention the situation directly, but let him know that you value him as a team member. Report it immediately. Consider being more stern when addressing these jerks. Speak up in the moment and shut it down.

  55. M*

    Re #4: I have always brought copies of my resume with me to interviews, but I have never – not once – been asked for one. I also never take pen and paper notes during an interview because i feel like it would make me look distracted!

    1. Samwise*

      The last time I was asked for one was in the 1990s. It was a power move by an asshole associate professor who was threatened by the mere existence of women who might be smarter than him (spoiler alert: he was not as smart as he thought he was). Indicating: I can’t be bothered to look at your cv because you are so inconsequential and why are we even interviewing you at all. Everyone in the room understood the message and there was some uneasy shifting in seats.

      Why yes, Professor Dickhead, I do have a copy of my cv! Would you also like a copy of my article on the theoretical underpinnings of llama herding culture, which I’ll be discussing this afternoon? in case you want to glance at it in advance? No? [small chuckle] Well, prepare to be amazed this afternoon!

      Haha. That got some smiles from other members of the search committee.

    2. Lilo*

      I once got massive side eye for not having a ready copy of a writing sample at an interview. I had just about every other document too.

  56. cardigarden*

    LW 4: I recently had this issue. I think by itself not bringing anything for note-taking is a little weird, but generally I default to a “hey, you do you” philosophy. But in my case, the lack of materials really enforced some of the other red flags I had picked up on (giant ego, could they shift away from previously being a big fish in a little pond, were they manage-able). So you should look at it within the wider scope of the rest of the interview.

    1. cardigarden*

      From reading upthread: It also seems like maybe there are different ideas of what constitutes note-taking in an interview. I’m not sure anyone who advocates for note-taking is suggesting that the interviewee is taking notes for the group to be distributed at the end. I currently work in academia and our interviews are… excessive (full day or sometimes 1.5 days depending on position/rank to cover conversations and sessions with all stakeholders), but I use notes to jot down a few words to jog my memory for relevant information that the interviewers are providing me, to remember who said what, etc.

      Something I noticed from other (outside the US) commenters is that it seems like in the US there’s a huge component in the interview for the job seeker to ask questions of their own to see if the company is a good fit for them. There’s also a general assumption from the company that questions from the candidate indicate interest in the position beyond a paycheck and benefits and an engagement with the process. I would find it super weird if a company thought it was weird of me to ask questions.

  57. DrSalty*

    #2 – is this not sexual harassment, in addition to religious harassment? No one should be commenting on anyone’s sex life or sexual preferences at work

    1. SweetFancyPancakes*

      And even if it’s not sexual harassment now, this kind of behavior can spill over into official harassment really easily- how long before they start leaving porn or sex toy catalogs on his desk (this is what happened to my sister after she got engaged- a couple of older women in the office thought it was hy-larious to “educate” the young one).

  58. MCMonkeyBean*

    I see a lot of people talking about not bringing a resume but I actually have something else I highly recommend people bring with them if they can (though I still am on the side of having a couple resumes on hand just in case)

    The last time I interviewed I typed up notes for myself to reference during the interview. It included some things from my resume that I thought were most relevant to that specific interview as well as some bullets to remind me of possible answers to some of the most common questions, some notes about why I’m interested in the job and the company. Basically notes to help for those moments when someone asks you a question and your mind goes totally blank! Obviously you shouldn’t just sit there and read off of the notes, but having them on hand was very helpful for me.

  59. Fluffy Fish*

    OP4 – TBH the concept of bringing resume copies has always irritated me. Why don’t you have a copy of my resume? Since your company already required I provide it? Seems that you would be the unprepared one, not me.

    To me its yet another example of the weird unwritten hoops companies expect employee to jump through that boils down to power struggles.

    I’m not young either so no chalking this up to generational difference. I don’t bring my resume and I don’t take notes. An employer who penalizes me for that is an employer I’m not interested in working for. Finding employment has never been an issue for me.

    1. Former Retail Lifer*

      I’ve already submitted a resume and probably also entered all of that exact same info into the application. If you can’t make do with that and still require me to hand you a resume on paper, it’s a red flag for me.

      1. Fishsticks*

        Ugh, yes. If I had to submit all the information that is on my resume piece by piece in your automatic application form, and THEN attach my resume and cover letter separately anyway, the least the employer can do is, uh, look at the resume before the interview.

  60. L. Bennett*

    I always bring something to write on for an interview (or any meeting), even if I don’t use it. Yet, based on the comments here, I guess I’m in the minority? I just thought it was standard practice for any professional meeting.

    1. The Other Dawn*

      I’m in the minority with you. I might not use it, but at least I have it if I need it.

      As an interviewer, I wouldn’t expect the candidate to take notes, but I would expect they come prepared in the event they need to write something down. I’ll never forget the guy my boss and I interviewed at a previous company. He didn’t bring anything with him at all. He decided he needed to take notes, so instead of asking for a piece of paper, which I had, he grabbed the 3×3 PostIt pad next to the phone and took eight PostIts worth of notes in teeny tiny writing.

    2. bighairnoheart*

      I don’t think you’re actually in the minority. I just think the commenters today are weirdly skewed towards non-note-takers (no judgement either way! But bringing a pen and paper to an interview and writing something down during an interview is common and acceptable, regardless of what a few strange comments are saying).

  61. Ssssssss*

    #4 – Since a lot of the folks you’re interviewing are less experienced, why not have a short document you email over with the interview confirmation that goes over some common interview norms or other useful info?
    Here’s where to park, we recommend bringing a notebook and 3 copies of your resume for backup, typical interview attire for our company usually looks like this…

    1. L. Bennett*

      I like this. It would also help you to see how many people actually read and follow instructions, which is useful info to have on potential candidates.

  62. Yorick*

    LW4: I have never learned information in a job interview that I would need to write down to remember. It’s perfectly fine to bring a notebook and take notes, but it doesn’t seem necessary.

      1. Falling Diphthong*

        “… it doesn’t seem necessary as a universal requirement for all job candidates, so probably shouldn’t be something I judge them all on.”

        Particularly as OP admitted they don’t notice if the materials are used to take any notes, just that they are set out on the table beforehand.

        (To be clear, I would be in the group who brought pen and paper to the interview. But I wouldn’t ascribe any meaning to it beyond my personal tactile quirk for retaining information by printing it by hand.)

        1. Yorick*

          This is what I meant. I definitely think it’s ok to bring something to write notes, but think it’s weird for someone to expect it so much.

      2. Fluffy Fish*

        That feels really rude and an ungenerous interpretation.

        OP thinks it is necessary.

        Saying that it doesn’t seem necessary is saying its optional. As in each individual can decide for themselves and OP shouldn’t hold it against them. Not that its no one should be taking notes.

  63. Laptop Lady*

    L3: I agree that I wouldn’t directly apply for this role so soon after being hired, but I’ve found that the best internal moves you make usually aren’t started through the internal job portal. Depending on how large your organization is, I’d recommend getting to know your new coworkers better 1:1 by taking inviting them for coffee, or intro chats…one of these meetings could be the hiring manager for that role. In that conversation, you could share a little about your background and when it comes up naturally in convo, you could talk a little about how you’d be happy to help lend an extra hand while they’re hiring for the new role. This way you get a sense for what your perfect fit team/role would actually entail (maybe it isn’t the dream role) and you’re building relationships with this team for the future. You definitely have to use good sense and transparency while you’re doing this so it doesn’t appear like you’re going behind your new bosses back, but just having these informal coffee chats with a variety of folks in the org can be a great benefit in more ways than this role opening. Also, use your judgement on how busy you are with the new role and if you realistically have the time to go above and beyond with some additional tasks/or if you get the sense they could take advantage of that….

  64. HonorBox*

    OP2 – This is juvenile and awful behavior from your coworkers.

    As well as reporting it, I think I’d remind someone next time you hear them talk about Bill’s virginity that what they’re doing constitutes harassment and just cruel. You might also want to in a not-so-obvious way let Bill know that you’re on his side. Perhaps encourage him to say something to HR, too.

    When you report it, please point out that Bill has not been the one talking about religion or sex, but rather someone found the video, and people have been talking about it (harassing Bill) often since. This is a coworker problem and poor Bill is having to endure a lot.

  65. SMH*

    I had one co worker who would point out if I came in later than others or changed my schedule at all. Finally in one conversation I told her ‘I don’t report to you.’ She got the message and we actually became friends but sometimes you just have to be direct.

  66. Just Another Zebra*

    I admit that I enjoy when there is healthy discussion in these comments, like letter #4. If nothing else, it demonstrates that things being “the standard” are not the standard at all.

    I’ve never taken notes in an interview. Not once. There’s never been a need! I may have a paper typed up in advance, usually with my interviewers name, title, a few bullets from the job posting, but I usually review it in my car before going inside. To me, an interview is about the engagement, the back and forth of questions and answers. I wouldn’t take notes for the same reason I wouldn’t type my answers to “standard” questions up in advance – it defeats the point of the interview, IMO.

    And as for a resume… I may have one on hand, but if you, the interviewer, don’t have a copy of it and have clearly not read it (or the online questions that I filled in with answer found directly from my resume), that’s a red flag for me. If I have to be overly prepared and know the company’s history and mission statement, I expect you to have at least skimmed a bulleted, one page document.

  67. Former Retail Lifer*

    #4: I’m in my 40s and I don’t bring anything to an interview. You have my resume and application that were submitted online, so there’s no need to bring another copy. If I want to jot something down, I’ll do it on a phone app, but I’ve honestly never needed to take notes in an interview. In the industry I’m in, there’s little to no difference in pay, PTO offered, benefits, etc. between companies, and even if there was something that stood out, I’d remember it. I’ve been in the workforce since 1994 and I’ve never had any need to take notes in an interview.

  68. Bill and Heather's Excellent Adventure*

    LW2, oh my gosh, poor Bill! Please inform HR about what’s going on, this is horrible. Might even constitute sexual harassment although IANAL so I’m happy to be corrected.

  69. Alex*

    From LW3’s description of their work environment, I wonder if they might be in a federal government organization (particularly in Canada) – this kind of move is so, so common in a lot of the departments I’ve worked for, to the point that managers frequently send opportunities to their own team members. A lot of positions pretty much exist to serve as that kind of stepping stone, and rotations and acting positions are an extremely common way to move around internally. If that’s the case, LW3 probably could actually get away with applying for the internal position, as long as they’re not too cavalier about it. *IF* LW has a good feel about their manager, I’d talk to them about it and ask what kind of mechanisms are in place for internal transfer – or at least if they have a sense of how often those kinds of opportunities arise.

    1. learnedthehardway*

      If the person has been with the company less than 6 months, they really shouldn’t be applying for other roles internally. In fact, they should check with HR what the minimum time is for them to be in a role before they do apply for other positions – most companies have a minimum period before they will move someone internally.

      If, as Allison mentioned, this is a critical role that is very hard to fill, and there is a compelling reason from the company’s perspective to consider this new employee for the role, then speaking to their manager might make sense. But I’d be careful to not be seen as someone who is not committed to the role for which I had been hired.

  70. LB33*

    Curious about the one with Bill – if the advice is to talk to the boss, would you recommend the LW talk to Bill about that first (maybe he doesn’t want to escalate it?), or just go ahead and do it?

    1. HonorBox*

      I’d just do it. While it would be nice to make sure Bill is aware, it sounds like OP is uncomfortable with the situation, which also could constitute harassment too.

  71. Zee*

    #4 – It’s funny, I ALWAYS take notes in interviews because I’ve got a terrible memory (interview nerves don’t help either), and I’ve gotten the side-eye quite a bit from interviewers who I guess don’t think I should be doing that and/or don’t want to wait 5 seconds for me to finish writing something down.

    1. Pikachu*

      #4- I do not do well in interview situations. I find them extremely high-stress and I am not suited to the performative nature of them. I tend to get stage fright and my mind goes blank. Palms sweaty, knees weak, arms heavy, mom’s spaghetti, all of it. I’ve practiced and worked with coaches but I have never gotten more comfortable in interviews.

      I learned to bring an annotated copy of my resume. It includes a few bullet points/keywords to remind me of things I want to mention about specific line items in my resume, in case I forget in the moment. It’s not one I would share with interviewers.

      I take some basic notes, but it’s usually circling or noting on my copy what seemed important/not important to the interviewer, things I did or didn’t get to mention, tidbits about unique benefits that I might not already know, tech platforms the company uses that I should look into, etc.

      Most interviews don’t become jobs, so I try to use every one as an opportunity to get better for the next one too. It’s nice to have a brief reference to turn to when you’re preparing for the next.

  72. TheCakeIsALie*

    Re: notes – I have gone on many a pre-pandemic job interview, worked with staffing agencies, solicited advice during informational interviews….never once have I received advice to bring a notepad to a job interview. I’ve taken notes during virtual interviews, and after in-person interviews I will likely jot things down on my phone after the interview, but it has never occurred to me to bring an actual notepad to a job interview. I had no idea people did this. I’m a Millennial who has received plenty of job searching advice from Gen X and Baby Boomers.

  73. HailRobonia*

    RE #2: I am a gay man, and in one of my early jobs in some conversation I let it slip that I “had no experience with women” which I had intended as “hey folks, I’m gay.” Instead, my coworker interpreted it as “I’m a virgin” and gossip spread. It got back to me that some managers were speculating when I will lose my virginity, people assuming I was very religious or conservative, etc.

  74. Megan*

    The LW about the lack of paper to take notes and resume rubs me really wrong, especially in the changing job climate that favors job seekers. I have always thought it was the height of hubris for employers to expect a resume day-of from the interviewee, when a resume was provided to simply apply for the job. You are telling me an employer can’t do the bare minimum of printing out a resume that they have already been provided to prepare for an interview? This thought is coupled with the fact that many young job seekers do not have access to printers.

    1. Fishsticks*

      It always felt like such a weird, petty insult to be expected to provide a copy of my resume for the interview when the people interviewing me should already have the resume, otherwise how the heck would they know who I am or that I applied for the job?

    2. Eff Walsingham*

      Do people really not have access to printers at all? My local library provides this service for $0.20 per page. It seems very popular with job seekers, students, and people needing to tangle with government agencies. And those such as myself, whose printer is being precious this week, or we’ve run out of ink, or something.

      1. Danish*

        My local library is unfortunately about an hour away from me and like most job seekers, I have a job that takes most of my daily energy and time. The process of getting there, finding a printer, and printing out resumes already sounds like so much additional work for me to do on top of my actual job and the Known Huge Effort of job hunting that I wouldn’t do it unless I was deeply desperate, putting all my hopes on this one interview, and wanted to account for every single possible roadblock like “interviewer didn’t bother to bring a copy”

        not to say it’s not a good suggestion, or that people shouldn’t be doing it. Just pointing out that “libraries exist” is still a non-zero amount of cost and effort to ask of a job seeker.

        1. Eff Walsingham*

          Probably library access varies geographically. In the Canadian cities I’ve looked for jobs in so far, there’ve been satellite locations of libraries in many neighbourhoods, despite some local politicians saying they’re a waste of money. So I can use their (libraries’) services across the street from where I buy groceries and bank, etc.

          It sucks, but there’s a long list of expectations on job seekers that take effort outside the core task of basic prep (selecting / maintaining wardrobe that one has no other use for, for example) that have nothing to do with one’s ability to do the job. It’s not fair, but I don’t see it changing.

          These days, I think there’s a greater likelihood that technical difficulties will intrude on an interview, than ye olde “we can’t find your paper resume because everyone thinks someone else had it last”. If I walk in and the panel can’t share or print my resume because their intranet is down or whatever, I can offer them a paper copy, or to get out my phone and email it to them, whatever works best for them. I figure it’s good to appear organized and flexible, and I feel better when I try to prepare for everything that could go wrong. I might not hold it against a candidate who didn’t do likewise, but I’d probably be impressed if they did.

  75. Donner*

    OP3:

    Yes. You can apply for the job. Just apply. You don’t need our permission. Use the usual excuse for when leaving a position is awkward, which is, “I wasn’t intending to seek out a new position, but when I saw how perfect this job was, I just had to apply!” Note that you can only use that once.

  76. Jess*

    LW#1: I had an older co worker pull that crap and at first I treated him with professional courtesy but then one day he caught me in a moment and I sarcastically quipped “Nope, I’ve gone rogue”. and that was the last time he felt the need to inquire.

    Some thing i wish I’d learned sooner is lending social legitimacy to these types just encourages their one-upmanship.

    1. RagingADHD*

      Yeah, once when someone repeatedly asked me where I was going every time I stood up, I got so fed up that I blurted out “Multi-state crime spree, you want I should bring you something?”

      It actually turned out to be the right answer. Never happened again.

      1. Weaponized Pumpkin*

        Just saw a reel musing that you can tell who had siblings — people with siblings announce where they’re going when they leave and ask where others are going / can they bring something back, whereas only children are used to no one being there to care so they just go about their business without announcing it. I am sure that’s not universal but it gave me something to think about!

        1. Cher Horowitz*

          Funny thing about this is that my anecdotal experience is the exact opposite. The people with siblings get up and leave sans a word and the only kids are all about can I bring you anything?

  77. JustMe*

    LW 1 – obviously I don’t know what country you’re in or what your coworker’s background is, but it’s possible this could be a cultural thing. This could be cultural (I’ve seen this happen with people from non-Anglophone countries) or related to background/income/upbringing (where I grew up, people are more plain-speaking, salt-of-the-earth, and direct, which may sometimes come across as rude or intrusive but is just a way of making conversation, finding out what’s going on, etc. I had to unlearn some of it when I went to fancy liberal arts college in another state). It may feel a little weird or invasive, but it’s probably not intended that way. I’d be inclined to shrug it off as just a quirk and move on.

  78. Workfromhome*

    #4 I do find this a=rather amusing in the modern age.

    You go to the website to apply for a job.
    You must attach a copy of your resume.
    Then you need to essentially reenter all the information on your resume into the “application”

    Now you also need to bring essentially a THIRD copy of your resume in case they chose to ignore the first two times you gave them the information.

    1. Angstrom*

      I always thought it was for MY reference, so when the interviewer says “you say in your resume that…..” and it sounds wrong, I can easily find the source of the misunderstanding.

      1. Unkempt Flatware*

        But OP writing in as a hiring manager shows this is not the case or expectation.

  79. Michelle Smith*

    LW2: If the bullying is as you describe it, I would say you not only have standing to report it, but an obligation to do so. If you’re concerned, you can frame the report as making *you* uncomfortable and exposing the company to liability. Even if the subject of the harassment is not reporting it or doesn’t have a problem with it, you have standing to report when it’s bothering you personally as well. (For all they know you are also similarly situated or share those religious beliefs.) There is no reason adults should be making fun of someone for not having sex, regardless of the reason for it. And I say this as someone who did not have a choice about when and how I “lost” my “virginity.” I think it’s a stupid construct and yet I am STILL flabbergasted that grown adults actually think that this kind of bullying over personal choices is happening in a workplace in 2023. It could not be farther from anyone’s business. It is no longer 2005, we can do better.

  80. Pikachu*

    I’m just sitting here wondering how long the behavior in #2 would have gone on if Bill was a woman. My guess is definitely not this long.

    The whole thing is just … barf. Poor Bill.

    1. Bill and Heather's Excellent Adventure*

      I think there would be less mockery that she was ‘still’ a virgin but there would be a risk of harasssment about being the first to change that. ://

  81. Delta Delta*

    #4 – I like to take notes because it helps information stick for me, but I certainly wouldn’t look crosswise at someone who didn’t take notes. I once scheduled an informational interview with a professor in college, as I was applying to a very specific program within my major, and I wanted to meet one of the professors. I arrived and realized I had forgotten a pencil. I asked if I could borrow one so I could take some notes, and she looked at me like I suggested we step outside and barbecue a kitten.

    I did not get into that program.

    I went to law school instead.

  82. TootsNYC*

    her: Just checking.
    You, curiously: Why would you need to check? Are you in charge of some scheduling that no one’s told me about?

  83. Workfromhome*

    #1 There are a few ways you can go with this:

    1. When they ask you something that is none of their business simply do not respond.

    Did you check with the boss? silence/delete.

    2. Did you check with boss.? Why? Justs checking?
    No need to check anymore you can assume that I have followed all the processes.
    (Personally I might write You can assume I have it under control or know what I’m doing but depends on your environment)

    If they keep checking, then you can get more aggressive.
    Did you check with boss.? Why? Justs checking?
    “Why are you checking? I’ve told you there is no need to check.”

    3. Depending you might go to your boss and play dumb. Jane keeps asking me to confirm things that I regularly have been doing for years. Just wanted to check if there were any changes, I was not aware of .

    Next time Jane is just checking: “I talked to boss he specifically instructed me there is no need for anyone to check on these things so you dont need to spend any time checking.”

    Finally if it continues: Your checking is going against boss’s directive to me you do we need to set up a meeting with him together to confirm?

  84. Mimmy*

    #4 – I’ve always thought taking notes during interviews was discouraged. In fact, the career center at one of my universities recommended NOT taking notes during interviews except to jot down any names; I think they believe it distracts you from listening and being engaged. I think taking notes should be okay – maybe not copious sentences, but perhaps jot down a few keywords about the day-to-day work and answers to your questions. I sometimes have a hard time taking notes and listening simultaneously, but I try because it helps me remember the details I need to evaluate whether I’d be interested if I’m invited for another interview.

    1. Mimmy*

      That said, it should also be okay for a candidate to not have anything to take notes with. Everyone has their own way of taking in information. Some need to write things down, others can simply listen.

      As for bringing copies of your resume: I haven’t been to an in-person interview in ages, but I did sometimes bring an extra copy of my resume. Yes, the employer should be prepared and have my resume on hand, but sometimes things happen: the printer runs out of ink, the computers are down, etc. I believe in giving a little bit of grace (absent other red flags, of course).

  85. librarianmom*

    LW1 — Just because a question is asked doesn’t mean it requires you to answer. Develop a blind eye and a deaf ear —- just don’t “see” the email, don’t “hear” the question. If pressed, be vague and change the subject. Do that enough times the message will get through and eventually your rude coworker will figure out its a fruitless endeavor to ask nosy questions.

  86. Rachel*

    Letter writer 2: I feel so bad for your coworker. Thah is also sexual harassment. His sex life should be discussed at work ever.

  87. Marni*

    For number three, is there a good script for maybe talking to their manager about this to feel out the reaction? I’m thinking something like, “This job that was posted is exactly the kind of thing I’d like to move into in the future, can you give me advice on how to best position myself in my current job for that type of trajectory?“ That opens the door for the manager to encourage you to apply if they actually feel it would be appropriate, but that’s pretty unlikely. But it does set you up to hopefully move that direction as you continue with the company.

  88. peacheslcg*

    I have an interview tomorrow that goes from 8:30 -4:00 and is with 5 different groups of people/stakeholders. You bet your ass I am taking notes. I usually rely on my memory, but as I get older, writing down helps (especially in these all day slogs). I am also going to write down some questions for each group so that I don’t forget to ask anything

  89. high job offer rate*

    #4- Haven’t read all the comments. And I haven’t been an interviewee many times (been an interviewer more), but have received offers for all jobs I have interviewed, with the exception of one job that I withdrew from following the interview.

    When I am the interviewee, I take something to write on and write with. I usually don’t take notes, but I want to be able to if something comes up. For me, the importance of having a small notebook and a pen is that I prepare in advance questions that I want to ask. Sometimes I write the answers, but mainly I want to make sure I don’t forget my questions, if they did not come up during the interview.

  90. Hermione Danger*

    #3 – What about talking to your manager about the position as though it’s something you’d like to do in the future? If you say something like, “I’m really happy where I am now, but I saw this job posting come up, and eventually, it’s a role I’d love to take on. What would the path from here to there look like for me?” That will let your boss know what your goals are, and also could help you get information both about company norms for inside roles and timelines and the unwritten requirements for that specific job.

    #5 – From my understanding, bullet points should generally not be complete sentences anyway. They’re a way to deliver information quickly. The same context is understood for each bullet point in a list, so it doesn’t need to be included or repeated each time.

  91. three cheers for*

    Y’know, I’m not anti-notes, it’s just something I’ve never done (I’m mid-GenX). It’s a self-perpetuating thing, really — I’ve never done it, so it never occurs to me to do it, so I’ve never done it. Note-taking for me is somewhat of a distraction, and in other contexts where I’ve tried to stay engaged while taking notes, the end-result was half-indecipherable. If it’s important to you that candidates take notes, there any reason you can’t tell them to bring something when setting up the interview time? I’d imagine the more important takeaway would be “does this person listen to instructions?” rather than “does this person know about this thing that will annoy me if they don’t?”

  92. Jessica Fletcher*

    I think #2 should also make a little extra effort to be nice to Bill, to let him know not everyone is laughing at him. He must feel terrible. I would find it hard to come to work if I knew everyone was making fun of me.

  93. cncx*

    Just got off of a job search where the interviews were exclusively in my second and third language. Taking notes and specifically writing down key words and phrases was absolutely critical to me retaining information. I guess that’s my “cognitive limitation”

  94. Tea. Earl Grey. Hot.*

    #4 My job has branded pencils and notepads in every room – if anyone finds that they need to jot something down during a meeting, many of my colleagues just avail themselves of the supplies.

    #2 – That’s so shitty and awful. And thanks to my recently-completed compliance training, I can attest that you don’t need standing to report something like this – and you’d be doing your company a favor if you do.

    #1 – Years ago (Bill Clinton was still president to give you some scope), I was a college student eager to intern at my favorite magazine. I went to the meeting, and ignored my mom’s warning to bring a copy of my resume. I lost out on the internship as a result, so I’ve been mega paranoid about bringing multiple copies ever since. I got my current job during the pandemic and had a virtual interview, but I had it up on the screen while we were discussing and was prepared to share if needed. Necessary? Maybe not. But I still cringe all these years later when I think about how it felt to be asked for a copy and not have one.

  95. It is what it is*

    LW #1:
    Assuming all attempts to ignore the questions have failed, I would go with – As my peer, why do you ask? – Repeat as necessary. As your peer, you do not need to answer any questions in that vein.

    You can be polite but firm. As my peer, why do you ask? If you get the “just checking” response then you could say I deal with manager on things such as my schedule.

    If coworker continues to ask questions that are none of their business as a peer I would say coworker, as you are my peer, I am not willing to discuss xyz with you. Trust that I have spoken to manager about xyz and have their support.

    If they continue I would say coworker,you are not my manager/supervisor, please do not ask me these types of questions again. If you have concerns about how my work/schedule directly affects your work please speak to manager. I will not discuss this with you further.

    You may want to give manager a heads up about what has been going in with coworker and that you’re handling it by xyz. But you aren’t obligated to say anything I would probably give a heads up but that’s just me.

    Also, make certain you aren’t asking other coworkers these types of questions because nosy coworker will be watching.

  96. Pip*

    Re LW 2: Absolutely LW should speak up about the horrible and discriminatory way Bill is being treated, but if I were OP and even Bill, I’d be looking for a work environment where no one needs to be told they can’t discuss and/or ridicule another employee’s sex life. It’s like having to tell people they can’t pee on the carpet. I truly hope his co-workers are made to stop their egregious behavior, but I can’t imagine Bill being happy at that company again. He can’t un-hear whatever he’s heard.

  97. JustaTech*

    Uh, I take all my notes on paper (in work meetings, in non-work meetings, at the doctor’s office, to-do lists).
    I don’t do this because I “haven’t progressed” but because 1) if I don’t write it down I won’t remember, 2) I personally write faster than I type in a note-taking situation, 3) for the way my brain works phones and computers are just giant boxes of distractions, where paper doesn’t offer enticements like email or chat.

    I also don’t criticize my coworkers who take notes on their computers in meetings, or don’t take notes at all.
    Taking notes on paper is how I stay engaged in a meeting, but I know it’s not the same for everyone.

  98. Sarah FW*

    Re: #4 –
    I stopped bringing copies of my resume 10+ years ago, because every interviewer had it pulled up on their computer when I came in (or in my current job of a group interview with owners, the HR director printed copies for everyone). I also never anticipate taking notes but have occasionally pulled out my phone if there was something specific I promised to send them or some other action step to remember. It never occurred to me this would be seen as being unprepared (and I’ve never gotten any hint anyone thought of me that way).

    That said, I apply/interview pretty selectively and have only interviewed once without getting the job (and that we both openly agreed it wasn’t the right fit) so maybe I’m super weird?

    1. Sarah FW*

      Adding: I do take paper notes in meetings/on phone calls and keep a paper to-do list in a planner on my desk.

      1. I Wrote This in the Bathroom*

        I do too. I like to completely disconnect my brain from work at the end of the day, and coming back to a to-do list for the next day the next morning really helps.

  99. Anonosaurus*

    For LW#1 situations, I’ve found it can sometimes work to say something along the lines of “good job you’re not my boss, I’d be getting my pink slip in no time!” with sufficient edge to get the point across. of course it should be ok to be direct and say “stop checking up on me, it’s unnecessary and rude” but I’ve worked in many offices where that would trigger large quantities of drama so I tend to favour the sharply pointed joke approach. or you could just ignore her – what’s she going to do? complain to your boss that you aren’t reporting back to her?

  100. Janeric*

    LW1 — I love that Alison’s advice for an intrusive coworker mirrors the parenting advice for nervous children.

  101. Well...*

    Some UK funding agencies have been pushing for narrative CVs lately, where you describe your contributions rather than list them. I had to rewrite my CV this way for a grant application. It was annoying, but I’ve been told it’s to elucidate your contributions more clearly for equity purposes (you can do that with bullet points, but not everyone will unprompted).

  102. Daisy-dog*

    Whoa, 700+ comments. I would have thought they’d be for #2 or #3, but mostly #4 and #5! Surprised, but not as much because I guess we all have experience with interviews and resumes.

    1. I Wrote This in the Bathroom*

      Yeah, I came prepared to fight for Bill and was distracted by talk of notepads and resumes.

      1. JustaTech*

        Let’s be honest, who in the comments here is going to actually say “yeah, pick on Bill!”?

        When everyone who comments agrees then it doesn’t generate as many comments.

        1. metadata minion*

          Yeah, but with something this egregious there’s often a flood of “Woah, that’s not ok!”.

          1. Daisy-dog*

            Exactly, just an army of supporters. There’s still plenty of support, but it’s overrun hahaha

  103. ofotherworlds*

    I didn’t know that I was supposed to bring along something to take notes on when I’m being interviewed for a white collar job. I guess that means that I need to bust out the cheap Amazon tablet. Using my phone to take notes has been mistaken for texting or not paying attention, pencil and paper notes by me would be illegible and hurt my hands, and I can’t count on having a surface where I could set my laptop, which is what I would prefer to take notes on. (I do know to bring paper copies of my resume.)

  104. basementmatt*

    I am someone who doesn’t take notes. I didn’t take them in high school, and I don’t take them now 40 years later, at work. The times I’ve tried to take notes, I end up just listening and absorbing. It would never occur to me to take a notepad to an interview. And I’d judge someone for judging me for the lack.

  105. Expelliarmus*

    Hasn’t Alison mentioned on here multiple times that if anything, bringing your own resume to the interview is good prep for in case your recruiter (if you’re using one) changed your resume for the worse before sending it to the interviewer? And that bringing your own is a good way to indicate that you’re being honest about your own qualifications. So at least for the sake of that, it wouldn’t hurt to have an extra copy or two on hand.

  106. overheard*

    #3
    In my first job, I started together with another person on the same team, same position, same day. A few weeks in, he told our manager that he actually wanted to switch to another position on another team within our same company, because that would suit him better.
    I knew this because I overheard our manager discuss it with another manager (not the one for the new role) and both seemed to agree this has lowered their opinion of said new employee and they regretted hiring him at all.
    (I was not evesdropping deliberately, it was an open office and only one desk was between me and their not that quit conversation, many of my colleagues heard as well. The discussed colleague was not in the office that day. Obviously, that was not a good decision of those managers to discuss quite so openly)
    They did support him in the move (I think mostly because they didn’t want him in the team anymore) and he did get the internal transfer to the position he wanted, where he did fine for a few years until he moved to another company. But I’m pretty sure he burned that bridge with the original manager, which he might not realise himself.

  107. BridgeofFire*

    For LW #3, my workplace actually has a policy about transferring internally. After starting a new position, whether it’s as a fresh hire or a new transfer, it’s a minimum of 3 months before you can transfer intra-department (for example, going from a chef at the food court to a chef at our on-site fine dining restaurant), or 6 months for an inter-department transfer (such as going from a custodian to security), and this is a company/organization that prides itself on allowing room for workers to grow and find their comfort zones. Just giving my own personal experience to show what expectations could be on that front, based on some anecdotal data.

  108. GlitterIsEverything*

    LW 1, I’d be really tempted to either ignore the question altogether, or reply with “That’s a conversation between myself and Boss.” If they pushed for more after, I’d confront it directly. That might sound like “Have I given you a reason to think that I’m not communicating with Boss on these kinds of things?”

    1. Unkempt Flatware*

      I’d never answer but continue telling her, “Jane, please stop asking me questions like that”. That would be the only reply she’d get from me.

  109. Avery*

    Adding to the anecdotes (let’s face it, that’s what these replies generally are) about what’s brought to interviews…
    I always bring a few copies of my resume, and there have been several times that that has come in handy for me, whether because I needed to reference it myself or because some/all interviewers didn’t have copies of their own. I only saw the latter as a sign of the company being disorganized/unprepared once, when it was clearly part of a bigger problem (my interviewer hadn’t known he was doing the interview or what the position I was interviewing for was, let alone anything about me as a candidate).
    I usually bring a pen and paper, but rarely use them as more than a fidget tool/positive signal. I’m the type of ADHD where trying to take notes during the meeting would just distract me. Never had any interviewer comment on this practice either way.
    And for the record, I’m American, Millennial, and went from general admin work to the legal field.

  110. Danielle*

    I work in academia and I’d say it’s pretty common for our job candidates not to take notes— which may seem surprising, since these are people applying for jobs as professors! I remember during my last one, I recommended a source that the candidate might want to check out for a paper they’re writing, and they specifically reached over and took a pad out of their bag to jot down that info. I feel like it’s just not as much of a “thing” as it maybe once was— with the caveat that academia, as always, could just be its own special goat rodeo.

  111. SavvyLibrarian*

    On the notes front: some folks just have a good memory. I am one of these people. In general I prefer not to take notes because I am more present, more engaged, and I remember more if I am not taking notes.

    If I take notes I tend to miss important things, forget what I want to say, and have less time to think about my responses to questions. Notes should not be required for everyone and not taking them us not always a sign of inexperience or a person who doesn’t know better. Not everyone’s brain works the same way nor do they need to.

  112. Grizabella the Glamour Cat*

    On the nosy coworker, I like “Why do you ask?” or just “Why?” I also came up with another possible variation: “Why do you need to know?”

    Coworker: Did [Boss] approve your schedule change?
    LW: Why do you need to know?
    Coworker: Just checking!
    LW: But why do you need to know?
    Etc.

    I like this because it really puts Nosy Parker on the spot to explain herself. I might not use it right off the bat, but pull it out if “Why do you ask?” doesn’t get the desired result. If she continues to be obnoxiously persistent, maybe change it up a little:

    Coworker: Did [Boss] approve your schedule change?
    LW: Why do you need to know?
    Coworker: Just checking!
    LW: Oh, so do you don’t really need to know. Got it!

    It never hurts to have alternate scripts ready for use. ;-p

  113. parsley*

    LW 5, I proofread CVs in the UK for recruiters for a living and I also remove all pronouns as standard.

  114. Rika*

    LW4. Dutchie here. I’ve been on the workforce for 21 years and have worked for several companies. I’ve never heard the advice of bringing something to take notes on to an interview, but I expect if I interviewed someone who brought a notepad I would take it as a sign of nervousness, not necessarily preparation. Not that it would count against them.

  115. AliJJ83*

    LW4 – I can’t speak for the US, but in the UK it’s no longer considered standard for candidates to take notes in interviews or bring any documents unless specifically specified, e.g. my last interview, I was asked to take my certificates in, so that’s all I took, didn’t take notes, and I got the job. I wouldn’t even think to take my CV in as I would expect the interviewer to have it and be aware of what’s on there; if they didn’t, I would consider it a red flag. But if you’re US-based, it could just be that norms are changing.

    LW5 – it seems the US way is the same as the UK; I would never use first person pronouns on my CV, mostly because it makes it too wordy and when looking at a CV at first glance, most employers seem to want to get the gist of skills and experience. So yeah, the standard convention is not to use pronouns.

Comments are closed.